OBM - Teoría de Números.pdf

107
Polos Olímpicos de Treinamento Curso de Teoria dos Números - Nível 2 Samuel Barbosa Feitosa Aula 1 Divisibilidade I Teorema 1. (Algoritmo da Divis˜ao) Para quaisquerinteiros positivos a e b, existe um ´ unico par (q,r) de inteiros n˜ ao negativos tais que b = aq + r e r<a. Os n´ umeros q e r s˜ao chamados de quociente e resto, respectivamente, da divis˜ao de b por a. Exemplo 2. Encontre um n´ umero natural N que, ao ser dividido por 10, deixa resto 9, ao ser dividido por 9 deixa resto 8, e ao ser dividido por 8 deixa resto 7. O que acontece ao somarmos 1 ao nosso n´ umero? Ele passa a deixar resto 0 na divis˜ao por 10, 9 e 8. Assim, um poss´ ıvel valor para N ´ e 10 · 9 · 8 1. Exemplo 3. a) Verifique que a n 1=(a 1)(a n-1 + a n-2 + ... + a + 1) b) Calcule o resto da divis˜ao de 4 2012 por 3. Para o item a), usando a distributividade e efetuando os devidos cancelamentos no lado direito, podemos escrever: a n + a n-1 + ... + a 2 + a a n-1 a n-2 ... a 1= a n 1. Para o item b), veja que 3=4 1 e assim ´ e natural substituir os valores dados na express˜ ao do primeiro item: 4 2012 1 = 3(4 2011 + ... + 4 + 1). Isso significa que q = (4 2011 + ... + 4 + 1) e que r =1. Observa¸c˜ ao 4. O teorema anterior admite um enunciado mais geral: Para quaisquer intei- ros a e b, com a =0, existe um ´ unico par de inteiros (q,r) tais que b = aq + r, 0 r< |a|. Por exemplo, o resto da divis˜ao de 7 por 3 ´ e 2 e o quociente ´ e 3. Iremos agora estudar propriedades a respeito das opera¸c˜oes com restos. Teorema 5. (Teorema dos Restos) Se b 1 e b 2 deixam restos r 1 e r 2 nadivis˜aopor a,respectivamente, ent˜ao:

Transcript of OBM - Teoría de Números.pdf

Page 1: OBM - Teoría de Números.pdf

Polos Olímpicos de TreinamentoCurso de Teoria dos Números - Nível 2Samuel Barbosa Feitosa

Aula 1

Divisibilidade I

Teorema 1. (Algoritmo da Divisao) Para quaisquer inteiros positivos a e b, existe um unicopar (q, r) de inteiros nao negativos tais que b = aq + r e r < a. Os numeros q e r saochamados de quociente e resto, respectivamente, da divisao de b por a.

Exemplo 2. Encontre um numero natural N que, ao ser dividido por 10, deixa resto 9, aoser dividido por 9 deixa resto 8, e ao ser dividido por 8 deixa resto 7.

O que acontece ao somarmos 1 ao nosso numero? Ele passa a deixar resto 0 na divisao por10, 9 e 8. Assim, um possıvel valor para N e 10 · 9 · 8− 1.

Exemplo 3. a) Verifique que an − 1 = (a− 1)(an−1 + an−2 + . . .+ a+ 1)

b) Calcule o resto da divisao de 42012 por 3.

Para o item a), usando a distributividade e efetuando os devidos cancelamentos no ladodireito, podemos escrever:

an + an−1 + . . .+ a2 + a− an−1 − an−2 − . . .− a− 1 = an − 1.

Para o item b), veja que 3 = 4−1 e assim e natural substituir os valores dados na expressaodo primeiro item:

42012 − 1 = 3(42011 + . . .+ 4 + 1).

Isso significa que q = (42011 + . . .+ 4 + 1) e que r = 1.

Observacao 4. O teorema anterior admite um enunciado mais geral: Para quaisquer intei-ros a e b, com a 6= 0, existe um unico par de inteiros (q, r) tais que b = aq+ r, 0 ≤ r < |a|.Por exemplo, o resto da divisao de −7 por −3 e 2 e o quociente e 3.

Iremos agora estudar propriedades a respeito das operacoes com restos.

Teorema 5. (Teorema dos Restos) Se b1 e b2 deixam restos r1 e r2 na divisao por a,respectivamente,entao:

Page 2: OBM - Teoría de Números.pdf

POT 2012 - Teoria dos Numeros - Nıvel 2 - Aula 1 - Samuel Feitosa

b1 + b2 deixa o mesmo resto que r1 + r2 na divisao por a

b1b2 deixa o mesmo resto que r1r2 na divisao por a.

Demonstracao. Por hipotese, existem q1, q2 e q tais que: b1 = aq1 + r1, b2 = aq2 + r2 er1 + r2 = aq + r, logo:

b1 + b2 = a(q1 + q2 + q) + r.

Como 0 < r < |a|, b1 + b2 deixa resto r quando dividido por a. A demonstracao para oproduto e deixada ao cargo do leitor.

Observacao 6. Em alguns casos, e preferıvel que o professor faca uma demonstracao doresultado anterior para a = 3 ou a = 5 apenas com o intuito de deixar os alunos maisconfortaveis a respeito do resultado. E preferıvel que mais tempo seja gasto resolvendoexemplos e problemas. Na secao de congruencias, os alunos terao um contato mais apro-priado com o enunciado anterior.

Exemplo 7. Qual o resto que o numero 1002 · 1003 · 1004 deixa quando dividido por 7?

Como 1002 deixa resto 1 por 7, o numero acima deixa o mesmo resto que 1 · 2 · 3 = 6 por 7.

Exemplo 8. Qual o resto que o numero 45000 deixa quando dividido por 3?

Como 4 deixa resto 1 por 3, 45000 deixa o mesmo resto que 1 · 1 · . . . · 1︸ ︷︷ ︸

5000

= 1 por 3.

Exemplo 9. Qual o resto que o numero 22k+1 deixa quando dividido por 3?

Note que 20 deixa resto 1 por 3, 21 deixa resto 2 por 3, 22 deixa resto 1 por 3, 23 deixaresto 2 por 3, 24 deixa resto 1 por 3. Precebeu alguma coisa? Como 100 e par, o restodevera ser 1. Como 22 deixa resto 1, entao 22k = 22 · 22 · . . . · 22

︸ ︷︷ ︸

k

deixa o mesmo resto que

1 · 1 · . . . · 1︸ ︷︷ ︸

k

= 1 e 22k+1 = 22k · 2 deixa o mesmo resto que 1 · 2 = 2 por 3.

Exemplo 10. Qual o resto de n3 + 2n na divisao por 3?

Se o resto de n por 3 e r, o resto de n3 + 2n e o mesmo de r3 + 2r. Para r = 0, esseresto seria 0. Para r = 1, seria o mesmo resto de 3 que e 0. Finalmente, para r = 2, oresto seria o mesmo de 8 + 4 = 12 que tambem e 0. Assim, nao importa qual o resto de n

por 3, o numero n3 + 2n sempre deixara resto 0. Uma ideia importante nessa solucao foidividı-la em casos. Tambem poderıamos ter resolvido esse exemplo apelando para algumafatoracao:

n3 + 2n = n3 − n+ 3n = n(n2 − 1) + 3n = n(n− 1)(n+ 1) + 3n.

Como n−1, n e n+1 sao consecutivos, um deles e multiplo de 3. Assim, o ultimo termo daigualdade anterior e a soma de dois multiplos de 3 e consequentemente o resto procuradoe 0.

2

Page 3: OBM - Teoría de Números.pdf

POT 2012 - Teoria dos Numeros - Nıvel 2 - Aula 1 - Samuel Feitosa

Observacao 11. Fatoracoes podem ser muito uteis para encontrarmos os valores explıcitosde q e r.

Exemplo 12. Prove que, para cada n natural,

(n+ 1)(n+ 2) . . . (2n)

e divisıvel por 2n.

Veja que

(n+ 1)(n+ 2) . . . (2n) =1 · 2 · · · 2n

1 · 2 · · ·n.

Para cada numero natural k no produto escrito no denominador, temos uma aparicao de2k no produto escrito no numerador. Basta efetuarmos os cancelamentos obtendo:

(n+ 1)(n+ 2) . . . (2n) = 2n · 1 · 3 · · · (2n− 1).

Exemplo 13. (Olimpıada de Leningrado 1991) Cada um dos naturais a, b, c e d e divisıvelpor ab− cd, que tambem e um numero natural. Prove que ab− cd = 1.

Se chamarmos p = ab − cd, teremos a = px, b = py, c = pz e d = pt onde x, y, z e t saointeiros. Assim, p = p2(xy−zt). Consequentemente 1 = p(xy−zt) e concluımos que p = 1,pois p e natural.

Exemplo 14. A soma digital D(n) de um inteiro positivo n e definida recursivamente comosegue:

D(n) =

{n se 1 ≤ n ≤ 9,D(a0 + a1 + . . .+ am) se n > 9,

onde a0, a1, . . . , am sao todos os dıgitos da expressao decimal de n na base 10, i.e.,

n = am10m + am−110m−1 + . . .+ a110 + a0

Por exemplo, D(989) = D(26) = D(8) = 8. Prove que: D((1234)n) = D(n), para n =1, 2, 3 . . .

Como 10n−1n = (10−1)(10n−1+10n−2+ . . .+1), podemos concluir que 10n sempre deixaresto 1 na divisao por 9. Assim, n = am10m+am−110

m−1+ . . .+a110+a0, deixa o mesmoresto que am + am−1 + . . . + a0 na divisao por 9. Desse modo, D(n) nada mais e do queo resto na divisao por 9 do numero n. Como 1234 deixa resto 1 por 9, o numero (1234)ndeixa o mesmo resto que 1 · n por 9, ou seja, D((1234)n) = D(n).

Observacao 15. O exemplo anterior contem o criterio de divisibilidade por 9, i.e., n deixao mesmo resto que D(n) na divisao por 9. O criterio de divisibilidade por 3 e analogo pois10n tambem sempre deixa resto 1 por 3.

Exemplo 16. Encontre todos os pares de inteiros positivos a e b tais que 79 = ab+2a+3b.

3

Page 4: OBM - Teoría de Números.pdf

POT 2012 - Teoria dos Numeros - Nıvel 2 - Aula 1 - Samuel Feitosa

Fatoremos a expressao anterior. Somando 6 aos dois lados da equacao, obtemos:

85 = 6 + ab+ 2a+ 3b

= (3 + a)(2 + b)

Assim, (3 + a) e (2 + b) sao divisores positivos de 85 maiores que 1. Os unicos divisorespositivos de 85 sao 1, 5, 19, 85. Logo, os possıveis pares de valores para (3 + a, 2 + b) sao(5, 19) ou (19, 5) que produzem as solucoes (a, b) = (2, 17) e (16, 3).

Problema 17. (Olimpıada Russa) Prove que se2n − 2

ne um inteiro, entao

22n−1 − 2

2n − 1tambem e um inteiro.

Se k =2n − 2

n, entao

22n−1 − 2

2n − 1=

2(22n−2 − 1)

2n − 1

= 2

(2nk − 1

2n − 1

)

= 2

(

(2n − 1)(2n(k−1) + 2n(k−2) + . . .+ 2n + 1)

2n − 1

)

= 2(2n(k−1) + 2n(k−2) + . . .+ 2n + 1),

e um numero inteiro.

Problemas Propostos

Problema 18. Encontre os inteiros que, na divisao por 7, deixam um quociente igual aoresto.

Problema 19. Determinar os numeros que divididos por 17 dao um resto igual ao quadradodo quociente correspondente.

Problema 20. (OCM 1985) Encontre o quociente da divisao de a128 − b128 por

(a64 + b64)(a32 + b32)(a16 + b16)(a8 + b8)(a4 + b4)(a2 + b2)(a+ b)

Problema 21. (OCM 1994) Seja A = 777 . . . 77 um numero onde o dıgito ”7”aparece 1001vezes. Determinar o quociente e o resto da divisao de A por 1001.

Problema 22. Encontre um inteiro que deixa resto 4 na divisao por 5 e resto 7 na divisaopor 13

Problema 23. Encontre o menor inteiro que, dividido por 29 deixa resto 5, e dividido por31 da resto 28.

4

Page 5: OBM - Teoría de Números.pdf

POT 2012 - Teoria dos Numeros - Nıvel 2 - Aula 1 - Samuel Feitosa

Problema 24. Prove que, para todo inteiro positivo n o numero n5 − 5n3 + 4n e divisıvelpor 120.

Problema 25. (Fatoracoes Importantes)

a) Seja S = 1+ z+ z2+ z3+ . . .+ zn−1. Veja que S+ zn = 1+ zS entao S(z−1) = zn−1.Conclua que, para quaisquer x e y vale:

xn − yn = (x− y)(xn−1 + xn−2y + xn−3y2 + . . .+ x2yn−3 + xyn−2 + yn−1)

b) Mostre que se n e ımpar vale:

xn + yn = (x+ y)(xn−1 − xn−2y + xn−3y2 − . . .+ x2yn−3 − xyn−2 + yn−1)

Problema 26. Prove que, o numero 199 + 299 + 399 + 499 + 599 e multiplo de 5.

Problema 27. Mostre que o numero 1n + 8n − 3n − 6n e multiplo de 10 para todo naturaln.

Problema 28. Encontre o resto da divisao 3710 − 1 por 11.

Problema 29. Prove que 22225555 + 55552222 e divisıvel por 7.

Problema 30. Encontre o ultimo dıgito do numero 19891989.

Problema 31. Mostre que se n divide a entao 2n − 1 divide 2a − 1.

Problema 32. (Cone Sul 1996) Provar que o numero

1995 · 19971996 − 1996 · 19971995 + 1

19962

e um inteiro.

Problema 33. Mostre que para n ımpar, n divide 1n + 2n + . . .+ (n− 1)n

Problema 34. Existe um natural n tal que nn + (n+ 1)n e divisıvel por 2011?

Problema 35. Quantos numeros inteiros positivos n existem tais que n+ 3 divide n2 + 7?

Problema 36. Encontre o numero de inteiros n tais que

1. 1000 < n < 8000.

2. nn+1 + (n+ 1)n e divisıvel por 3.

Problema 37. Sejam m e n naturais tais que mn+ 1 e multiplo de 24, mostre que m+ n

tambem e multiplo de 24.

5

Page 6: OBM - Teoría de Números.pdf

POT 2012 - Teoria dos Numeros - Nıvel 2 - Aula 1 - Samuel Feitosa

Problema 38. (Irlanda 1997) Encontre todos os pares de inteiros (x, y) tais que 1+1996x+1998y = xy.

Dicas e Solucoes

18. Os numeros sao {0, 8, 16, 24, . . . , 8 · 7}.

18. Escreva n = 17q + q2 e note que 0 ≤ q2 < 17. Assim, q = 0, 1, 2, 3, 4.

19. Use a diferenca de quadrados sucessivas vezes para obter (a− b) como quociente.

21. O numero do problema e igual a 7(101001−1)9 . Alem disso, 10999+1

103+1e inteiro e 101001−1

103+1=

100 · 10999+1103+1

− 100103+1

.

22. Os numeros que satisfazem essa propriedade sao os numeros da forma 65k + 59.

24. Basta mostrar que n5 − 5n3 + 4n e multiplo de 3, 8 e 5. Na divisao por 5, temosquatro restos possıveis: {0, 1, 2, 3, 4}. Assim, o numero n5−5n3+4n possui o mesmoresto na divisao por 5 que um dos cinco numeros: {05− 5 · 03+40, 15− 5 · 13+4, 25−5 · 23 + 8, 35 − 5 · 33 + 12, 45 − 5 · 43 + 16}. Como todos esses numeros sao multiplosde 5, segue que n5 − 5n3 + 4n e multiplo de 5 para todo n inteiro. O procedimentocom 3 e 8 e semelhante.

25. Para o item a), troque z por xy. Para o item b), substitua y por −y no item anterior.

26. Pelo problema anterior, como 99 e ımpar temos: 199 + 499 = (1 + 4)(198 + 197 · 4 +. . .+ 1 · 497 + 498). Daı, segue que 199 + 499 e multiplo de 5. Analogamente podemosmostrar que 299 + 399 e multiplo de 5.

27. O numero em questao e mutiplo de 2 pois e a soma de dois ımpares e dois pares.Para ver que tambem e multiplo de 5, basta notar que 5 divide 1n − 6n e 8n − 3n.Isso pode ser facilmente mostrado usando a fatoracao do exercıcio 25.

31. Se a = nk, temos (2n − 1)(2n(k−1) + 2n(k−2) + . . .+ 2n + 1) = 2nk − 1.

32. Veja que 1995·19971996−1996·19971995+1 = 1995·(19971996−1)−1996·(19971995−1).Pela fatoracao de xn − yn,

1996 · (19971995 − 1)

19962= (19971994 + 19971993 + . . .+ 1),

e inteiro. Alem disso, pela mesma fatoracao,

1995 · (19971996 − 1)

19962= 1995·

(19971995 − 1

1996+

19971994 − 1

1996+ . . .+

1997− 1

1996+

1996

1996

)

,

e uma soma de numeros inteiros.

6

Page 7: OBM - Teoría de Números.pdf

33. Como n e impar,

(n− i)n + in = ((n− i) + i)((n− i)n−1 − (n− i)n−2i+ . . .− (n− i)in−2 + in−1).

34. Faca n = 1005 e use a fatoracao de xn + yn.

37. Fatore a expressao como:

(x− 1998)(y − 1996) = xy − 1998y − 1996x+ 1998 · 1996 = 19972.

Os divisores de 19972 sao {±1,±1997,±19972}. Resolvendo os sistemas correspon-dentes a essas possibilidades, temos: (x, y) = (1999, 19972 + 1996), (1997,−19972 +1996), (3995, 3993), (1,−1), (19972 + 1998, 1997), (−19972 + 1998, 1995).

Referencias

[1] F. E. Brochero Martinez, C. G. Moreira, N. C. Saldanha, E. Tengan - Teoria dosNumeros um passeio com primos e outros numeros familiares pelo mundo inteiro,Projeto Euclides, IMPA, 2010.

[2] E. Carneiro, O. Campos and F. Paiva, Olimpıadas Cearenses de Matematica 1981-2005(Nıveis Junior e Senior), Ed. Realce, 2005.

[3] S. B. Feitosa, B. Holanda, Y. Lima and C. T. Magalhaes, Treinamento Cone Sul 2008.Fortaleza, Ed. Realce, 2010.

[4] D. Fomin, A. Kirichenko, Leningrad Mathematical Olympiads 1987-1991, MathProPress, Westford, MA, 1994.

[5] D. Fomin, S. Genkin and I. Itenberg, Mathematical Circles, Mathematical Words, Vol.7, American Mathematical Society, Boston, MA, 1966.

[6] I. Niven, H. S. Zuckerman, and H. L. Montgomery, An Introduction to the Theory ofNumbers.

Page 8: OBM - Teoría de Números.pdf

Polos Olímpicos de TreinamentoCurso de Teoria dos Números - Nível 2Prof. Samuel Feitosa

Aula 2

Divisibilidade II

Definicao 1. Dados dois inteiros a e b, com a 6= 0, dizemos que a divide b ou que a eum divisor de b ou ainda que b e um multiplo de a e escrevemos a | b se o r obtido peloalgoritmo de divisao aplicado a a e b e 0, ou seja, se b = aq para algum inteiro q.

Lema 2. Sejam a, b, c, d inteiros. Temos

i) (”d divide”) Se d | a e d | b, entao d | ax+ by para quaisquer x e y inteiros.

ii) (”Limitacao”) Se d | a, entao a = 0 ou |d| ≤ |a|.

iii) (Transitividade) Se a | b e b | c, entao a | c.

Em particular, segue da propriedade i) que d | a+ b e d | a− b.

Exemplo 3. (Olimpıada de Maio 2006) Encontre todos os naturais a e b tais que a|b+ 1 eb|a+ 1.

Pela propriedade da Limitacao, temos a ≤ b + 1 e b ≤ a + 1. Daı, a − 1 ≤ b ≤ a + 1.Vejamos os casos:

(i) a = b. Como a|b+ 1 e a | b(pois b = a) temos que a | [(b+ 1)− b] = 1. Assim, a = 1Nesse caso, so temos a solucao (a, b) = (1, 1)

(ii) a = b+ 1. Como b|a+ 1 e b|a− 1(pois b = a− 1) temos que b|[(a+ 1)− (a− 1)] = 2.Assim, b = 1 ou b = 2 e nesse caso, so temos as solucoes (3, 2) e (2, 1).

(iii) a = b− 1. Esse caso e analogo ao anterior e as solucoes para (a, b) sao (1, 2) e (2, 3).

Exemplo 4. (Criterio de Divisibilidade por 7) Existem alguns metodos praticos para deci-dirmos se um numero e multiplo de outro. Certamente o leitor ja deve ter se deparado comalgum criterio de divisibilidade. Existe um criterio por 7 bastante popular: Para saber seum inteiro e multiplo de 7, basta apagar seu ultimo dıgito, multiplica-lo por 2 e o subtrairdo numero que restou. Se o resultado e multiplo de 7, entao o numero original tambem emultiplo de 7.

Page 9: OBM - Teoría de Números.pdf

POT 2012 - Teoria dos Numeros - Nıvel 2 - Aula 1 - Samuel Feitosa

Podemos aplicar esse algoritmo sucessivas vezes ate que o resultado obtido seja facil-mente verificavel como um multiplo de 7. Por exemplo, para o numero 561421 podemosescrever:

56142− 2 = 56140

5614− 0 = 5614

561− 8 = 553

55− 6 = 49

Como 49 e multiplo de 7, nosso numero original tambem e. Por que esse processo funciona?Se o nosso numero original esta escrito na forma 10a + b, entao o numero obtido apos aoperacao descrita e a − 2b. Basta mostrarmos que se 7 | a − 2b, entao 7 | 10a + b. Se7 | a − 2b, pela propriedade (i) do lema, concluımos que 7 | 10a − 20b. Como 7 | 21b,tambem temos que 7 | [(10a− 20b) + 21b] = 10a+ b.

Exemplo 5. Mostre que se 7 | 3a+ 2b entao 7 | 4a− 2b.

Veja que 7 | 7a e 7 | 3a + 2b, entao 7 | [7a − (3a + 2b)] = 4a − 2b. Na pratica, o quefizemos foi multiplicar o numero 3a+2b por algum inteiro para posteriormente subtraımosum multiplo de 7 conveniente e obtermos o numero 4a − 2b. Existem outras formas defazermos isso. Observe os numeros 3 · 0, 3 · 1, 3 · 2, 3 · 3, 3 · 4, 3 · 5, 3 · 6. O numero 3 · 6 deixao mesmo resto que 4 por 7, pois 3 · 6 = 7 · 2 + 4. Como 7|3a + 2b podemos concluir que7|(18a + 12b) e consequentemente 7 | [18a + 12b − 14a)] = 4a + 12b. Mas 7 | 14b, entao7 | [4a+ 12b− 14b] = 4a− 2b.

Para o proximo exemplo, o leitor precisara lembrar dos criterios de divisibilidade por 9e 3 vistos na aula passada.

Exemplo 6. Usando os dıgitos 1, 2, 3, 4, 5, 6, 7, construımos varios numeros de sete dıgitosdistintos. Existem dois deles, distintos, tais que um divide o outro?

Nao. Suponha, por absurdo, que m < n sejam dois desses numeros, com m | n. Claramentem | n−m e 9 | n−m, pois n e m possuem a mesma soma dos dıgitos e consequentementepossuem o mesmo resto na divisao por 9. Por outro lado, sabemos a soma dos dıgitos dem: 1+2+ · · ·+7 = 3 ·9+1. Daı, m nao possui fator 9 e podemos garantir que 9m | n−m.Mas entao 9m ≤ n−m ⇒ 10m ≤ n ⇒ n tem pelo menos oito dıgitos, uma contradicao.

Exemplo 7. (Leningrado 1989) Seja A um numero natural maior que 1, e seja B umnumero natural que e um divisor de A2 + 1. Prove que se B −A > 0, entao B −A >

√A.

Seja B − A = q. Assim, A + q | A2 + 1. Como (A − q)(A + q) = A2 − q2 e divisıvel porA+ q, podemos concluir que A+ q | [(A2 + 1)− (A2 − q2)] = q2 + 1. Pela propriedade delimitacao, A+ q ≤ q2 +1. Nessa desigualdade, nao podemos ter q = 1 pois A > 1. Usandoentao que q > 1, temos A ≤ q2 − q + 1 < q2, ou seja,

√A < q.

Problema 8. (AIME 1986) Qual e o maior inteiro n para o qual n3 + 100 e divisıvel porn+ 10?

2

Page 10: OBM - Teoría de Números.pdf

POT 2012 - Teoria dos Numeros - Nıvel 2 - Aula 1 - Samuel Feitosa

Para achar explicitamente o quociente de n3+100 por n+10 podemos fazer uso de algumafatoracao. Utilizaremos a soma dos cubos n3 + 103 = (n+ 10)(n2 − 10n+ 100). Como,

n3 + 100 = (n+ 10)(n2 − 10n+ 100)− 900,

podemos concluir que o numero 900 deve ser multiplo de n+ 10. O maior inteiro n para oqual n+ 10 divide 900 e 890. Veja que se n = 890, o quociente da divisao de n3 + 100 porn+ 10 e n2 − 10n+ 100− 1 = 8902 − 10 · 890 + 99.

Exemplo 9. (Extraıdo de [1]) Encontre todos os inteiros positivos n tais que 2n2 + 1 |n3 + 9n− 17.

Utilizando o “2n2 + 1 divide” para reduzir o grau de n3 + 9n− 17, temos que

{

2n2 + 1 | n3 + 9n− 17

2n2 + 1 | 2n2 + 1

=⇒ 2n2 + 1 | (n3 + 9n− 17) · 2 + (2n2 + 1) · (−n)

⇐⇒ 2n2 + 1 | 17n− 34

Como o grau de 17n − 34 e menor do que o de 2n2 + 1, podemos utilizar a “limitacao”para obter uma lista finita de candidatos a n. Temos 17n − 34 = 0 ⇐⇒ n = 2 ou|2n2 + 1| ≤ |17n − 34| ⇐⇒ n = 1, 4 ou 5. Destes candidatos, apenas n = 2 e n = 5 saosolucoes.

Exemplo 10. (Leningrado 1990) Sejam a e b numeros naturais tais que b2 + ba+ 1 dividea2 + ab+ 1. Prove que a = b.

Pela propriedade de limitacao, b2+ba+1 ≤ a2+ab+1 e daı b ≤ a. Alem disso, b2+ab+1 >

a− b. A igualdade b(a2 + ab+ 1)− a(b2 + ba+ 1) = b− a implica que a− b e divisıvel porb2 + ba+1. Se a− b 6= 0, entao b2 + ab+1 ≤ a− b. Mas isso e um absurdo, logo a− b = 0.

Problemas Propostos

Problema 11. Mostre que se 3 | a+ 7b entao 3 | a+ b.

Problema 12. Mostre que se 7 | a+ 3b entao | 13a+ 11b

Problema 13. Mostre que se 19 | 3x+ 7y entao 19 | 43x+ 75y

Problema 14. Mostre que se 17 | 3a+ 2b entao 17 | 10a+ b

Problema 15. Encontre todos os inteiros positivos n tais que n+ 2009 divide n2 + 2009 en+ 2010 divide n2 + 2010.

Problema 16. Seja n > 1 e k um inteiro positivo qualquer. Prove que (n− 1)2|(nk − 1) se,e somente se , (n− 1)|k.

3

Page 11: OBM - Teoría de Números.pdf

POT 2012 - Teoria dos Numeros - Nıvel 2 - Aula 1 - Samuel Feitosa

Problema 17. (OBM 2005) Prove que a soma 1k + 2k + . . .+ nk, onde n e um inteiro e k

e ımpar, e divisıvel por 1 + 2 + . . .+ n.

Problema 18. O numero de seis dıgitos X = abcdef satisfaz a propriedade de que abc−def

e divisıvel por 7. Prove que X tambem e divisıvel por 7.

Problema 19. (Bielorussia 1996) Inteiros m e n, satisfazem a igualdade

(m− n)2 =4mn

m+ n− 1.

a) Prove que m+ n e um quadrado perfeito.

b) Encontre todos os pares (m,n) satisfazendo a equacao acima.

Problema 20. (Olimpıada de Leningrado) Os numeros naturais a,b e c tem a propriedadeque a3 e divisıvel por b, b3 e divisıvel por c e c3 e divisıvel por a. Prove que (a+ b+ c)13 edivisıvel por abc.

Problema 21. (OBM 2000) E possıvel encontrar duas potencias de 2, distintas e com omesmo numero de algarismos, tais que uma possa ser obtida atraves de uma reordenacaodos dıgitos da outra? (Dica: Lembre-se do criterio de divisibilidade por 9)

Problema 22. (IMO 1998) Determine todos os pares de inteiros positivos (x, y) tais quexy2 + y + 7 divide x2y + x+ y.

Dicas e Solucoes

11. Como 3 | 6b, segue que 3 | [(a+ 7b)− 6b] = a+ b.

12. Como 7 | a+ 3b, segue que 7 | 13a+ 39b = (13a+ 11b) + 28b. Mas 7 | 28b, portanto7 | [(13a+ 11b) + 28b− 28b] = 13a+ 11b.

13. Como 19 | 3x + 7y, segue que 19 | 27(3x + 7y) = (43x + 75y) + (38x + 114y). Mas19 | 19(2x+6y), portanto 19 | [(43x+75y)+(38x+114y)−19(2x+6y)] = 43x+75y.

14. Como 17 | 3a+ 2b, segue que 17 | 27a+ 18b = (10a+ b) + 17(a+ b).

16. Veja que

nk − 1

(n− 1)2=

(

nk−1 − 1

n− 1+

nk−2 − 1

n− 1+ . . .+

n− 1

n− 1+

k

n− 1

)

.

Como os numeros nl−1

n−1 sempre sao inteiros, o numero do lado esquerdo da equacao

sera inteiro se, e somente se, o numero k

n−1 for inteiro.

4

Page 12: OBM - Teoría de Números.pdf

POT 2012 - Teoria dos Numeros - Nıvel 2 - Aula 1 - Samuel Feitosa

17. Comece dividindo o problema quando em dois casos: n e par ou n e ımpar. Sabemosque 1+ 2+ . . .+ n = n(n+1)

2 . Para n ımpar, basta mostrar que o numero em questaoe divisıvel por n e n+1

2 . O proximo passo e lembrar do problema 33 da aula 1. Pelafatoracao de xn + yn, temos que ik + (n− i)k e divisıvel por n. Faca outros tipos depares para mostrar a divisibilidade por n

2 . O caso quando n e par e analogo.

18. Veja que X = 103 · abc + def = 1001abc − (abc − def). Como 1001 e multiplo de 7,concluımos que X e a soma de dois multiplos de 7.

19. Somando 4mn em ambos os lados, obtemos:

(m+ n)2 =4mn

m+ n− 1+ 4mn

=4mn(m+ n)

m+ n− 1⇒

(m+ n) =4mn

m+ n− 1

= (m− n)2.

Assim, m + n e o quadrado de um inteiro. Se m − n = t, entao m + n = t2 e(m,n) = ( t

2+t

2 , t2−t

2 ). E facil verificar que para qualquer t inteiro esse par e solucaodo problema.

20. Analise a expansao pelo binomio de Newton.

21. Nao. Suponha, por absurdo, que existam duas potencias de 2, 2m < 2n, satisfazendoo enunciado. Como 2n e um multiplo de 2m, podemos ter: 2n = 2·2m, 4·2m, 8·2m, . . ..Alem disso, como ambos possuem a mesma quantidade de dıgitos, temos 1 < 2n

2m <

10. Assim, as unicas possibilidade sao 2n = 2 · 2m, 4 · 2m, 8 · 2m. Pelo criterio dedivisibilidade por 9, como 2m e 2n possuem os mesmos dıgitos, podemos concluirque 2n − 2m e um multiplo de 9. Entretanto, nenhuma das possibilidade anterioressatisfaz essa condicao e chegamos em um absurdo.

22. Comecaremos usando a ideia do exemplo 10. A igualdade y(x2y + x+ y)− x(xy2 +y + 7) = y2 − 7x implica que y2 − 7x e divisıvel por xy2 + y + 7. Se y2 − 7x ≥ 0,como y2 − 7x < xy2 + y + 7, segue que y2 − 7x = 0. Assim, (x, y) = (7t2, 7t) paraalgum t ∈ N. E facil checar que esses pares sao realmente solucoes. Se y2 − 7x < 0,entao 7x− y2 > 0 e divisıvel por xy2 + y + 7. Daı, xy2 + y + 7 ≤ 7x− y2 < 7x, quenos permite concluir que y ≤ 2. Para y = 1, temos x+8 | 7x− 1 e consequentementex+ 8 | 7(x+ 8)− (7x− 1) = 57. Entao as unicas possibilidades sao x = 11 e x = 49,cujos pares correspondentes sao (11, 1), (49, 1). Para y = 2, temos 4x + 9 | 7x − 4e consequentemente 7(4x + 9) − 4(7x − 4) = 79 e divisıvel por 4x + 9. Nesse caso,nao obtemos nenhuma solucao nova. Todas as solucoes para (x, y) sao: (7t2, 7t)(t ∈N), (11, 1) e (49, 1).

5

Page 13: OBM - Teoría de Números.pdf

Referencias

[1] F. E. Brochero Martinez, C. G. Moreira, N. C. Saldanha, E. Tengan - Teoria dosNumeros um passeio com primos e outros numeros familiares pelo mundo inteiro,Projeto Euclides, IMPA, 2010.

[2] E. Carneiro, O. Campos and F. Paiva, Olimpıadas Cearenses de Matematica 1981-2005(Nıveis Junior e Senior), Ed. Realce, 2005.

[3] S. B. Feitosa, B. Holanda, Y. Lima and C. T. Magalhaes, Treinamento Cone Sul 2008.Fortaleza, Ed. Realce, 2010.

[4] D. Fomin, A. Kirichenko, Leningrad Mathematical Olympiads 1987-1991, MathProPress, Westford, MA, 1994.

[5] D. Fomin, S. Genkin and I. Itenberg, Mathematical Circles, Mathematical Words, Vol.7, American Mathematical Society, Boston, MA, 1966.

[6] I. Niven, H. S. Zuckerman, and H. L. Montgomery, An Introduction to the Theory ofNumbers.

Page 14: OBM - Teoría de Números.pdf

Polos Olímpicos de TreinamentoCurso de Teoria dos Números - Nível 2Prof. Samuel Feitosa

Aula 3

O Algoritmo de Euclides

Exemplo 1. Seja S um conjunto infinito de inteiros nao negativos com a seguinte propri-edade: dados dois quaisquer de seus elementos, o valor absoluto da diferenca entre elestambem pertence a S. Se d e o menor elemento positivo de S, prove que S consiste detodos os multiplos de d.

Considere um elemento m qualquer de S. Pelo algoritmo da divisao, m = qd + r com0 ≤ r < d. Como todos os numeros m − d,m − 2d,m − 3d, . . . ,m − qd = r pertencema S e d e o menor elemento positivo de tal conjunto, devemos ter obrigatoriamente quer = 0. Sendo assim, podemos concluir que todos os elementos de S sao multiplos de d.Resta mostrarmos que todos os multiplos de d estao em S. Seja kd um multiplo positivoqualquer de d. Como S e infinito, existe um inteiro m ∈ S tal que m = qd > kd. Assimtodos os numeros m− d,m− 2d, . . . ,m− (q − k)d = kd estao em S.

Definicao 2. Um inteiro a e um divisor comum de b e c se a | b e a | c. Se b e c nao saoambos nulos, denotaremos por mdc(b, c) o maximo divisor comum de b e c.

Como um inteiro nao nulo possui apenas um numero finito de divisores, se b e c sao ambosnao nulos, o numero mdc(b, c) sempre existe, isto e, sempre esta bem definido.

Lema 3. (Euclides) Se x 6= 0, mdc(x, y) = mdc(x, x+ y)

Demonstracao. Seja d um divisor comum de x e y. Entao d | x+ y e consequentemente dtambem a um divisor comum de x e x+ y. Reciprocamente, se f e um divisor comum dex+ y e x, f tambem divide (x+ y)− y = x e assim f e um divisor comum de x e y. Comoos conjuntos de divisores comuns dos dois pares de numeros mencionados sao os mesmos,o maior divisor comum tambem e o mesmo.

Entao podemos calcular:

mdc(123, 164) = mdc(123, 41) = mdc(41, 123) = mdc(41, 82) = mdc(41, 41) = 41.

Page 15: OBM - Teoría de Números.pdf

POT 2012 - Teoria dos Numeros - Nıvel 2 - Aula 3 - Samuel Feitosa

Exemplo 4. Tres maquinas I, R, S imprimem pares de inteiros positivos em tickets. Paraa entrada (x, y), as maquinas I, R, S imprimem respectivamente (x−y, y), (x+y, y), (y, x).Iniciando com o par (1, 2) podemos alcancar

a) (819, 357)?

b) (19, 79)?

Para o item a), calculemos inicialmente mdc(819, 357):

mdc(819, 357) = mdc(462, 357) = mdc(105, 357) = mdc(105, 252) = . . . = mdc(21, 21) = 21.

Pelo Lema de Euclides, o mdc entre os dois numeros em um ticket nunca muda. Comomdc(1, 2) = 1 6= 21 = mdc(819, 357), nao podemos alcancar o par do item a).

Para o item b), indiquemos com → uma operacao de alguma das maquinas. Veja que:

(2, 1)R→ (3, 1)

S→ (1, 3)

R→ (4, 3)

R→ . . .

R→ (19, 3)

S→ (3, 19)

R→ (22, 19)

R→ (41, 19)

R→

(60, 19)R→ (79, 19).

Observacao 5. Procurar invariantes sempre e uma boa estrategia para comparar confi-guracoes diferentes envolvidas no problema. Confira o problema proposto 31.

Definicao 6. Dizemos que dois inteiros p e q sao primos entre si ou relativamente primosse mdc(p, q) = 1. Dizemos ainda que a fracao p

qe irredutıvel se p e q sao relativamente

primos.

Exemplo 7. (IMO 1959) Prove que21n+ 4

14n+ 3e irredutıvel para todo numero natural n.

Pelo lema de Euclides, mdc(21n+4, 14n+3) = mdc(7n+4, 14n+3) = mdc(7n+1, 7n+2) =mdc(7n+ 1, 1) = 1.

O seguinte lema sera provado na proxima aula.

Lema 8. (Propriedades do MDC) Seja mdc(a, b) = d, entao:

i) Se k 6= 0, mdc(ka, kb) = kd.

ii) mdc

(

a

d,b

d

)

= 1.

iii) Se mdc(a, c) = 1, entao mdc(a, bc) = d.

Exemplo 9. (Olimpıada Inglesa) Se x e y sao inteiros tais que 2xy divide x2+y2−x, proveque x e um quadrado perfeito

2

Page 16: OBM - Teoría de Números.pdf

POT 2012 - Teoria dos Numeros - Nıvel 2 - Aula 3 - Samuel Feitosa

Se d = mdc(x, y), entao x = da e y = db, com mdc(a, b) = 1. Do enunciado, temos:

2abd2 | d2a2 + d2b2 − da ⇒

d2 | d2a2 + d2b2 − da ⇒

d2 | −da ⇒

d | a.

Logo, a = dc, para algum c. Como x | y2, obtemos d2c | d2b2, ou seja, c|b2 e mdc(c, b2) = c.Usando que mdc(a, b) = 1 e que todo divisor comum de b e c tambem e um divisor comumde a e b, podemos concluir que mdc(c, b) = 1. Usando o item iii) do lema anterior,mdc(c, b2) = 1. Assim, c = 1 e x = d2c = d2.

Exemplo 10. No planeta X, existem apenas dois tipos de notas de dinheiro: $5 e $78. Epossıvel pagarmos exatamente $7 por alguma mercadoria? E se as notas fossem de $ 3 e $

78?

Veja que 2 × 78 − 31 × 5 = 1 e consequentemente 14 × 78 − 217 × 5 = 7. Basta darmos14 notas de de $ 78 para recebermos 217 notas de $ 5 como troco na compra de nossamercadoria. Usando as notas de $3 e $78 nao e possıvel pois o dinheiro pago e recebidocomo troco por algo sempre e multiplo de 3 e 7 nao e multiplo de 3.

Queremos estudar a versao mais geral desse exemplo. Quais sao os valores que podemospagar usando notas de $a e $b? Em particular, estaremos interessados em conhecer qual omenor valor que pode ser pago. Para responder essa pergunta, precisaremos do algoritmode Euclides:

Teorema 11. (O Algoritmo de Euclides) Para os inteiros b e c > 0, aplique sucessivamenteo algoritmo da divisao para obter a serie de equacoes:

b = cq1 + r1, 0 < r1 < c,

c = r1q2 + r2, 0 < r2 < r1,

r1 = r2q3 + r3, 0 < r3 < r2,...

rj−2 = rj−1qj + rj , 0 < rj < rj−1,

rj−1 = rjqj+1

A sequencia de restos nao pode diminuir indefinidamente pois 0 ≤ ri < ri−1 e existe apenasum numero finito de naturais menores que c. Assim, para algum j, obteremos rj+1 = 0.O maior divisor comum de b e c sera rj, ou seja, o ultimo resto nao nulo da sequencia dedivisoes acima.

Demonstracao. Pelo Lema de Euclides,

mdc(x+ qy, y) = mdc(x+ (q − 1)y, y) = mdc(x+ (q − 2)y, y) = . . . = mdc(x, y).

3

Page 17: OBM - Teoría de Números.pdf

POT 2012 - Teoria dos Numeros - Nıvel 2 - Aula 3 - Samuel Feitosa

Entao,mdc(b, c) = mdc(c, r1) = mdc(r1, r2) = . . . = mdc(rj−1, rj) = rj .

Exemplo 12. Calcule mdc(42823, 6409).

Pelo Algoritmo de Euclides,

42823 = 6× 6409 + 4369

6409 = 1× 4369 + 2040

4369 = 2× 2040 + 289

2040 = 7× 289 + 17

289 = 17× 17.

Portanto, mdc(42823, 6409) = 17.

Podemos extrair mais informacoes do Algoritmo de Euclides. Para isso, iremos organizaras equacoes do exemplo acima de outra forma.

Essencialmente, a equacao mdc(x+qy, y) = mdc(x, y) nos diz que podemos subtrair q vezesum numero de outro sem alterar o maximo divisor comum do par em questao. Realizandoesse procedimento sucessivas vezes, subtraindo o numero menor do maior, podemos obterpares com numeros cada vez menores ate que chegarmos em um par do tipo (d, d). Como omaximo divisor comum foi preservado ao longo dessas operacoes, d sera o maximo divisorcomum procurado. Iremos repetir o exemplo anterior registrando em cada operacao quantasvezes um numero e subtraido do outro. Isso sera feito atraves de dois pares de numerosauxiliares:

(42823, 6409) | (1, 0)(0, 1)

(4369, 6409) | (1,−6)(0, 1)

(4369, 2040) | (1,−6)(−1, 7)

(289, 2040) | (3,−20)(−1, 7)

(289, 17) | (3,−20)(−22, 147)

(17, 17) | (355,−2372)(−22, 147)

Da primeira linha para a segunda, como subtraımos 6 vezes o numero 6409 de 42823,subtraımos 6 vezes o par (0, 1) de (1, 0), obtendo: (1, 0) − 6(0, 1) = (1,−6). Se em umadada linha, temos:

(x, x+ qy)) | (a, b)(c, d);

entao, a proxima linha devera ser:

(x, y) | (a, b)(c− aq, d− bq);

4

Page 18: OBM - Teoría de Números.pdf

POT 2012 - Teoria dos Numeros - Nıvel 2 - Aula 3 - Samuel Feitosa

porque representara a operacao de subtrairmos q vezes o primeiro numero do segundo. Vejaque o par (a, b) foi subtraido de (c, d) exatamente q vezes. Os numeros escritos nos ultimosdois pares representam os coeficientes dos numeros originais para cada numero do primeiropar. Por exemplo, analisando a linha:

(289, 2040) | (3,−20)(−1, 7);

obtemos que:

289 = 3× 42823− 20× 6409,

2040 = −1× 42823 + 7× 6409.

Em cada linha, essa propriedade e mantida pois a mesma subtracao que e realizada noprimeiro par tambem e realizada entre os dois ultimos pares. Analisando o ultimo par,podemos escrever 17 como combinacao de 42823 e 6409 de duas formas diferentes:

17 = −22× 42823 + 147× 6409,

17 = 355× 42823 +−2372× 6409,

Assim, se no planeta X tivessemos apenas notas de $42823 e $6409, poderıamos compraralgo que custasse exatamente $17.

Como conclusao da discussao anterior e do algoritmo de Euclides, podemos concluir que:

Teorema 13. (Bachet-Bezout) Se d = mdc(a, b), entao existem inteiros x e y tais queax+ by = d.

De fato, a discussao anterior tambem nos mostra um algoritmo para encontrarmos x e y.Voltando a discussao sobre o planeta X, podemos concluir em virtude do teorema anteriorque qualquer valor multiplo de d podera ser pago usando apenas as notas de $a e $b.Como todo valor pago, necessariamente e um multiplo do maximo divisor comum de a eb, descobrimos que o conjunto que procuravamos consiste precisamente do conjunto dosmultiplos de d.

Observacao 14. (Para professores) A prova mais comum apresentada para o teorema an-terior baseia-se na analise do conjunto de todas as combinacoes lineares entre a e b e quasesempre se preocupa apenas com mostrar a existencia de x e y. Acreditamos que o algoritmopara encontrar x e y facilite o entendimento do teorema para os alunos mais jovens. Entre-tanto, frequentemente utilizemos apenas a parte da existencia descrita no enunciado. Alemdisso, preferimos discutir um exemplo numerico ao inves de formalizarmos uma prova esugerimos que o professor faca o mesmo com mais exemplos em aula.

Exemplo 15. (Olimıada Russa 1995) A sequencia a1, a2, ... de naturais satisfaz mdc(ai, aj) =mdc(i, j) para todo i 6= j Prove que ai = i para todo i.

5

Page 19: OBM - Teoría de Números.pdf

POT 2012 - Teoria dos Numeros - Nıvel 2 - Aula 3 - Samuel Feitosa

Para qualquer inteiro n, mdc(a2n, an) = mdc(2n, n) = n, consequentemente n | an. Sejad um divisor qualquer de an diferente de n, entao d | mdc(ad, an). De mdc(ad, an) =mdc(d, n), podemos concluir que d | n. Sendo assim, todos os divisores de an que saodiferentes de n sao divisores de n. Como ja sabemos que an = nk, para algum k, naopodemos ter k > 1 pois nk nao divide n e assim concluımos que an = n.

Exemplo 16. Mostre que mdc(2120 − 1, 2100 − 1) = 220 − 1.

Pelo lema de Euclides,

mdc(2120 − 1, 2100 − 1) = mdc(2120 − 1− 220(2100 − 1), 2100 − 1),

= mdc(220 − 1, 2100 − 1),

= mdc(220 − 1, 2100 − 1− 280(220 − 1)),

= mdc(220 − 1, 280 − 1),

= mdc(220 − 1, 280 − 1− 260(220 − 1)),

= mdc(220 − 1, 260 − 1),

= mdc(220 − 1, 260 − 1− 240(220 − 1)),

= mdc(220 − 1, 240 − 1),

= mdc(220 − 1, 240 − 1− 220(220 − 1)),

= mdc(220 − 1, 220 − 1) = 220 − 1.

Exemplo 17. (Olimpıada Russa 1964) Sejam x, y inteiros para os quais a fracao

a =x2 + y2

xy

e inteira. Ache todos os possıveis valores de a.

A primeira estrategia e cancelar os fatores comuns com o objetivo de reduzir o problemaao caso em que x e y sao primos entre si. Seja d = mdc(x, y), com

{

x = d · x0y = d · y0

, mdc(x0, y0) = 1,

entao

a =x2 + y2

xy=

x02 + y0

2

x0y0·

Nessa condicao, como x0 divide y20 e y0 divide x20, cada um deles e igual a 1, donde

a =12 + 12

1 · 1= 2.

6

Page 20: OBM - Teoría de Números.pdf

POT 2012 - Teoria dos Numeros - Nıvel 2 - Aula 3 - Samuel Feitosa

Definicao 18. Os inteiros a1, a2, . . . , an, todos diferentes de zero, possuem multiplo comumb se ai|b para i = 1, 2, . . . , n(note que a1a2 . . . an e um multiplo comum). O menor multiplocomum positivo para tal conjunto de inteiros e chamado de mınimo multiplo comum e seradenotado por mmc(a1, a2, . . . , an).

Proposicao 19. Se a e b sao nao nulos, entao: mmc(a, b) ·mdc(a, b) = |ab|.

(A prova desta proposicao tambem sera deixada para a proxima secao)

Exemplo 20. (Olimpıada Russa 1995) Sejam m e n interios positivos tais que:

mmc(m,n) +mdc(m,n) = m+ n.

Prove que um deles e divisıvel pelo o outro.

Se d = mdc(m,n), entao podemos escrever m = da e n = db. Pela proposicao anterior,

mmc(m,n) =d2ab

d= dab.

Temos:

mmc(m,n) +mdc(m,n)−m− n = 0 ⇒

dab+ d− da− db = 0 ⇒

ab+ 1− a− b = 0 ⇒

(a− 1)(b− 1) = 0.

Portanto, ou a = 1 e m | n ou entao b = 1 e n | m.

Exemplo 21. (Torneio das Cidades 1998) Prove que, para quaisquer inteiros positivos a eb, a equacao mmc(a, a+ 5) = mmc(b, b+ 5) implica que a = b.

Para o item a), como (a+ 5)− a = 5, temos mdc(a, a+ 5) e igual a 1 ou 5. O mesmo valepara mdc(b, b+ 5). Pela proposicao anterior, temos:

mmc(a, a+ 5) =a(a+ 5)

mdc(a, a+ 5),

mmc(b, b+ 5) =b(b+ 5)

mdc(b, b+ 5).

Suponha que mdc(a, a+5) = 5 e mdc(b, b+5) = 1, entao a(a+5) = 5b(b+5). Consequen-temente, a e multiplo de 5 e a(a+ 5) e multiplo de 25. Isso implica que b(b+ 5) tambem emultiplo de 5 e que mdc(b, b+ 5) > 1. Uma contradicao. Analogamente, nao podemos termdc(a, a+ 5) = 1 e mdc(b, b+ 5) = 5. Sendo assim, mdc(a, a+ 5) = mdc(b, b+ 5) e:

a(a+ 5)− b(b+ 5) = 0 ⇒

(a− b)(a+ b+ 5) = 0.

Como a+ b+ 5 > 0, concluımos que a = b.

7

Page 21: OBM - Teoría de Números.pdf

POT 2012 - Teoria dos Numeros - Nıvel 2 - Aula 3 - Samuel Feitosa

Exemplo 22. Uma maquina f executa operacoes sobre o conjunto de todos os pares deinteiros positivos. Para cada par de inteiros positivos, ela fornece um inteiro dado pelasregras:

f(x, x) = x, f(x, y) = f(y, x), (x+ y)f(x, y) = yf(x, x+ y).

Determine f(2012, 2012! + 1).

Claramente mmc(x, x) = x e mmc(x, y) = mmc(y, x). Usando a proposicao anterior e olema de Euclides temos:

(x+ y)mmc(x, y) = (x+ y)xy

mdc(x, y)= y ·

x(x+ y)

mdc(x, x+ y)= y ·mmc(x, x+ y)

Temos entao uma forte suspeita de que f = mmc. Seja S o conjunto de todos os pa-res de inteiros positivos (x, y) tais que f(x, y) 6= mmc(x, y), e seja (m,n) o par em Scom a soma m + n minima. Note que todo par da forma (n, n) nao esta em S poisf(n, n) = n = mmc(n, n). Assim, devemos ter m 6= n. Suponha sem perda de generalidadeque n > m. Portanto:

nf(m,n−m) = [m+ (n−m)]f(m,n−m) ⇒

= (n−m)f(m,m+ (n−m)) ⇒

f(m,n−m) =n−m

n· f(m,n)

Como o par (m,m − n) nao esta em S, dado que a soma de seus elementos e menor quem+ n, temos:

f(m,n−m) = mmc(m,n−m) ⇒n−m

n· f(m,n) = (n−m)mmc(m,m+ (n−m)) ⇒

f(m,n) = mmc(m,n)

Uma contradicao. Desse modo, S deve ser um conjunto vazio e f(x, y) = mmc(x, y)para todos os pares de inteiros positivos. Como 2012 | 2012!, mdc(2012, 2012! + 1) = 1 econsequentemente mmc(2012, 2012! + 1) = 2012(2012! + 1).

Problemas Propostos

Problema 23. Calcule:

a) mdc(n, n2 + n+ 1).

b) mdc(3× 2012, 2× 2012 + 1).

8

Page 22: OBM - Teoría de Números.pdf

POT 2012 - Teoria dos Numeros - Nıvel 2 - Aula 3 - Samuel Feitosa

c) mdc

(

240 + 1

28 + 1, 28 + 1

)

.

Problema 24. Encontre mdc(2n+ 13, n+ 7)

Problema 25. Prove que a fracao 12n+130n+2 e irredutıvel.

Problema 26. Sejam a, b, c, d inteiros nao nulos tais que ad− bc = 1. Prove que a+bc+d

e umafracao irredutıvel.

Problema 27. Mostre que mdc(am − 1, an − 1) = amdc(m,n) − 1.

Problema 28. Mostre que se mdc(a, b) = 1, entao:

mdc(a+ b, a2 − ab+ b2) = 1 ou 3

Problema 29. Dado que mdc(a, 4) = 2, mdc(b, 4) = 2, prove que:

mdc(a+ b, 4) = 4.

Problema 30. Prove que, para todo natural n,

mdc(n! + 1, (n+ 1)! + 1) = 1.

Problema 31. No exemplo 4, determine todos os pares que podem ser obtidos comecando-secom o par (1, 2).

Problema 32. Qual o maximo divisor comum do conjunto de numeros:

{16n + 10n− 1, n = 1, 2, 3 . . .}?

Problema 33. A sequencia Fn de Farey e a sequencia de todos as fracoes irredutıveisa

bcom 0 ≤ a ≤ b ≤ n arranjados em ordem crescente.

F1 = {0/1, 1/1}F2 = {0/1, 1/2, 1/1}F3 = {0/1, 1/3, 1/2, 2/3, 1/1}F4 = {0/1, 1/4, 1/3, 1/2, 2/3, 3/4, 1/1}F5 = {0/1, 1/5, 1/4, 1/3, 2/5, 1/2, 3/5, 2/3, 3/4, 4/5, 1/1}F6 = {0/1, 1/6, 1/5, 1/4, 1/3, 2/5, 1/2, 3/5, 2/3, 3/4, 4/5, 5/6, 1/1}

Claramente, toda fracao ab< 1 com mdc(a, b) = 1, esta em algum Fn. Mostre que se m/n

e m′/n′ sao fracoes consecutivas em Fn temos |mn′ − nm′| = 1.

Problema 34. (Resvista Quantum - Jornal Kvant) Todas as fracoes irredutıveis cujos de-nominadores nao excedem 99 sao escritas em ordem crescente da esquerda para a direita:

1

99,1

98, . . . ,

a

b,5

8,c

d, . . .

Quais sao as fracoesa

bec

dem cada lado de

5

8?

9

Page 23: OBM - Teoría de Números.pdf

POT 2012 - Teoria dos Numeros - Nıvel 2 - Aula 3 - Samuel Feitosa

Problema 35. (OBM) Para cada inteiro positivo n > 1, prove que 1 + 12 +

13 + . . .+ 1

nnao

e inteiro.

Problema 36. Determine todas as solucoes em inteiros positivos para 1a+ 1

b= 1

c.

Problema 37. Inteiros positivos a e b, relativamente primos, sao escolhidos de modo quea+ b

a− bseja tambem um inteiro positivo. Prove que pelo menos um dos numeros ab + 1 e

4ab+ 1 e um quadrado perfeito.

Problema 38. (IMO 1979) Sejam p, q numeros naturais primos entre si tais que:

p

q= 1−

1

2+

1

3− . . .−

1

1318+

1

1319.

Prove que p e divisıvel por 1979.

Respostas, Dicas e Solucoes

23. (a)

mdc(n, n2 + n+ 1) = mdc(n, n2 + n+ 1− n(n+ 1)),

= mdc(n, 1),

= 1.

(b)

mdc(3× 2012, 2× 2012 + 1) = mdc(3× 2012− (2× 2012 + 1), 2× 2012 + 1),

= mdc(2012− 1, 2× 2012 + 1),

= mdc(2012− 1, 2× 2012 + 1− 2(2012− 1)),

= mdc(2012− 1, 3),

= mdc(2012− 1− 3× 670, 3),

= mdc(2, 3) = 1.

Outra opcao seria observar que o mdc procurado deve dividir o numero 3(2 ×2012 + 1)− 2(3× 2012) = 3 e que 2× 2012 + 1 nao e multiplo de 3.

(c)

mdc

(

240 + 1

28 + 1, 28 + 1

)

= mdc(

232 + 224 + 216 + 28 + 1, 28 + 1)

,

= mdc(

(232 − 1) + (224 + 1) + (216 − 1) + (28 + 1) + 1, 28 + 1)

,

= mdc(1, 28 + 1) = 1.

10

Page 24: OBM - Teoría de Números.pdf

POT 2012 - Teoria dos Numeros - Nıvel 2 - Aula 3 - Samuel Feitosa

24.

mdc(2n+ 13, n+ 7) = mdc(2n+ 13− 2(n+ 7), n+ 7),

= mdc(2n+ 13− 2(n+ 7), n+ 7),

= mdc(−1, n+ 7) = 1

25.

mdc(12n+ 1, 30n+ 2) = mdc(12n+ 1, 30n+ 2− 2(12n+ 1)),

= mdc(12n+ 1, 6n),

= mdc(12n+ 1− 2(6n), 6n),

= mdc(1, 6n) = 1

26. Seja f = mdc(a + b, c + d). Entao f | d(a + b) − b(c + d) = 1 e consequentementef = 1.

27. Veja que

mdc(am − 1, an − 1) = mdc(am−n − 1 + (an − 1)am−n, an − 1)

= mdc(am−n − 1, an − 1)

O resultado segue aplicando o Algoritmo de Euclides aos expoentes.

28. Seja f = mdc(a + b, a2 − ab + b2). Entao f | (a + b)2 − (a2 − ab + b2) = 3ab. Semdc(f, a) > 0, devemos ter mdc(f, b) > 0 pois f | a + b. O mesmo argumento valepara mdc(f, b) > 0. Assim, mdc(f, a) = mdc(f, b) = 1. Portanto, f | 3.

30. Pelo lema de Euclides,

mdc(n! + 1, (n+ 1)! + 1) = mdc(n! + 1, (n+ 1)! + 1− (n+ 1)(n! + 1))

= mdc(n! + 1,−n)

= mdc(n! + 1− n[(n− 1)!],−n) = 1

34. Sejam l = mmc{1, 2, . . . , n} e ai = l/i. A soma considerada e

a1 + a2 + . . .+ anl

.

Queremos analisar o expoente do fator 2 no numerador e no denominador. Seja k talque 2k ≤ n < 2k+1. Entao 2k||l e ai e par para todo i 6= 2k. Como a2k e ımpar, segueque o numerador e ımpar enquanto que o denominador e par. Consequentemente afracao anterior nao representa um inteiro.

11

Page 25: OBM - Teoría de Números.pdf

36. Sejam d = mdc(a, b), a = dx, b = dy. Consequentemente mdc(x, y) = 1 e podemosescrever a equacao como:

1

a+

1

b=

1

c⇒

bc+ ac = ab

dyc+ dxc = d2xy

c(x+ y) = dxy

Como mdc(xy, x+ y) = 1 pois mdc(x, y) = 1, devemos ter xy | c e consequentementec = xyk. Assim, d = k(x + y). O conjunto solucao e formado pelas triplas (a, b, c)onde (a, b, c) = (kx(x + y), ky(x + y), xyk) com mdc(x, y) = 1 e x, y e k inteirospositivos.

38. Use a identidade de Catalao:

1−1

2+

1

3−

1

4+ . . .−

1

2n=

1

n+ 1+

1

n+ 2+ . . .+

1

2n

Em seguida, agrupe os termos da forma1

n+ i+

1

2n− i+ 1e analise o numerador da

fracao obtida.

Referencias

[1] S. B. Feitosa, B. Holanda, Y. Lima and C. T. Magalhaes, Treinamento Cone Sul 2008.Fortaleza, Ed. Realce, 2010.

[2] D. Fomin, A. Kirichenko, Leningrad Mathematical Olympiads 1987-1991, MathProPress, Westford, MA, 1994.

[3] D. Fomin, S. Genkin and I. Itenberg, Mathematical Circles, Mathematical Words, Vol.7, American Mathematical Society, Boston, MA, 1966.

[4] I. Niven, H. S. Zuckerman, and H. L. Montgomery, An Introduction to the Theory ofNumbers.

Page 26: OBM - Teoría de Números.pdf

Polos Olímpicos de TreinamentoCurso de Teoria dos Números - Nível 2Prof. Samuel Feitosa

Aula 4

Numeros Primos, MDC e MMC.

Definicao 1. Um inteiro p > 1 e chamado numero primo se nao possui um divisor d

satisfazendo 1 < d < p. Se um inteiro a > 1 nao e primo, ele e chamado de numerocomposto. Um inteiro m e chamado de composto se |m| nao e primo.

O proximo teorema nos diz que os primos sao as ”pecas”fundamentais dos numeros inteiros:

Teorema 2. Todo inteiro n, maior que 1, pode ser expresso como o produto de numeroprimo.

Demonstracao. Se o inteiro n e um primo, entao ele mesmo e o produto de um unico fa-tor primo. Se o inteiro n nao e primo, existe uma decomposicao do tipo: n = n1n2 com1 < n1 < n e 1 < n2 < n. Repetindo o argumento para n1 e n2, podemos escrever n comoo produto de primos ou podemos obter parcelas menores escrevendo n como um produtode naturais. Como nao existe uma sucessao infinita de naturais cada vez menores, apos umnumero finito de operacoes desse tipo, poderemos escrever n como um produto de numerosprimos.

Quantos numeros primos existem?

Teorema 3. (Euclides) Existem infinitos numeros primos.

Demonstracao. Suponha, por absurdo, que exita apenas uma quantidade finita de primos:p1, p2, . . . , pn. Considere o numero X = p1p1 . . . pn+1. Pelo teorema anterior, esse numerodeve ser o produto de alguns elementos do conjunto de todos os numeros primos. Entre-tanto, nenhum dos primos pi divide X.

Exemplo 4. Existe um bloco de 1000 inteiros consecutivos nao contendo nenhum primo?

Sim. Um exemplo e o conjunto 1001!+2, 1001!+3, . . . , 1001!+1001. Veja i | 1001!+ i paratodo i = 2, 3, . . . , 1001.

Page 27: OBM - Teoría de Números.pdf

POT 2012 - Teoria dos Numeros - Nıvel 2 - Aula 4 - Samuel Feitosa

Exemplo 5. (Torneio das Cidades) Existe um bloco de 1000 inteiros consecutivos contendoapenas um primo?

Para cada bloco de 1000 numeros consecutivos, contemos sua quantidade de numeros pri-mos. Por exemplo, no bloco 1, 2, 3, . . . , 1000, temos 168 numeros primos (mas so usaremoso fato de que existem mais de dois primos nesse bloco). Comparando os blocos consecuti-vos k + 1, k + 2, . . . , k + 1000 e k + 2, k + 3, . . . , k + 1001, ou o numero de numeros primosaumenta em uma unidade, ou fica constante ou diminui em uma unidade. Analisando to-dos os blocos consecutivos desde 1, 2, . . . , 1000 ate 1001! + 2, 1001! + 3, . . . , 1001! + 1001,o numero de numeros primos deve ser igual a 1 em algum deles. Para ver isso, usare-mos um argumento de continuidade discreta: Comecando com o numero 168 e realizandoalteracoes de no maximo uma unidade na quantidade de primos em cada bloco, para che-garmos no numero 0, necessariamente deveremos passar pelo numero 1 em algum momento.

Relembremos um importante resultado da aula passada:

Teorema 6. (Bachet- Bezout) Se d = mdc(a, b), entao existem inteiros x e y tais queax+ by = d.

Proposicao 7. Sejam a, b e c inteiros positivos com a | bc e mdc(a, b) = 1. Entao, a | c.

Demonstracao. Pelo teorema anterior, existem x e y inteiros tais que ax+ by = 1. Assim,acx+ bcy = c. Como a | acx e a | bcy, podemos concluir que a | c.

Em particular, se p e um numero primo e p | ab, entao p | a ou p | b. Podemos usar essefato para garantir a unicidade em nosso primeiro teorema, obtendo o importante:

Teorema 8. (Teorema Fundamental da Aritmetica) A fatoracao de qualquer inteiro n > 1,em fatores primos, e unica a menos da ordem dos fatores.

Exemplo 9. (Russia 1995) E possıvel colocarmos 1995 numeros naturais ao redor de umcırculo de modo que para quaisquer dois numeros vizinhos a razao entre o maior e o menorseja um numero primo?

Nao, e impossıvel. Suponha, por absurdo, que isso seja possıvel e denotemos pora0, a1, . . . , a1995 = a0 tais inteiros. Entao, para k = 1, . . . , 1995,

ak−1

ake primo ou o in-

verso de um primo. Suponha que a primeira situacao ocorra m vezes e a segunda ocorra1995 − m vezes entre esses quocientes. Como o produto de todos os numeros da formaak−1

ak, para k = 1, . . . , 1995 e igual a 1, podemos concluir que o produto de m primos deve

ser igual ao produto de 1995−m primos. Em virtude da fatoracao unica, m = 1995−m.Um absurdo pois 1995 e ımpar.

Proposicao 10. Se as fatoracoes em primos de n e m sao:

n = pα1

1 pα2

2 . . . pαk

k ,

m = pβ1

1 pβ2

2 . . . pβk

k .

2

Page 28: OBM - Teoría de Números.pdf

POT 2012 - Teoria dos Numeros - Nıvel 2 - Aula 4 - Samuel Feitosa

Entao, mdc(m,n) = pγ11 p

γ22 . . . p

γkk e mmc(m,n) = pθ11 pθ22 . . . p

θkk , onde γi e o menor dentre

{αi, βi} e θi e o maior dentre {αi, βi}.Proposicao 11. Se a e b sao inteiros positivos, mostre que mmc(a, b)mdc(a, b) = ab.

Demonstracao. Basta usar a proposicao anterior e observar que:

max{x, y}+min{x, y} = x+ y.

Exemplo 12. (Torneio das Cidades 1998) E possıvel que mmc(a, b) = mmc(a + c, b + c)para alguma conjunto {a, b, c} de inteiros positivos?

Nao. Suponha que a+c e b+c possuem algum divisor primo p. Como p | mmc(a+c, b+c),caso existam tais inteiros, devemos ter que p | mmc(a, b). Assim, usando que pelo menosum dentre a e b e divisıvel por p podemos concluir que c tambem e divisıvel por p. Entao,podemos cancelar o fator p:

mmc

(

a

p,b

p

)

=mmc(a, b)

p=

mmc(a+ c, b+ c)

p= mmc

(

a+ c

p,b+ c

p

)

.

Efetuando alguns cancelamentos, podemos supor entao que a+c e b+c nao possuem fatoresprimos em comum. Obtivemos um absurdo pois:

mmc(a+ c, b+ c) = (a+ c)(b+ c) > ab ≥ mmc(a, b).

Exemplo 13. (OCM 2005) Determinar os inteiros n > 2 que sao divisıveis por todos osprimos menores que n.

Como mdc(n, n − 1) = 1, se n − 1 possui algum fator primo, ele nao dividira n. Assim,n− 1 < 2. Consequentemente nao existe tal inteiro.

Exemplo 14. Mostre que n4 + n2 + 1 e composto para n >1.

Veja que n4 + n2 + 1 = n4 + 2n2 + 1 − n2 = (n2 + 1)2 − n2 = (n2 + n + 1)(n2 − n + 1).Para n > 1, n2 − n+ 1 = n(n− 1) + 1 > 1 e assim n4 + n2 + 1 e o produto de dois inteirosmaiores que 1.

Exemplo 15. Mostre que n4 + 4n e composto para todo n > 1.

Se n e par, certamente o numero em questao e divisıvel por 4. Para o caso em que n eimpar, iremos usar a fatoracao:

a4 +4b4 = a4 +4a2b2 +4b4 − 4a2b2 = (a2 +2b2)− 4b2b2 = (a2 − 2ab+2b2)(a2 +2ab+2b2).

Para n da forma 4k + 1, faca a = n e b = 4k. Para n da forma 4k + 3, faca a = n eb = 22k+1.

Exemplo 16. Se 2n+1 e um primo ımpar para algum inteiro positivo n, prove que n e umapotencia de 2.

3

Page 29: OBM - Teoría de Números.pdf

POT 2012 - Teoria dos Numeros - Nıvel 2 - Aula 4 - Samuel Feitosa

Ja vimos que an − 1 = (a− 1)(an−1 + an−2 + . . .+ 1). Se n e impar,

(−a)n − 1 = (−a− 1)((−a)n−1 + (−a)n−2 + . . .+ 1) ⇒an + 1 = (a+ 1)(an−1 − an−2 + . . .− a+ 1)

Sendo assim, se n possuısse algum divisor primo ımpar p com n = pb, poderıamos escrever:2n+1 = (a+1)(an−1−an−2+ . . .−a+1), onde a = 2b. Como an−1−an−2+ . . .−a+1 > 1,o numero 2n + 1 nao seria primo.

Exemplo 17. Dados que p, p+ 10 e p+ 14 sao numeros primos, encontre p.

Vamos analisar os possıveis restos na divisao por 3 de p. Se p deixa resto 1, entao p + 14e um multiplo de 3 maior que 3 e consequentemente nao podera ser um numero primo. Seo resto e 2, entao p + 10 e um multiplo de 3 maior que 3 e tambem nao podera ser umnumero primo. Assim, o resto de p por 3 e 0 e consequentemente p = 3.

Exemplo 18. (Austria-Polonia) Dados naturais n e a > 3 ımpar, mostre que a2n − 1 tem

pelo menos n+ 1 divisores primos distintos.

Usando a fatoracao da diferenca de quadrados, temos que:

a2k − 1 = (a2

k−1

+ 1)(a2k−2

+ 1) . . . (a+ 1)(a− 1).

Assim, a2m

+ 1 | a2k − 1 se k > m. Como a e impar, podemos concluir que:

mdc(a2k

+ 1, a2m

+ 1) = mdc(a2k − 1 + 2, a2

m

+ 1) = mdc(2, a2m

+ 1) = 2.

Sendo assim, na fatoracao:

a2n − 1

2n=

(a2n−1

+ 1)

2

(a2n−2

+ 1)

2. . .

(a+ 1)

2

(a− 1)

2,

temos o produto de pelo menos n inteiros primos entre si e consequentemente seus fatores

primos sao distintos. Para cada termo (a2i+1)2 , temos um fator primo pi+1 diferente de 2.

Daı, a2n − 1 possui pelo menos n+ 1 fatores primos distintos, a saber, {2, p1, p2, . . . , pn}.

Exemplo 19. (Rioplatense 1999) Sejam p1, p2, . . . , pk primos distintos. Considere todos osinteiros positivos que utilizam apenas esses primos (nao necessariamente todos) em suafatoracao em numeros primos, formando assim uma sequencia infinita

a1 < a2 < · · · < an < · · · .

Demonstre que, para cada natural c, existe um natural n tal que

an+1 − an > c.

4

Page 30: OBM - Teoría de Números.pdf

POT 2012 - Teoria dos Numeros - Nıvel 2 - Aula 4 - Samuel Feitosa

Suponha, por absurdo, que exista c > 0 tal que an+1 − an ≤ c, ∀n ∈ N. Isso significa queas diferencas entre os termos consecutivos de (an)n≥1 pertencem ao conjunto {1, 2, . . . , c},logo sao finitas. Sejam d1, d2, . . . , dr essas diferencas. Seja αi o maior expoente de pi queaparece na fatoracao de todos os dj .

Considere entao o numero M = pα1+11 pα2+1

2 · · · pαk+1k . E claro que M pertence a sequencia,

ou seja, M = an, para algum n. Vejamos quem sera an+1. Por hipotese, existe i tal quean+1− an = di. Como an+1 > an, existe um primo pj que divide an+1 com expoente maiorou igual a αj + 1. Caso contrario,

an < an+1 < pα1+11 pα2+1

2 · · · pαk+1k = an ,

absurdo. Daı, pαj+1j |an ⇒ p

αj+1j |di, novamente um absurdo, pela maximalidade de αj .

Logo, o conjunto de todas as diferencas nao pode ser finito e, portanto, dado qualquerc > 0, existe um natural n tal que an+1 − an > c.

Problemas Propostos

Problema 20. Dado que p, 2p+ 1 e 4p2 + 1 sao numeros primos, encontre p.

Problema 21. Dado o par de primos p e 8p2 + 1, encontre p.

Problema 22. Dado o par de primos p e p2 + 2, prove que p3 + 2 tambem e um numeroprimo.

Problema 23. Dado que p, 4p2 + 1 e 6p2 + 1 sao numeros primos, encontre p.

Problema 24. Os numeros de Fermat sao os numeros da forma 22n

+ 1. Prove que oconjunto dos divisores primos dos termos da sequencia de Fermat e infinito.

Problema 25. Mostre que todo inteiro n pode ser escrito de maneira unica na forma n = ab,onde a e um inteiro livre de quadrado e b e um quadrado perfeito. Um inteiro e dito livrede quadrado se nao e divisıvel por nenhum quadrado perfeito maior que 1.

Problema 26. Prove que todo primo maior que 3 e da forma 6k+1 ou 6k+5.

Problema 27. Prove que todo inteiro da forma 3k+2 tem um fator primo da mesma forma.

Problema 28. Prove que existem infinitos primos da forma 4k+3 e 6k +5.

Problema 29. Prove que se n e composto, entao possui um fator primo p ≤ √n.

Problema 30. (OBM 1998) Sao dados 15 numeros naturais maiores que 1 e menores que1998 tais que dois quaisquer sao primos entre si. Mostre que pelo menos um desses 15numeros e primo.

Problema 31. Mostre que n|(n-1)! para todo numero composto n.

5

Page 31: OBM - Teoría de Números.pdf

POT 2012 - Teoria dos Numeros - Nıvel 2 - Aula 4 - Samuel Feitosa

Problema 32. Suponha que n >1. Mostre que a soma dos inteiros dos inteiros positivosnao excedendo n divide o produto dos inteiros positivos nao excedendo n se, e somente se,n e composto.

Exemplo 33. (Russia 1995) Encontre todos os primos p para os quais p2 +11 tenha exata-mente seis divisores distintos, incluindo 1 e p2 + 11.

Problema 34. (Irlanda 2002 ) Encontre todas as solucoes inteiras positivas de p(p+ 3) +q(q + 3) = n(n+ 3), onde p, q sao primos.

Exemplo 35. Prove que qualquer quadrado perfeito positivo tem mais divisores que deixamresto 1 na divisao por 3 do que divisores que deixam resto 2 na divisao por 3.

Dicas e Solucoes

19. Analisemos o resto de p na divisao por 3. Se p deixar resto 1, o numero 2p + 1 seradivisıvel por 3. Se p deixar resto 2, o numero 4p+ 1 sera divisıvel por 3. Em ambosos casos, 2p+ 1, 4p+ 1 > 3 e obtemos assim um absurdo.

20. Analisemos o resto de p na divisao por 3. Se p deixa resto 1 ou 2, p2 deixa resto 1e consequentemente 8p2 + 1 deixa resto 0 por 3 mas certamente e maior que 3. Umabsurdo, logo p = 3.

21. Analisemos o resto na divisao por 3. Se p nao e multiplo de 3, p2 + 2 e divisıvel por3 e maior que 3. Um absurdo, logo p = 3 e p3 + 2 = 29.

22. Analise os restos na divisao por 5.

23. Iremos usar a fatoracao do exemplo 17:

22n − 1 = (22

n−1

+ 1)(22n−2

+ 1) . . . (2 + 1)(2− 1).

Assim, se k > m,

mdc(22k

+ 1, 22m

+ 1) = mdc(22k − 1 + 2, 22

m

+ 1) = mdc(2, 22m

+ 1) = 1,

produzindo que quaisquer dois numeros de Fermat distintos sao primos entre si e issonecessariamente implica que o conjunto de seus divisores primos e infinito.

24. Analise os restos na divisao por 2 e 3.

27. Tente imitar a prova de Euclides para a existencia de infinitos primos.

29. Se n e composto, podemos escrever n = ab com 1 < a ≤ b ≤<. Assim, a2 ≤ n ea ≤ √

n. Para terminar, basta considerar qualquer divisor primo de a.

30. Dado 1 < n < 1998, se ele nao for primo, usando o exercıcio anterior, ele tem queter um fator primo menor que 1998, ou seja, um fator primo menor que 45. Como soexistem 14 primos menores que 45, e sao 15 numeros, um deles sera primo.

6

Page 32: OBM - Teoría de Números.pdf

31. Escreva n = ab e analise as aparicoes de a e b no produto (n− 1) · (n− 2) . . . 2 · 1.

33. Se p 6= 3, 3 | p2 + 11. Analogamente, se p 6= 2, 4 | p2 + 11. Assim, exceto nes-ses dois casos, 12 | p2 + 11 e podemos encontrar mais que 6 divisores distintos:{1, 2, 3, 4, 6, 12, p2+11}. Agora, teste p = 2 e p = 3 para verificar que p = 3 e a unicasolucao.

34. Sejan = 3γ · pα1

1 · · · pαnn · qβ1

1 · · · qβmm

a decomposicao de n em fatores primos, onde cada pi deixa resto 1 por 3 e cada qjdeixa resto 2 por 3. Entao

n2 = 32γ · p2α1

1 · · · p2αnn · q2β1

1 · · · q2βmm .

Um divisor de n2 deixa resto 1 por 3 se e somente se possuir uma quantidade par deprimos qj , contados com repeticao. Mais especificamente, se e somente se a soma dosexpoentes de q1, . . . , qm for par. Assim, a quantidade de divisores dessa forma e iguala

D1 = (2α1 + 1) · · · (2αn + 1)

[

1

2(2β1 + 1)(2β2 + 1) · · · (2βm + 1) + 1

]

.

Enquanto para se obter um divisor que deixe resto 2 por 3, precisamos de umaquantidade ımpar de fatores primos da forma 3k+2. Assim, a quantidade de divisoresdessa forma e:

D2 := (2α1 + 1)(2α2 + 1) · · · (2αn + 1)

(

1

2(2β1 + 1)(2β2 + 1) · · · (2βm + 1)

)

.

Daı, segue facilmente que D1 > D2.

Referencias

[1] E. Carneiro, O. Campos and F. Paiva, Olimpıadas Cearenses de Matematica 1981-2005(Nıveis Junior e Senior), Ed. Realce, 2005.

[2] S. B. Feitosa, B. Holanda, Y. Lima and C. T. Magalhaes, Treinamento Cone Sul 2008.Fortaleza, Ed. Realce, 2010.

[3] D. Fomin, A. Kirichenko, Leningrad Mathematical Olympiads 1987-1991, MathProPress, Westford, MA, 1994.

[4] D. Fomin, S. Genkin and I. Itenberg, Mathematical Circles, Mathematical Words, Vol.7, American Mathematical Society, Boston, MA, 1966.

[5] I. Niven, H. S. Zuckerman, and H. L. Montgomery, An Introduction to the Theory ofNumbers.

Page 33: OBM - Teoría de Números.pdf

Polos Olímpicos de TreinamentoCurso de Teoria dos Números - Nível 2Prof. Samuel Feitosa

Aula 5

Congruencias I

Definicao 1. Dizemos que os inteiros a e b sao congrentes modulo m se eles deixam omesmo resto quando divididos por m. Denotaremos isso por a ≡ b (mod m).

Por exemplo, 7 ≡ 2 (mod 5), 9 ≡ 3 (mod 6), 37 ≡ 7 (mod 10) mas 5 6≡ 3 (mod 4). Vejaque a ≡ b (mod m) se, e somente se, m | a− b.

Teorema 2. Se a ≡ b (mod m) e c ≡ d (mod m), entao:

i) a+ c ≡ b+ d (mod m)

ii) a− c ≡ b− d (mod m)

iii) ka ≡ kb (mod m) ∀k ∈ Z

iv) ac ≡ bd (mod m)

v) ak ≡ bk (mod m) ∀k ∈ N

vi) Se mdc(k,m) = d, entao ka ≡ kb (mod m) ⇔ a ≡ b (mod m/d)

Demonstracao. Sejam q1 e q2 tais que:

a− b = q1m

c− d = q2m

Entao, (a+ c)− (b+ d) = (q1 + q2)m. Logo, a+ c e b+ d deixam o mesmo resto por m econsequentemente a+ c ≡ b+d (mod m). Usando que a− b (mod a)k− bk e que m | a− b,concluımos que m (mod a)k − bk. Os demais itens serao deixados para o leitor.

Em termos praticos, podemos realizar quase todas as operacoes elementares envolvendoigualdade de inteiros. Uma das diferencas cruciais e a operacao de divisao como mostra oultimo item do teorema anterior.

Page 34: OBM - Teoría de Números.pdf

POT 2012 - Teoria dos Numeros - Nıvel 2 - Aula 5 - Samuel Feitosa

Exemplo 3. Calcule o resto de 4100 por 3.

Como 4 ≡ 1 (mod 3), temos 4100 ≡ 1100 = 1 (mod 3).

Exemplo 4. Calcule o resto de 4100 por 5.

Como 4 ≡ −1 (mod 5), temos 4100 ≡ (−1)100 = 1 (mod 5).

Exemplo 5. Calcule o resto de 4100 por 7.

Voce deve ter percebido que encontrar relacoes do tipo a ≡ ±1 (mod m) podem simplificarbastante o calculo de ak (mod m). Procuremos alguma relacao como essa para 4 e 7. Vejaque:

40 ≡ 1 (mod 7), 41 ≡ 4 (mod 7), 42 ≡ 2 (mod 7), 43 ≡ 1 (mod 7).

Assim,499 = (43)33 ≡ 133 = 1 (mod 7).

Como 43 ≡ 1 (mod 7), os restos das potencias de 4 na divisao por 7 se repetem periodica-mente de 3 em 3 pois 43k+r ≡ 43k · 4r ≡ 4r (mod 7).

Exemplo 6. Qual o resto de 3636 + 4141 na divisao por 77?

Inicialmente devemos perceber que existe uma relacao entre os numeros do problema: 36+41 = 77. Assim:

−36 ≡ 41 (mod 77),

(−36)41 ≡ 4141 (mod 77),

3636(1− 365) ≡ 3636 + 4141 (mod 77).

Nosso proximo passo e encontrar o resto de 365 na divisao por 77. Como 36 ≡ 1 (mod 7),365 ≡ 1 (mod 7). Alem disso, 36 ≡ 3 (mod 1)1 produzindo 365 ≡ 35 ≡ 1 (mod 1)1.Como mdc(7, 11) = 1 e ambos dividem 365 − 1, podemos concluir que 77 | 365 − 1. Logo,3636 + 4141 deixa resto 0 na divisao por 77.

Exemplo 7. Prove que p2 − 1 e divisıvel por 24 se p e um primo maior que 3.

Se p e um primo maior que 3, p ≡ ±1 (mod 3) e p ≡ 1 (mod 2). Daı, p2 ≡ 1 (mod 3).Alem disso, se p = 2k + 1, segue que p2 = 4k(k + 1) + 1 ≡ 1 (mod 8) pois k(k + 1) e par.Como mdc(8, 3) = 1 e ambos dividem p2 − 1, segue que 24 | p2 − 1.

Exemplo 8. (OCM-2001) Achar o menor natural n tal que 2001 e a soma dos quadradosde n inteiros

Podemos concluir da solucao do problema anterior que todo todo inteiro ımpar ao quadradodeixa resto 1 por 8. Usemos isso para estimar o valor de n. Sejam x1, x2, . . . , xn inteirosımpares tais que:

x21 + x22 + . . . x2n = 2001.

2

Page 35: OBM - Teoría de Números.pdf

POT 2012 - Teoria dos Numeros - Nıvel 2 - Aula 5 - Samuel Feitosa

Analisando a congruencia modulo 8, obtemos:

x21 + x22 + . . . x2n = 2001 (mod 8)

1 + 1 + . . .+ 1 ≡ 1 (mod 8)

n ≡ 1 (mod 8)

Como 2001 nao e quadrado perfeito, nao podemos ter n = 1. O proximo candidado paran seria 1 + 8 = 9. Se exibirmos um exemplo para n = 9, teremos achado o valor mınimo.Veja que:

2001 = 432 + 112 + 52 + 12 + 12 + 12 + 12 + 12 + 12.

Exemplo 9. (IMO) Seja s(n) a soma dos dıgitos de n. Se N = 44444444, A = s(N) eB = s(A). Quanto vale s(B)?

Pelo criterio de divisibilidade por 9, N ≡ A ≡ B (mod 9). Inicialmente calculemos oresto de N por 9. Como 4444 ≡ 16 ≡ 7 (mod 9), precisamos encontrar 74444 (mod 9).Seguindo os metodos dos primeiros exemplos, seria interessante encontrarmos um inteiro rtal que 7r ≡ ±1 (mod 9). O menor inteiro positivo com essa propriedade e r = 3. Como4444 = 1481 · 3 + 1, temos:

74444 ≡ 71481·3+1 ≡ (73)1481 · 7 ≡ 7 (mod 9).

Nosso proximo passo e estimar o valor de s(B). Como N = 44444444 < 105·4444, A =s(N) ≤ 5 · 4444 · 9 = 199980. Alem disso, B = s(A) ≤ 1 + 9 · 5 = 46 e s(B) ≤ 12. O unicointeiro menor ou igual a 12 com resto 7 por 9 e o proprio 7, daı s(B) = 7.

Exemplo 10. Prove que 11n+2 + 122n+1 e divisıvel por 133 para qualquer natural n.

Duas relacoes que podemos extrair dos numeros envolvidos sao: 144−11 = 133 e 133−12 =121. Assim:

144 ≡ 11 (mod 133),

122 ≡ 11 (mod 133),

122n ≡ 11n (mod 133),

122n+1 ≡ 11n · 12 (mod 133),

122n+1 ≡ 11n · (−121) + 133 · 11n (mod 133),

122n+1 ≡ −11n+2 (mod 133).

Exemplo 11. Prove que n5 + 4n e divisıvel por 5 para todo inteiro n

Inicialmente note que n5 + 4n = n(n4 + 4). Se n ≡ 0 (mod 5), nao ha o que fazer. Sen ≡ ±1 (mod 5), n4+4 ≡ 1+4 = 0 (mod 5). Finalmente, se n ≡ ±2 (mod 5), n2 ≡ 4 ≡ −1(mod 5) e consequentemente n4 + 4 ≡ 1 + 4 = 0 (mod 5).

Exemplo 12. Seja n > 6 um inteiro positivo tal que n− 1 e n+ 1 sao primos. Mostre quen2(n2 + 16) e divisıvel por 720. A recıproca e verdadeira?

3

Page 36: OBM - Teoría de Números.pdf

POT 2012 - Teoria dos Numeros - Nıvel 2 - Aula 5 - Samuel Feitosa

Veja que n e da forma 6k, pois n− 1 e n+ 1 sao primos maiores que 3, portanto da forma6k − 1 e 6k + 1, respectivamente. Logo,

n2(n2 + 16) = 144(9k4 + 4k2).

Resta provar que 9k4 +4k2 e um multiplo de 5. Vamos analisar a igualdade acima modulo5.

i) Se k ≡ 0, 2 ou 3 (mod 5), temos 9k4 + 4k2 ≡ 0 (mod 5);

ii) Se k ≡ 1 (mod 5) ⇒ n ≡ 1 (mod 5), temos n− 1 ≡ 0 (mod 5), um absurdo;

iii) Se k ≡ 4 (mod 5) ⇒ n ≡ 4 (mod 5), temos n+1 ≡ 0 (mod 5), novamente um absurdo.

Isso conclui a demonstracao. A recıproca nao e verdadeira. Basta tomar, por exemplo,n = 90.

Problemas Propostos

Problema 13. Determine o resto de 220 − 1 na divisao por 41.

Problema 14. Qual o resto de 12000 + 22000 + . . .+ 20002000 na divisao por 7?

Problema 15. Qual o resto na divisao de 270 + 370 por 13?

Problema 16. Qual o resto de 3200 por 100?

Problema 17. (Estonia 2000) Determine todos os possıveis restos da divisao de um qua-drado de um numero primo com o 120 por 120.

Problema 18. Qual o ultimo dıgito de 777777?

Exemplo 19. Prove que 22225555 + 55552222 e divisıvel por 7.

Problema 20. Prove que o numero n3 + 2n e divisıvel por 3 para todo natural n.

Problema 21. Prove que n2 + 1 nao e divisıvel por 3 para nenhum n inteiro.

Problema 22. Prove que n3 + 2 nao e divisıvel por 9 para nenhum n inteiro.

Problema 23. Prove que p2 − q2 e divisıvel por 24 se p e q sao primos maiores que 3.

Problema 24. Prove que se 2n + 1 e 3n + 1 sao ambos quadrados perfeitos, entao n edivisıvel por 40.

Problema 25. Prove que 7|22n+1 + 3n+2.

Problema 26. Seja d(n) a soma dos dıgitos de n. Suponha que n+ d(n) + d(d(n)) = 1995.Quais os possıveis restos da divisao de n por 9?

Problema 27. Prove que nao existem inteiros positivos x1, x2, . . . , x14 tais que:

x41 + x42 + . . .+ x414 = 1599.

4

Page 37: OBM - Teoría de Números.pdf

Problema 28. Escreva uma unica congruencia que e equivalente ao par de congruenciasx ≡ 1 (mod 4) e x ≡ 2 (mod 3).

Problema 29. Prove que 2015 − 1 e divisıvel por 11 · 31 · 61

Problema 30. (Alemanha 1997) Determine todos os primos p para os quais o sistema

p+ 1 = 2x2

p2 + 1 = 2y2

tem uma solucao nos inteiros x, y.

Problema 31. Mostre que se n divide um numero de Fibonacci entao ele dividira umainfinidade.

Dicas e Solucoes

13 - 31 Em breve.

Referencias

[1] E. Carneiro, O. Campos and F. Paiva, Olimpıadas Cearenses de Matematica 1981-2005(Nıveis Junior e Senior), Ed. Realce, 2005.

[2] S. B. Feitosa, B. Holanda, Y. Lima and C. T. Magalhaes, Treinamento Cone Sul 2008.Fortaleza, Ed. Realce, 2010.

[3] D. Fomin, A. Kirichenko, Leningrad Mathematical Olympiads 1987-1991, MathProPress, Westford, MA, 1994.

[4] D. Fomin, S. Genkin and I. Itenberg, Mathematical Circles, Mathematical Words, Vol.7, American Mathematical Society, Boston, MA, 1966.

[5] I. Niven, H. S. Zuckerman, and H. L. Montgomery, An Introduction to the Theory ofNumbers.

Page 38: OBM - Teoría de Números.pdf

Polos Olímpicos de TreinamentoCurso de Teoria dos Números - Nível 2Prof. Samuel Feitosa

Aula 6

Congruencias II

Na aula de hoje, aprenderemos um dos teoremas mais importantes do curso: o ”pe-queno”teorema de Fermat. Comecaremos relembrando um resultado da aula passada:

Lema 1. Se ka ≡ kb (mod m) e mdc(m, k) = 1, entao a ≡ b (mod m).

Demonstracao. Como m | k(a− b) e mdc(m, k) = 1, segue que m | a− b.

Teorema 2. (Teorema de Fermat) Seja p um primo. Se p nao divide a entao

ap−1 ≡ 1 (mod p).

Alem disso, para todo inteiro a, ap ≡ a (mod p)

Demonstracao. Considere o conjunto de inteiros B = {a, 2a, 3a, . . . , (p− 1)a} onde a e uminteiro satisfazendo mdc(a, p) = 1. Nenhum deles e divisıvel por p e quaisquer dois delessao incongruentes modulo p, em virtude do lema anterior. Assim, o conjunto dos restosdos elementos de B coincide com o conjunto dos restos nao nulos na divisao por p, a saber,{1, 2, 3, . . . , p− 1}. Portanto,

a · 2a · 3a . . . (p− 1)a ≡ 1 · 2 · 3 · . . . (p− 1) (mod p),

ap−1(p− 1)! ≡ (p− 1)! (mod p).

Podemos cancelar o termo (p− 1)! em ambos os lados pois mdc((p− 1)!, p) = 1, concluindoassim a demonstracao do teorema.

Exemplo 3. Prove quen5

5+

n3

3+

7n

15e um inteiro para todo inteiro n.

Primeiramente note quen5

5+

n3

3+

7n

15=

3n5 + 5n3 + 7n

15. Como mdc(3, 5) = 1, basta

mostrarmos que o numerador e mutiplo de 3 e 5. Pelo teorema de Fermat:

3n5 + 5n3 + 7n ≡ 5n3 + 7n ≡ 5n+ 7n = 12n ≡ 0 (mod 3),

3n5 + 5n3 + 7n ≡ 3n5 + 7n ≡ 3n+ 7n = 10n ≡ 0 (mod 5).

Page 39: OBM - Teoría de Números.pdf

POT 2012 - Teoria dos Numeros - Nıvel 2 - Aula 5 - Samuel Feitosa

Problema 4. Mostre que n7 ≡ n (mod 42), ∀n ∈ N

Pelo teorema de Fermat,

n7 ≡ n (mod 7)

n7 ≡ (n3)2 · n ≡ n2 · n = n3 ≡ n (mod 3)

n7 ≡ (n2)3 · n ≡ n3 · n = (n2)2 ≡ n2 ≡ n (mod 2)

Como 2, 3 e 7 sao primos entre si, n7 ≡ n (mod 2 · 3 · 7 = 42).

Exemplo 5. (Bulgaria 95) Encontre o numero de inteiros n > 1 para os quais o numeroa25 − a e divisıvel por n para cada inteiro a.

Se n satisfaz o enunciado, p2(p primo) nao pode dividı-lo, pois p25 − p nao e divisıvelpor p2. Assim, n e multiplo de primos diferentes. Os fatores primos de n sao fatores de225 − 2 = 2 · 32 · 5 · 7 · 13 · 17 · 241. Entretanto, n nao e divisıvel por 17 e 241 pois 325 ≡ −3(mod 17) e 325 ≡ 32 (mod 241). Seguindo o exemplo anterior, podemos usar o teorema deFermat para mostrar que a25 ≡ a (mod p) para p ∈ {2, 3, 5, 7, 13}. Portanto, n deve serigual a um dos divisores de 2 · 3 · 5 · 7 · 13 diferente de 1. A quantidade de tais divisores e25 − 1 = 31.

Exemplo 6. Prove que para cada primo p, a diferenca

111 . . . 11222 . . . 22333 . . . 33 . . . 888 . . . 88999 . . . 99− 123456789

(onde cada digito esta escrito exatamente p vezes) e multiplo de p.

Uma boa maneira de associar os numeros do problema com o teorema de Fermat e perceberque:

111 . . . 11︸ ︷︷ ︸

p uns

=10p − 1

9.

Assim, podemos escrever o numero S = 111 . . . 11222 . . . 22333 . . . 33 . . . 888 . . . 88999 . . . 99como:

S =10p − 1

9· 108p + 2 ·

10p − 1

9· 107p + . . . 9 ·

10p − 1

99S = (10p − 1) · 108p + 2 · (10p − 1) · 107p + . . . 9 · (10p − 1)

Para p = 2 ou p = 3, o resultado do enunciado segue dos criterios de divisibilidade por 2 e3. Podemos entao nos concentrar no caso p > 3. Nesse caso, e suficiente mostrarmos que9(S − 123456789) e divisıvel por p pois mdc(p, 9) = 1. Pelo teorema de Fermat:

9S = (10p − 1) · 108p + 2 · (10p − 1) · 107p + . . . 9 · (10p − 1)

≡ (10− 1) · 108 + 2 · (10− 1) · 107 + . . . 9 + ·(10− 1) (mod p)

≡ 9 · 123456789 (mod p).

2

Page 40: OBM - Teoría de Números.pdf

POT 2012 - Teoria dos Numeros - Nıvel 2 - Aula 5 - Samuel Feitosa

Exemplo 7. Dado um primo p, prove que existem infinitos naturais n tais que p divide2n − n.

Se p = 2, n pode ser qualquer numero par. Suponha que p > 2. Considere (p− 1)2k, peloteorema de Fermat temos:

2(p−1)2k ≡ (2p−1)(p−1)2k−1

≡ 1(p−1)2k−1

= 1 ≡ (p− 1)2k (mod p).

Assim, para qualquer k, n = (p− 1)2k satisfaz o problema.

Lema 8. Se mdc(a,m) = 1 entao existe um inteiro x tal que

ax ≡ 1 (mod m).

Tal x e unico modulo m. Se mdc(a,m) > 1 entao nao existe tal x.

Demonstracao. Pelo teorema de Bachet-Bezout, existem inteiros x e y tais que ax+my = 1.Analisando essa congruencia modulo m, obtemos ax ≡ 1 (mod m). Se y e outro inteiroque satisfaz a congruencia, temos ax ≡ ay (mod m). Pelo primeiro lema, x ≡ y (mod m).Se d = mdc(a,m) > 1, nao podemos ter d | m e m | ax− 1 pois d ∤ ax− 1.

Teorema 9. (Teorema de Wilson) Se p e primo, entao

(p− 1)! ≡ −1 (mod p)

Demonstracao. Em virtude do lema anterior, para cada a ∈ {2, 3, . . . , p − 2}, existe umresto x ∈ {0, 1, 2, . . . , p−1} tal que ax ≡ 1 (mod p). Se x = 1 ou x = p−1, terıamos a = 1ou p − 1. Alem disso, nao podemos ter a = x pois os unicos restos que satisfazem a2 ≡ 1(mod p) sao 1 e p − 1 (Veja o problema 20). Com isso, podemos agrupar os numeros de{2, 3, . . . , p − 2} em pares onde o produto deixa resto 1 por p, o que nos permite concluirque o produto de todos eles tambem deixa resto 1 por p. Logo,

(p− 1)! ≡ 1 · (p− 1) ≡ −1 (mod p).

Exemplo 10. (Estonia 2000) Prove que nao e possıvel dividir qualquer conjunto de 18inteiros consecutivos em dois conjuntos disjuntos A e B tais que o produtos dos elementosde A seja igual ao produto dos elementos de B.

Suponha, por absurdo, que existam tais conjuntos. Considere o primo p = 19. Como oprodutos dos elementos de A e igual ao produtos dos elementos de B, se um dos conjuntoscontem um multiplo de 19, o outro necessariamente tambem contera. Como entre 18inteiros consecutivos nao existem dois multiplos de 19, nenhum dos conjuntos do problemacontem tais numeros. Seja x o resto na divisao por 19 dos produtos dos elementos de A.Calculemos entao o resto na divisao por 19 do produto de todos os 18 inteiros consecutivos:

x · x ≡ n(n+ 1)(n+ 2)(n+ 3) . . . (n+ 17)

≡ 1 · 2 · 3 . . . · 18

≡ −1 (mod 19)(Pelo teorema de Wilson).

Como x2 ≡ −1 (mod 19), x18 ≡ (−1)9 ≡ 1 (mod 1)9. Isso contraria o teorema de Fermate obtemos um absurdo.

3

Page 41: OBM - Teoría de Números.pdf

POT 2012 - Teoria dos Numeros - Nıvel 2 - Aula 5 - Samuel Feitosa

Definicao 11. Um conjunto S e chamado de sistema completo de resıduos modulo n, de-notado abreviadamente por scr, se para cada 0 ≤ i ≤ n − 1, existe um elemento de s ∈ S

tal que i ≡ s (mod n). Para qualquer a, o conjunto {a, a+ 1, a+ 2, . . . , a+ (n− 1)} e umexemplo de scr.

Exemplo 12. Se mdc(m, s) = 1, mostre que {t, t+ s, t+ 2s, . . . t+ (m− 1)s} e um scr.

Pelo primeiro lema, se t+ is ≡ t+ js (mod m), temos is ≡ js (mod m) e i ≡ j (mod m).Como i, j ∈ {0, 1, . . . ,m − 1}, i = j. Isso nos diz que temos m inteiros que deixam restosdistintos na divisao porm. Como existem exatamentem restos na divisao porm, o conjuntoe um scr.

Exemplo 13. Seja m um inteiro positivo par. Suponha que {a1, a1, . . . , am} e {b1, b2, . . . , bm}sao dois sistemas completos de resıduso modulo m. Prove que

S = {a1 + b1, a2 + b2, . . . , am + bm}

nao e um sistema completo de resıduos.

Suponha que S seja um scr, entao:

1 + 2 + . . .+m ≡ (a1 + b1) + (a2 + b2) + . . .+ (an + bn) (mod m)

≡ (a1 + a2 + . . .+ an) + (b1 + b2 + . . .+ bn)

≡ 2(1 + 2 + . . .+ n)

≡ 2(1 + 2 + . . .+m)

Isso implica que m |m(m+ 1)

2, ou seja,

m+ 1

2e inteiro. Um absurdo pois m e par.

Exemplo 14. (Polonia 1997) Prove que a sequencia an definida por a1 = 1 e

an = an−1 + a⌊n

2

contem infinitos termos divisıveis por 7.

Uma maneira natural para mostrarmos que existem infinitos inteiros multiplos de 7 nasequencia e verificar que o aparecimento de um multiplo de 7 acarreta o aparecimento deoutro multiplo na sequencia com um ındice maior. Suponha que ak e multiplo de 7. Sejaa2k−1 = s. Entao:

a2k−1 = s

a2k = s+ ak ≡ s (mod 7)

a2k+1 = a2k + ak ≡ s (mod 7)

4

Page 42: OBM - Teoría de Números.pdf

POT 2012 - Teoria dos Numeros - Nıvel 2 - Aula 5 - Samuel Feitosa

Ou seja, o aparecimento de um inteiro multiplo de 7 implica no aparecimento de 3 inteiroscom o mesmo resto por 7. Exploremos essa ideia mais uma vez.

a4k−3 = t

a4k−2 ≡ t+ a2k−1 ≡ t+ s (mod 7)

a4k−1 ≡ t+ s+ a2k−1 ≡ t+ 2s (mod 7)

a4k ≡ t+ 2s+ a2k ≡ t+ 3s (mod 7)

a4k+1 ≡ t+ 3s+ a2k ≡ t+ 4s (mod 7)

a4k+2 ≡ t+ 4s+ a2k+1 ≡ t+ 5s (mod 7)

a4k+3 ≡ t+ 5s+ a2k+2 ≡ t+ 6s (mod 7)

Se s e multiplo de 7, ja teremos conseguido outro multiplo de 7 na sequencia. Em casocontrario, o conjunto {t, t+ s, t+ 2s, . . . , t+ 6s} e um scr e contera um multiplo de 7.

Exemplo 15. Sejam x, y inteiros. Prove que 3x2 + 4y2 e 4x2 + 3y2 nao podem ser ambosquadrados perfeitos.

Comecemos com um lema bastante util:

Lema 16. Seja p um numero primo da forma 4k + 3. Entao

p | m2 + n2 ⇐⇒ p | m e p | n.

Facamos inicialmente a primeira implicacao. Se p ∤ m, entao mp−1 ≡ 1 (mod p), e daıtemos as equivalencias modulo p

n2 ≡ −m2

⇒ (nmp−2)2 ≡ −(mp−1)2

≡ −1

⇒ (nmp−2)p−1 ≡ (−1)p−1

2

≡ (−1)2k+1

≡ −1,

o que contraria o teorema de Fermat. Assim, p | m e p | n.

A recıproca e obvia. Voltando ao problema, suponha que existam w, z inteiros positivostais que

3x2 + 4y2 = w2 e

4x2 + 3y2 = z2.

Entao 7x2+7y2 = w2+z2 (∗). Afirmamos que a equacao (∗) nao possui solucao. Para isso,seja S o conjunto formado pelas solucoes inteiras (x, y, w, z) de (∗), e tome (a, b, c, d) ∈ S

5

Page 43: OBM - Teoría de Números.pdf

POT 2012 - Teoria dos Numeros - Nıvel 2 - Aula 5 - Samuel Feitosa

com c2 + d2 mınimo. Pelo lema, temos que 7|c e 7|d, e daı c = 7c′ e d = 7d′. Mas entaoa2 + b2 = 7c′2 + 7d′2 ⇒ (c′, d′, a, b) ∈ S, com

a2 + b2 < 7(a2 + b2) = c2 + d2,

o que contraria a minimalidade de (a, b, c, d).

Problemas Propostos

Problema 17. Prove que se p e primo entao

ap ≡ bp (mod p) ⇒ ap ≡ bp (mod p2)

Problema 18. Encontre os restos da divisoes de:

a) 3003000 − 1 por 1001

b) 7120 − 1 por 143

Problema 19. Encontre o resto de 111 . . . 11︸ ︷︷ ︸

p−1 uns

por p, onde p e um primo maior que 5.

Problema 20. Prove que se n e ımpar, entao n5 ≡ n (mod 240).

Problema 21. Sejam p e q primos distintos. Mostre que

i) (a+ b)p ≡ ap + bp (mod p)

ii) pq + qp ≡ p+ q (mod pq)

iii)

⌊pq + pq

pq

e par se p, q 6= 2.

Problema 22. Mostre que se p e primo e a2 ≡ b2 (mod p), entao a ≡ ±b (mod p).

Problema 23. Encontre os ultimos tres dıgitos de 79999

Problema 24. Prove que 2015 − 1 e divisıvel por 11 · 31 · 61

Problema 25. Sejam {a1, a2, ..., a101} e {b1, b2, ..., b101} sistemas completos de resıduosmodulo 101. Pode {a1b1, a2b2, ..., a101b101} ser um sistema completo de resıduos modulo101?

Problema 26. (Balcanica 2003) Existe um conjunto B de 4004 inteiros positivos tal que,para cada subconjunto A de B com 2003 elementos, a soma dos elementos em A nao edivisıvel por 2003?

Problema 27. Para um inteiro ımpar n > 1, seja S o conjunto de inteiros x,1 ≤ x ≤ n,tal que ambos x e x + 1 sao relativamente primos com n. Mostre que o produto de todosos elementos de S deixa resto 1 na divisao por n.

6

Page 44: OBM - Teoría de Números.pdf

POT 2012 - Teoria dos Numeros - Nıvel 2 - Aula 5 - Samuel Feitosa

Problema 28. Sejam n um inteiro positivo maior que 1 e p um primo positivo tal que n

divide p− 1 e p divide n3 − 1. Mostre que 4p− 3 e um quadrado perfeito.

Dicas e Solucoes

17. Pelo teorema de Fermat, a ≡ ap ≡ bp ≡ b (mod p). Assim,

ap−1 + ap−2b+ . . .+ abp−2 + bp−1 ≡ ap−1 + ap−1 + . . .+ ap−1

≡ pap−1

≡ 0 (mod p)

Como a− b ≡ 0 (mod p), temos:

ap − bp = (a− b)(ap−1 + ap−2b+ . . .+ abp−2 + bp−1) ≡ 0 (mod p2)

19. Veja que:

111 . . . 11︸ ︷︷ ︸

p−1 uns

=999 . . . 99

9

=10p−1 − 1

9

Pelo teorema de Fermat, o numerador 10p−1 − 1 e divisıvel por p visto que p 6= 5.Alem disso, usando que p 6= 2 e 3, segue que 10p−1

−19 tambem e multiplo de p.

20. Proceda como no exemplo 20.

21. i)Pelo teorema de Fermat:

(a+ b)p ≡ a+ b

≡ ap + bp (mod p).

ii) Pelo teorema de Fermat,

pq + qp ≡ 0 + q ≡ p+ q (mod p)

pq + qp ≡ p+ 0 ≡ p+ q (mod q)

22. Veja que (a− b)(a+ b) ≡ 0 (mod p) e assim a− b ≡ 0 (mod p) ou a+ b ≡ 0 (mod p).

25. Suponha, por abusurdo, que seja possıvel. Sejam ai e bj tais que ai ≡ bj ≡ 0(mod 101). Se i 6= j, o conjunto {a1b1, a2b2, ..., a101b101} teria dois inteiros com resto

7

Page 45: OBM - Teoría de Números.pdf

POT 2012 - Teoria dos Numeros - Nıvel 2 - Aula 5 - Samuel Feitosa

0 na divisao por p e nao poderia ser um scr. Suponha sem perda de generalidade quei = j = 101, entao:

100! ≡ (a1b1)(a2b2) . . . (a100b100)

≡ (a1a2 . . . a100)(b1b2 . . . b100)

≡ (100!)(100!)

≡ (100!)2 (mod 101)

Assim, 100! ≡ 1 (mod 101). Isso contradiz o teorema de Wilson.

26. Sim. Um exemplo de tal conjunto e a uniao de um conjunto de 2002 inteiros positivosque deixem resto 0 com outro conjunto composto por 2002 inteiros que deixem resto1 por 2003.

8

Page 46: OBM - Teoría de Números.pdf

Polos Olímpicos de TreinamentoCurso de Teoria dos Números - Nível 2Prof. Samuel Feitosa

Aula 7

Aula de Revisao e Aprofundamento

Observacao 1. E recomendavel que o professor instigue seus alunos a pensarem nos pro-blemas abaixo antes de resolve-los na aula.

Exemplo 2. (ASHME 1990) Para quantos inteiros N entre 1 e 1990 a fracaoN2 + 7

N + 4nao

e irredutıvel?

Seja d = (N +4, N2 +7). Um boa estrategia e procurar um multiplo de N +4 proximo deN2+7 pois assim conseguiremos estimar d. Usando a diferenca de quadrados, d | N2−16 econsequentemente d | N2+7−(N2−16) = 23. Como 23 e primo, a fracao nao sera irredutıvelapenas quando d = 23. Para isso acontecer, basta que 23 | N+4 pois N2+7 = N2−16+23.O maior multiplo de 23 menor que 1990 e 1978 = 23 · 86 e 1990+ 4 < 23 · 87. Sendo assim,a quantidade de inteiros procurada e 86.

Exemplo 3. Dados os primos p e q satisfazendo:

q | p2 + 1 e p | q2 − 1.

Prove que o numero p+ q + 1 e composto.

Como p | q2−1 = (q+1)(q−1), temos que p | q+1 ou p | q−1. No primeiro caso, p+q+1e um multiplo de p. No segundo caso, podemos escrever q − 1 = pk para algum natural k.Usando que q | p2 + 1, concluımos que q | p2 + 1 − (pk + 1) = p(p − k). Como p e q naopodem ser primos iguais, q | p− k. Temos tres casos a considerar:

1. p > k. Entao:

p− k ≥ kp+ 1,

(p+ 1)(1− k) ≥ 2

Page 47: OBM - Teoría de Números.pdf

2. p < k. Entao:

k − p ≥ kp+ 1,

(k + 1)(1− p) ≥ 2

3. p = k e q = p2+1. Como o unico primo par e 2, segue que p = 2, q = 5 e p+q+1 = 8.

Os dois primeiros casos conduzem a um absurdo. Logo, ou p+ q + 1 e multiplo de p ou eigual a 8.

Exemplo 4. (AIME 1985) Os numeros da sequencia

101, 104, 109, 116, . . .

sao da forma an = 100 + n2, onde n = 1, 2, 3, . . . Para cada n, seja dn o maximo divisorcomum de an e an+1. Encontre o valor maximo de dn quando n varia sobre todo o conjuntodos inteiros positivos.

Uma boa estrategia e buscar alguma fatoracao que nos permita identificar fatores comunsentre os termos da sequencia. Um termo generico da sequencia possui a forma an =k + n2. Sendo assim, a2k = k(4k + 1),a2k+1 = (k + 1)(4k + 1) e consequentementemdc(a2k, a2k+1) = 4k + 1. Nosso proximo passo sera mostrar que realmente esse e o valormaximo. Considere dois termos genericos a = an = k + n2, b = an+1 = k + (n+ 1)2 e sejad = mdc(a, b). Usando que d | b− a = 2n+1, obtemos d | (2n+1)(b− a) = 4n2− 1. Comod | 4a = 4n2 + 4k, segue que d | 4n2 + 4k − (4n2 − 1) = 4k + 1. Assim, 4k + 1 e realmenteo maior valor possıvel entre os termos da sequencia dn.

Exemplo 5. Prove que para qualquer inteiro n > 1, o numero n5+n4+1 nao e um numeroprimo.

Considere a fatoracao:

n5 + n4 + 1 = n5 + n4 + n3 − n3 − n2 − n+ n2 + n+ 1

= n3(n2 + n+ 1)− n(n2 + n+ 1) + (n2 + n+ 1)

= (n2 + n+ 1)(n3 − n+ 1)

Como n > 1, n3 − n+ 1 > 1 e obtemos assim o produto de dois inteiros maiores que 1.

Exemplo 6. (Olimpıada Grega) Encontre todos os inteiros n para os quais −54 +55 +5n eum quadrado perfeito.

Como −54 + 55 = 2500, queremos encontrar m e n tais que:

5n = m2 − 2500 = (m− 50)(m+ 50).

Isto implica que m+ 50 = 5j e m− 50 = 5i, com i < j. Assim,

100 = 5i(5j−i − 1).

Usando a fatoracao em primos de 100, encontramos que i = 2 e j−i = 1. Portanto, m = 75e n = 5.

2

Page 48: OBM - Teoría de Números.pdf

Exemplo 7. (Irlanda) Sejam p um numero primo, a e n e inteiros positivos. Prove que se

2p + 3p = an,

entao n = 1.

Se p = 2, claramente a = 13 e n = 1. Se p > 2, p e ımpar e 5 | 2p + 3p. Consequentemente5 divide a. Se fosse n > 1, 25 | an e terıamos:

0 ≡ an

5

≡ 2p + 3p

5= 2p−1 − 2p−2 · 3 + . . .+ 2 · −3p−2 + 3p−1

≡ 2p−1 + 2p−1 + . . .+ 2p−1

≡ p2p−1 (mod 5)

A unica possıbilidade e termos p = 5. Entretanto, 25 +35 nao e uma potencia perfeita naotrivial. Logo, n = 1.

Exemplo 8. Um inteiro n > 1 tem a seguinte propriedade: para todo divisor positivo d den, d+ 1 e um divisor de n+ 1. Prove que n e primo.

Seja p o menor fator primo de n e seja d =n

p. Entao,

np+ p

n+ p=

p(n+ 1)

p(d+ 1)=

n+ 1

d+ 1

e um numero inteiro. Como n + p tambem divide p(n + p), podemos concluir que n + p |(np+ p2)− (np+ p) = p2 − p. Em particular,

n+ p ≤ p2 − p

n ≤ p2 − 2p

n ≤ p2 − 2p+ 1 = (p− 1)2

n < p2

d < p.

Em virtude da minimalidade de p, d nao possui fatores primos e consequentemente n = p.

Exemplo 9. (Olimpıada Russa) Mostre que qualquer natural pode ser escrito como a dife-renca de dois numeros naturais tendo o mesmo numero de fatores primos.

Se n e par, podemos escreve-lo como 2n − n e e facil verificar que 2n e n possuem omesmo numero de fatores primos. Seja d o menor primo ımpar que nao divide n. Escrevan = dn− (d− 1)n. O termo dn contem exatamente um primo a mais que n. Pela escolhade d, todos os outros fatores primos diferentes 2 do numero d−1 sao divisores de n e assim(d− 1)n tambem contem extamente um primo a mais que n, a saber, o primo 2.

3

Page 49: OBM - Teoría de Números.pdf

Exemplo 10. Os numeros naturais a e b sao tais que

a+ 1

b+

b+ 1

a

e um numero inteiro. Mostre que o maximo divisor comun de a e b nao e maior que√a+ b.

Seja d = mdc(a, b), com a = md e b = nd. Entao:

md+ 1

nd+

nd+ 1

md=

m2d+m+ n2d+ n

mnd

e um inteiro. Em particular, d | m2d+m+ n2d+ n e consequentemente d | m+ n. Daı,

d ≤ m+ n√d ≤

√m+ n

d ≤√

d(m+ n)

=√a+ b

Exemplo 11. Encontre todos os conjuntos A ⊆ N de pelo menos dois elementos tais que

x, y ∈ A =⇒ x+ y

mdc(x, y)∈ A.

Vamos dividir o problema em dois casos: Primeiro caso: Se 1 6∈ A.

Mostremos que nesse caso devemos ter A = {2, 3, . . .}. Facamos isso seguindo as seguintesafirmacoes:

1. 2 ∈ A.Para ver isso, basta tomar dois elementos iguais.

2. Existe elemento ımpar em A.

Suponha, por absurdo, que nao existe elemento ımpar emA. Seja 2k o menor elementode par de A maior que 2. Logo,

k + 1 =2k + 2

mdc(2k, 2)∈ A.

Como k + 1 < 2k, k + 1 e ımpar. Absurdo! Note que todos os ımpares maiores quek + 1 pertencem a A. Para tal, basta escolhermos 2 e k + 1 para obtermos k + 3 eaplicar isso sucessivamente.

3. 3 ∈ A.

4

Page 50: OBM - Teoría de Números.pdf

Tome 2l − 1 > 2k + 1. Assim,

(2l − 1)(2k + 1), (2k + 1) ∈ A ⇒ (2l − 1)(2k + 1) + (2k + 1)

2k + 1= 2l ∈ A;

2l − 1, 2l + 1 ∈ A ⇒ 2l − 1 + 2l + 1

mdc(2l − 1, 2l + 1)= 4l ∈ A;

2l, 4l ∈ A ⇒ 4l + 2l

mdc(4l, 2l)= 3 ∈ A

Isso mostra que todos os ımpares maiores ou iguais a 3 estao presentes. Para isso,basta tomar cada ımpar e o 2.

4. Todos os numeros pares estao em A.Para isso, tome 2k − 1 e (2k − 1)2:

(2k − 1)2 + (2k − 1)

2k − 1= 2k ∈ A,

para todo o k ≥ 2.

Segundo caso: Se 1 ∈ A.

Nesse caso, afirmamos que A = N. Veja que:

1 ∈ A ⇒ 1 + 1

1= 2 ∈ A

x ∈ A ⇒ x+ 1

mdc(1, x)= x+ 1 ∈ A

Sendo assim, repetindo esse processo, seguira por inducao que A = N.

Exemplo 12. (Olimpıada Matematica Argentina) Sejam p1, p2, . . . , pn numeros primos.Bruno deve escolher n + 1 inteiros positivos que utilizem apenas estes primos em sua de-composicao. Bernardo deve escolher alguns desses numeros de modo que o produto delesseja um quadrado perfeito. Determine se e possıvel, para algum n, que Bruno escolha seusn+ 1 numeros de maneira que Bernardo nao consiga cumprir seu objetivo.

Vamos mostrar que Bernardo sempre consegue cumprir seu objetivo. Para decidirmos seum numero natural e quadrado perfeito, basta analisarmos a paridade dos expoentes de seusfatores primos. Para cada numero pm1

1pm2

2. . . pmn

n escolhido por Bruno, associe a n-upla dezeros e uns, (r1, r2, . . . , rn), onde cada ri e o resto na divisao por 2 de mi. A multiplicacaode dois inteiros se traduz na soma modulo 2 de tais uplas, i.e., a n-upla associada aoproduto pm1

1pm2

2. . . pmn

n ·pl11pl22. . . plnn e igual a (r1+q1 (mod 2), r2+q2 (mod 2), . . . , rn+qn

(mod 2)) onde ri e qi sao os restos na divisao por 2 de mi e li, respectivamente. Nossoproblema pergunta se e possivel Bernardo encontrar algumas uplas que somadas deem aupla (0, 0, 0, . . . , 0). Como temos n + 1 uplas, podemos formar 2n+1 − 1 > 2n somas desubconjuntos nao vazios de uplas. Cada soma corresponde a uma nova upla, como existem

5

Page 51: OBM - Teoría de Números.pdf

apenas 2n tipos de uplas distintas, alguma delas se repetira dentre as somas (pelo princıpioda casa dos pombos). Suponha que para dois conjuntos de uplas A e B tenhamos a mesmaupla associada como soma, entao a soma dos elementos de A e B que nao pertencem aA ∩B corresponde a upla (0, 0, . . . , 0).

Problemas Propostos

Problema 13. Sejam p > 2 um primo ımpar e n um inteiro positivo. Prove que p divide

1pn

+ 2pn

+ . . .+ (p− 1)pn

.

Problema 14. (Olimpıada Romena) Sejam a, b, c, d inteiros nao nulos com a 6= c e tais que

a

c=

a2 + b2

c2 + b2.

Prove que a2 + b2 + c2 nao pode ser um numero primo.

Problema 15. Encontre todos os n tais que n! e um quadrado perfeito.

Problema 16. (Hungria) O produto de alguns primos e dez vezes maior que a sua soma.Quais sao esses primos?(nao necessariamnete distintos).

Problema 17. Qual o maximo divisor comum entre dois numeros de Fibonacci consecuti-vos?Observacao: Os numeros de Fibonacci sao os numeros da sequencia definida por F1 = F2 =1 e Fn+1 = Fn + Fn−1.

Problema 18. Mostre que a soma de dois primos consecutivos nunca e o dobro de umprimo.

Problema 19. (Israel) Se Sn e a soma dos n primeiros numeros primos, prove que ha aomenos um quadrado perfeito entre Sn e Sn+1

Problema 20. (Olimpıada Balcanica) Prove que, para todo natural dado n, existe um na-tural m > n tal que a representacao decimal de 5m e obtida da representacao decimal de5n pelo acrescimo de algarismos a esquerda de 5n.

Problema 21. (Inglaterra 1995)

a) Encontre o primeiro inteiro positivo cujo quadrado termina em tres quatros.

b) Encontre todos os inteiros positivos cujo quadrado termina em tres quatros.

c) Mostre que nenhum quadrado perfeito termina em quatro quatros.

6

Page 52: OBM - Teoría de Números.pdf

Exemplo 22. (Olimpıada Indiana) Seja n um inteiro positivo tal que n e um divisor dasoma

1 + (1n−1 + 2n−1 + . . .+ (n− 1)n−1).

Prove que n nao e divisıvel por qualquer quadrado maior que 1.

Problema 23. Seja S um conjunto de primos tal que a, b ∈ S (a e b nao precisam serdistintos) implicam ab+ 4 ∈ S. Mostre que S tem que ser vazio

Dicas e Solucoes

1. Em breve!

7

Page 53: OBM - Teoría de Números.pdf

Polos Olímpicos de TreinamentoCurso de Teoria dos Números - Nível 2Prof. Samuel Feitosa

Aula 8

Equacoes Diofantinas I

Exemplo 1. Em Gugulandia, o jogo de basquete e jogado com regras diferentes. Existemapenas dois tipo de pontuacoes para as cestas: 5 e 11 pontos. E possıvel um time fazer 39pontos em uma partida?

Sejam x e y os numeros de cestas de 5 e 11 pontos, respectivamente. O problema se resumeem descobrirmos se existem inteiros nao negativos x e y tais que 5x+ 11y = 39. Ao invesde testarmos os valores de x e y, somemos 11 + 5 em ambos os lados da equacao:

5(x+ 1) + 11(y + 1) = 55.

Como 5 | 55 e 5 | 5(x + 1), segue que 5 | 11(y + 1) e, com mais razao, 5 | y + 1 poismdc(5, 11) = 1. Do mesmo modo, 11 | x+ 1. Assim,

55 = 5(x+ 1) + 11(y + 1) ≥ 5 · 11 + 11 · 5 = 110,

pois x+ 1, y + 1 ≥ 1. Obtemos uma contradicao.

Exemplo 2. Qual o menor inteiro positivo m para o qual todo numero maior que m podeser obtido como pontuacao no jogo de basquete mencionado anteriormente?

Como ja sabemos que 39 nao e possıvel, e natural comecarmos procurando os numerosmaiores que 39 que nao podem ser pontuacoes. Veja que:

40 = 5 · 8 + 11 · 0

41 = 5 · 6 + 11 · 1

42 = 5 · 4 + 11 · 2

43 = 5 · 2 + 11 · 3

44 = 5 · 0 + 11 · 4

Page 54: OBM - Teoría de Números.pdf

Ao somarmos 5 a cada uma dessas representacoes, obteremos representacoes para os proximos5 numeros. Repetindo esse argumento, poderemos escrever qualquer numero maior que 39na forma 5x + 11y com x e y inteiros nao negativos. Concluımos assim que m = 39. Po-derıamos mostrar que todo numero maior que 44 e da forma 5x + 11y com x e y inteirosnao negativos de outro modo. Se n > 44, considere o conjunto:

n− 11 · 0, n− 11 · 1, n− 11 · 2, n− 11 · 3, n− 11 · 4.

Como mdc(11, 5) = 1, o conjunto anterior e um sistema completo de restos modulo 5 econsequentemente existe y ∈ {0, 1, 2, 3, 4} tal que

n− 11 · y = 5x

Como n > 44, segue que x > 0.

Exemplo 3. Quais e quantos sao os inteiros positivos n que nao podem ser obtidos comopontuacao nesse jogo de basquete?

Precisaremos relembrar um teorema da aula 03:

Teorema 4. (Bachet-Bezout) Se d = mdc(a, b), entao existem inteiros x e y tais que

ax+ by = d.

A primeira observacao que fazemos e que uma vez encontrados inteiros x e y, qualquermultiplo de d pode ser representado como uma combinacao linear de a e b:

a(kx) + b(ky) = kd.

Isso e particularmente interessante quando mdc(a, b) = 1, onde obtemos que qualquer in-teiro e uma combinacao linear de a e b. Veja que isso nao entra em conflito com os exemplosanteriores pois os inteiros x e y mencionados no teorema podem ser negativos.

A proxima propopsicao contera o que procuramos:

Proposicao 5. Todo inteiro positivo k pode ser escrito(de modo unico) de uma e, somenteuma, das seguintes formas:

11y − 5x, ou 11y + 5x, com 0 ≤ y < 5 e x ≤ 0

Pelo teorema de Bachet-Bezout, existem m e n tais que 5m + 11n = 1. Sejam q e r oquociente e resto da divisao de kn por 5, i.e., kn = 5q + r, 0 ≤ r < 5. Assim,

k = 5(km) + 11(kn)

= 5(km) + 11(5q + r)

= 5(km+ 11q) + 11r.

2

Page 55: OBM - Teoría de Números.pdf

Basta fazer x = km+ 11q e r = y.

Para ver a unicidade, suponha que 11m ± 5n = 11a ± 5b com 0 ≤ m, a < 5. Entao11(m − a) = 5(±b ± n). Usando que mdc(11, 5) = 1, segue que 5 | m − a. A unica opcaoe termos m = a pois o conjunto {0, 1, 2, 3, 4} e um scr. Consequentemente ±5n = ±5b en = b.

Sendo assim, os elementos do conjunto

B(5, 11) = {11y − 5x ∈ Z∗

+; 0 ≤ y < 5 e x > 0}

constituem o conjunto das pontuacoes que nao podem ser obtidas. Seus elementos sao:

y = 1 ⇒ 11y − 5x = 1, 6

y = 2 ⇒ 11y − 5x = 2, 7, 12, 17

y = 3 ⇒ 11y − 5x = 3, 8, 13, 18, 23, 28

y = 4 ⇒ 11y − 5x = 4, 9, 14, 19, 24, 29, 34, 39

A quantidade de tais inteiros e

20 =(5− 1)

2·(11− 1)

2.

Vale o resultado geral:

Proposicao 6. Dados os inteiros positivos a e b com mdc(a, b) = 1, existem exatamente

(a− 1)

2·(b− 1)

2

numeros inteiros nao negativos que nao sao da forma ax+ by com x, y ≥ 0.

Provaremos tal resultado em uma aula futura fazendo o uso da funcao parte inteira.

Exemplo 7. Suponha agora que as pontuacoes das cestas do basquete de Gugulandia tenhammudado para a e b pontos com 0 < a < b. Sabendo que existem exatamente 35 valoresimpossıveis de pontuacoes e que um desses valores e 58, encontre a e b.

Perceba que devemos ter mdc(a, b) = 1 pois caso contrario qualquer valor que nao fossemultiplo de mdc(a, b) nao seria uma pontuacao possıvel e sabemos que existe apenas umnumero finito de tais valores. Em virtude da proposicao anterior, (a−1)(b−1) = 2·35 = 70.Analisemos os possıveis pares de divisores de 70 tendo em mente que a < b:

(a− 1)(b− 1) = 1 · 70 ⇒ (a, b) = (2, 71)

(a− 1)(b− 1) = 2 · 35 ⇒ (a, b) = (3, 36)

(a− 1)(b− 1) = 5 · 14 ⇒ (a, b) = (6, 15)

(a− 1)(b− 1) = 7 · 10 ⇒ (a, b) = (8, 11)

3

Page 56: OBM - Teoría de Números.pdf

Nao podemos ter (a, b) = (2, 71) pois 58 = 2 · 29. Excluindo os outros dois casos em quemdc(a, b) 6= 1, temos a = 8 e b = 11.

A equacao ax + by = c e um exemplo de uma equancao diofantina, i.e., uma equacaoem que buscamos valores inteiros para as variaveis. Tais equacoes recebem esse nome emhomenagem ao matematico grego Diofanto.

Exemplo 8. Determine todas as solucoes inteiras da equacao 2x+ 3y = 5.

Por paridade, 3y e ımpar, donde y = 2k + 1 para algum inteiro k. Daı,

x =5− 3(2k + 1)

2= 1− 3k,

e consequentemente todas as solucoes da equacao sao da forma (x, y) = (1− 3k, 2k + 1).

Exemplo 9. Determine todas as solucoes inteiras da equacao 5x+ 3y = 7.

Analisando agora modulo 3, 5x ≡ 7 ≡ 1 (mod 3). Essa condicao impoe restricoes sobre oresto de x na divisao por 3. Dentre os possıveis restos na divisao por 3, a saber {0, 1, 2}, ounico que satisfaz tal congruencia e o resto 2. Sendo assim, x e da forma 3k + 2 e

y =7− 5(3k + 2)

3= −1− 5k,

consequentemente, todas as solucoes da equacao sao da forma (x, y) = (3k + 2,−1− 5k).

Notemos que para a solucao da congruencia x = 2, obtemos a solucao (x, y) = (2, 1) daequacao. Baseado nesses exemplos, e natural imaginarmos que conhecendo uma solucao dacongruencia consigamos descrever todas as outras.

Teorema 10. A equacao ax+by = c, onde a, b, c sao inteiros, tem uma solucao em inteiros(x, y) se, e somente se, d = mdc(a, b) divide c. Nesse caso, se (x0, y0) e uma solucao, entaoos pares

(xk, yk) =

(

x0 +bk

d, y0 −

ak

d

)

, k ∈ Z

sao todas as solucoes inteiras da equacao.

Dada a discussao anterior, resta apenas encontrarmos a forma das solucoes. Se (x, y) eoutra solucao, podemos escrever:

ax+ by = ax0 + by0

a(x− x0) = b(y0 − y)

a

d(x− x0) =

b

d(y0 − y)

Como mdc(a/d, b/d) = 1, temos b/d | x − x0 e assim podemos escrever x = x0 + bk/d.Substituindo na equacao original obtemos y = y0 − ak/d.

4

Page 57: OBM - Teoría de Números.pdf

Exemplo 11. Encontre todas as solucoes inteiras da equacao 21x+ 48y = 6

O sitema e equivalente a 7x + 16y = 2. Uma solucao e (x, y) = (−2, 1). Pelo teoremaanterior, todas as solucoes sao da forma:

(xk, yk) = (−2 + 16k, 1− 7k).

Exemplo 12. Resolva nos inteiros a equacao 2x+ 3y + 5z = 11

Podemos transformar esse problema isolando qualquer uma das variaveis no problema queja sabemos resolver. Por exemplo, podemos resolver 2x + 3y = 11 − 5z. Supondo z fixo,podemos encontrar a solucao particular (x, y) = (4 − z, 1 − z). Assim, todas as solucoessao da forma:

(x, y) = (4− z + 3k, 1− z − 2k),

ou seja, as solucoes da equacao original sao da forma (x, y, z) = (4− z + 3k, 1− z − 2k, z)com k e z inteiros.

Vamos estudar agora alguns outros exemplos de equacoes diofantinas nao lineares:

Exemplo 13. Prove que a equacao 2n + 1 = q3 nao admite solucoes (n, q) em inteirospositivos.

E facil ver que a equacao nao admite solucoes se n = 1, 2, 3. Assim, podemos supor quen > 3. Fatorando, temos:

(q − 1)(q2 + q + 1) = 2n,

e consequentemente q = 2 ou q = 2k+1, para algum k ∈ N. Claramente, q = 2 nao produzsolucao. Entao q = 2k + 1 e q3 − 1 = 8k3 + 12k2 + 6k e uma potencia de 2, maior ou iguala 16. Entretanto:

8k3 + 12k2 + 6k = 2k(4k2 + 6k + 3),

nao e uma potencia de 2, pois 4k2 + 6k + 3 e ımpar. Assim, a equacao 2n + 1 = q3 naoadmite solucoes inteiras positivas.

Exemplo 14. (URSS 1991) Encontre todas as solucoes inteiras do sistema

{

xz − 2yt = 3xt+ yz = 1.

Uma boa estrategia sera aplicar alguma manipulacao algebrica, como somar as equacoes,multiplica-las, somar um fator de correcao, entre outras para obtermos alguma fatoracaoenvolvendo esses numeros. Nesse problema, vamos elevar ambas as equacoes ao quadrado.

{

x2z2 − 4xyzt+ 4y2t2 = 9x2t2 + 2xytz + y2z2 = 1.

Multiplicando a segunda por dois e somando com a primeira, temos:

x2(z2 + 2t2) + 2y2(z2 + 2t2) = 11

(x2 + 2y2)(z2 + 2t2) = 11.

5

Page 58: OBM - Teoría de Números.pdf

Como cada uma das parcelas acima e um inteiro nao-negativo, temos dois casos:

{

x2 + 2y2 = 11z2 + 2t2 = 1

⇒ (x, y, z, t) = (±3,±1,±1, 0).

ou{

x2 + 2y2 = 1z2 + 2t2 = 11

⇒ (x, y, z, t) = (±1, 0,±3,±1).

Logo, as unicas solucoes possıveis sao as quadruplas (±1, 0,±3,±1) e (±3,±1, ±1, 0).

Problemas Propostos

Problema 15. Encontre todas as solucoes de 999x− 49y = 5000.

Problema 16. Encontre todos os inteiros x e y tais que 147x+ 258y = 369.

Problema 17. Encontre todas as solucoes inteiras de 2x+ 3y + 4z = 5.

Problema 18. Encontre todas as solucoes inteiras do sistema de equacoes:

20x+ 44y + 50z = 10

17x+ 13y + 11z = 19.

Problema 19. (Torneio das Cidades 1997) Sejam a,b inteiros positivos tais que a2 + b2 edivisıvel por ab. Mostre que a = b.

Problema 20. Encontre uma condicao necessaria e suficiente para que

x+ b1y + c1z = d1 e x+ b2y + c2z = d2

tenham pelo menos uma solucao simultanea em inteiros x, y, z, assumindo que os coefici-entes sao inteiros com b1 6= b2.

Problema 21. (AMC 1989) Seja n um inteiro positivo. Se a equacao 2x+2y+n = 28 tem28 solucoes em inteiros positivos x, y e z, determine os possıveis valores de n.

6

Page 59: OBM - Teoría de Números.pdf

Polos Olímpicos de TreinamentoCurso de Teoria dos Números - Nível 2Prof. Samuel Feitosa

Aula 9

O Teorema de Euler

Nesta aula, obteremos uma generalizacao do teorema de Fermat.

Definicao 1. Dado n ∈ N, denotaremos o numero de naturais menores ou iguais a n erelativamente primos com n por φ(n).

Segue imediatamente da definicao de φ(n) que φ(1) = 1, φ(2) = 1, φ(3) = 2, φ(5) = 4 eφ(6) = 2. Se p e primo, φ(p) = p− 1.

Lema 2. Se p e um numero primo e k um numero natural, entao:

φ(pk) = pk−1(p− 1).

Os unicos numeros do conjunto {1, 2, . . . , pk} que nao sao relativamente primos com pk sao

aqueles que sao divisıveis por p. A quantidade de tais numeros epk

p= pk−1. Sendo assim,

φ(pk) = pk − pk−1 = pk−1(p− 1).

Nosso proximo objetivo sera encontrar uma formula para calcular explicitamente φ(m) emfuncao da fatoracao em primos de m. Precisaremos relembrar um exemplo estudado naaula 6:

Lema 3. Sejam m um numero natural, l um numero natural relativamente primo com m

e r um inteiro arbitrario. Entao, o conjunto:

r, l + r, 2l + r, . . . , (m− 1)l + r;

e um sistema completo de restos modulo m.

Suponha, por absurdo, que existem dois inteiros i e j com 0 ≤ i < j < m e para os quaistenhamos r + il ≡ r + jl (mod m). Assim, (j − i)l ≡ 0 (mod m). Como l e relativamenteprimo com m, devemos ter j − i ≡ 0 (mod m). Obtemos um absurdo pois 0 < j − i < m.Consequentemente, temos um conjunto de m inteiros todos incongruentes modulo m e,portanto, tal conjunto e um sistema completo de restos.

Page 60: OBM - Teoría de Números.pdf

POT 2012 - Teoria dos Numeros - Nıvel 2 - Aula 9 - Samuel Feitosa

Teorema 4. Se l e m sao numeros naturais primos entre si, entao:

φ(ml) = φ(m)φ(l).

Demonstracao. Como φ(1) = 1, o teorema anterior e valido quando m = 1 ou n = 1.Suponha entao que m, l > 1. Facamos uma contagem dupla. Primeiramente, usando adefinicao, φ(mn) e o numero de inteiros da tabela abaixo que sao relativamente primoscom ml.

1, 2, . . . , r, . . . , l,1 + l, l + 2, . . . , l + r, . . . , 2l,21 + l, 2l + 2, . . . , 2l + r, . . . , 3l,. . . , . . . , . . . , . . . , . . . , . . . ,(m− 1)1 + l, (m− 1)l + 2, . . . , (m− 1)l + r, . . . , ml,

Seja r ≤ m um numero natural qualquer. Considerando a r-esima coluna da tabela, semdc(r, l) > 1, nenhum de seus elementos e relativamente primo com l. Entao, se buscamosos elementos que nao possuem nenhum fator em comum com ml, devemos nos ater ascolunas com mdc(r, l) = 1. O numero de tais colunas e φ(l). Considerando agora a r-esimacoluna e supondo que mdc(r, l) = 1, em virtude do lema anterior, sabemos que os restos deseus elementos na divisao por m formam exatamente o conjunto {0, 1, . . . ,m} e dentre elesexistem exatamente φ(m) numeros relativamente primos com m. Sendo assim, podemoscontar os numeros relativamente primos com ml atraves do numero de colunas ”boas”e donumero de ”bons”elementos em cada uma delas, obtendo: φ(m)φ(l).

Corolario 5. Se n = pα1

1 pα2

2 . . . pαk

k e a fatoracao em primos de n, entao:

φ(n) = n

(

1− 1

p1

)(

1− 1

p2

)

. . .

(

1− 1

pk

)

De fato, pelo teorema anterior,

φ(n) = φ(pα1

1 pα2

2 . . . pαk

k )

= φ(pα1

1 )φ(pα2

2 ) . . . φ(pαk

k )

= pα1−11 (p1 − 1)pα2−1

2 (p2 − 1) . . . pαk−1k (pk − 1)

= pα1−11 pα2−1

2 . . . pαk−1k (p1 − 1)(p2 − 1) . . . (pk − 1)

= n

(

1− 1

p1

)(

1− 1

p2

)

. . .

(

1− 1

pk

)

Exemplo 6. Mostre que qualquer n ≥ 7 pode ser escrito na forma a+ b, com a e b naturaisprimos entre si, ambos maiores que 1.

Podemos escrever b = n − a e nosso objetivo e encontrar a com 1 < a < n − 1 tal quemdc(a, n− a) = 1. Para isso, basta que mdc(a, n) = 1. Pelo corolario anterior,

φ(n) = pα1−11 pα2−1

2 . . . pαk−1k (p1 − 1)(p2 − 1) . . . (pk − 1)

2

Page 61: OBM - Teoría de Números.pdf

POT 2012 - Teoria dos Numeros - Nıvel 2 - Aula 9 - Samuel Feitosa

Se a expressao anterior e igual a 2, necessariamente devemos ter αi = 1 e pi = 2 ou 3 paratodo i. Sendo assim, n ≤ 6. Logo, φ(n) > 2 e existe pelo menos outro numero naturaldiferente de 1 e n− 1 que e relativamente primo com n.

Exemplo 7. Prove que existem infinitos inteiros positivos n tais que

φ(n) =n

3.

Basta tomar n = 2 · 3m, onde m e um inteiro positivo. Entao:

φ(n) = φ(2 · 3m) = φ(2)φ(3m) = 2 · 3m−1 =n

3.

Exemplo 8. Se n e um inteiro positivo composto, entao

φ(n) ≤ n−√n

Se n = pα1

1 pα2

2 . . . pαk

k , usando que n e composto, podemos garantir que existe um fatorprimo pi tal que pi ≤

√n. Assim,

φ(n) = n

(

1− 1

p1

)(

1− 1

p2

)

. . .

(

1− 1

pk

)

≤ n

(

1− 1

pi

)

≤ n

(

1− 1√n

)

= n−√n

Teorema 9. (Teorema de Euler) Se mdc(a,m) = 1, entao

aφ(m) ≡ 1 (mod m)

Demonstracao. A prova deste teorema sera muito similar a prova do teorema de Fermat.Sejam r1, r2, . . . , rφ(m) os restos em {0, 1, 2, . . . ,m − 1} que sao relativamente primos comm. Considere o conjunto {ar1, ar2, . . . , arφ(m)}. Se dois de seus membros deixam o mesmoresto por m, digamos:

ari ≡ arj (mod m);

temos ri ≡ rj (mod m) pois mdc(a,m) = 1. Claramente isso e uma contradicao. Alemdisso, mdc(ari,m) = mdc(m, ri) = 1. Analisando os restos na divisao por m dos membrosdesse novo conjunto, podemos concluir que tal conjunto coincide com o conjunto dos restosiniciais. Assim,

r1 · r2 · . . . · rφ(m) ≡ ar1 · ar2 · . . . · arφ(m)

≡ aφ(m)r1 · r2 . . . · rφ(m)

Como mdc(r1 · r2 · . . . · rφ(m),m) = 1, podemos cancelar esse termo em ambos os membrosda congruencia anterior obtendo assim o teorema de Euler.

3

Page 62: OBM - Teoría de Números.pdf

POT 2012 - Teoria dos Numeros - Nıvel 2 - Aula 9 - Samuel Feitosa

Exemplo 10. Encontre os ultimos tres dıgitos de 79999

Como φ(1000) = 400, usando o Teorema de Euler, obtemos:

710000 = (7400)25

≡ 1 (mod 1000)

Note que 7 · 143 = 1001 ≡ 1 (mod 1000). Assim,

79999 ≡ 79999 · 7 · 143≡ 710000 · 143≡ 143 (mod 1000)

Logo, 79999 termina em 143.

Exemplo 11. (Putnam 1972) Prove que nao existe um inteiro n > 1 tal que n|2n − 1.

Se existem tais inteiros positivos, denotemos por m o menor deles. Claramente m e ımpar,pelo teorema de Euler, podemos garantir que:

m | 2φ(m) − 1.

Seja d = mdc(m,φ(m)). Pelo problema 27 da aula 3, temos 2d−1 = mdc(2m−1, 2φ(m)−1).Como m | mdc(2m − 1, 2φ(m) − 1), d > 1. Alem disso, d ≤ φ(m) < m e d | 2d − 1. Isso eum absurdo pois m e o menor inteiro maior que 1 com tal propriedade.

Exemplo 12. (Olimpıada de Matematica Argentina) Demostre que para cada numero na-tural n, existe uma potencia de 2 cuja expansao decimal tem entre seus ultimos n dıgitos

(da direita) mais de2n

3dıgitos que sao iguais a 0.

Se 2k tiver um resto muito pequeno modulo 10n, poderemos garantir que existirao muitos ze-ros consecutivos entre seus ultimos dıgitos. Para obtermos a equacao 2k ≡ r (mod 10n) comr pequeno, e interessante comecarmos analisando 2k (mod 5n) uma vez que mdc(2, 5n) = 1.Facamos isso. Pelo teorema de Euler, temos:

2φ(n) ≡ 1 (mod 5n) ⇒2φ(n)+n ≡ 2n (mod 10n).

Como 2n = 8n/3 < 10n/3, podemos concluir que 2n possui menos quen

3dıgitos e, conse-

quentemente, entre os ultimos n dıgitos de 2φ(n)+n existem pelo menos n− n

3=

2n

3dıgitos

consecutivos iguais a zero.

Exemplo 13. (IMO 1971) Prove que a sequencia 2n − 3(n > 1) contem uma subsequenciade numeros primos entre si dois a dois.

4

Page 63: OBM - Teoría de Números.pdf

POT 2012 - Teoria dos Numeros - Nıvel 2 - Aula 9 - Samuel Feitosa

Uma boa estrategia e construir uma sequencia recursivamente. Suponha que ja tenhamosescolhido os termos a1, a2, . . . , ak na sequencia de modo que mdc(ai, aj) = 1. Como pode-remos escolher o proximo termo ak+1 da forma 2n − 3? Claramente mdc(2, ai) = 1. Desdeque φ(ai) | n, poderemos usar o teorema de Euler para obter:

2n − 3 ≡ 1− 3

6≡ 0 (mod ai)

Sendo assim, pelo teorema 4, basta escolhermos:

n = φ(a1 · a2 · . . . · ak) = φ(a1)φ(a2) . . . φ(ak);

que naturalmente sera um multiplo de cada φ(ai). Logo, podemos definir

ak+1 = 2φ(a1·a2·...·ak) − 3

e assim temos uma sequencia de termos infita satisfazendo as condicoes do enunciado.

Problemas Propostos

Problema 14. Encontre todos os numeros naturais n para os quais φ(n) nao e divisıvel por4.

Problema 15. Prove que se p > 2 e 2p+ 1 sao ambos numeros primos, entao para n = 4pvale que

φ(n+ 2) = φ(n) + 2.

Problema 16. Encontre todas as solucoes nos numeros naturais da equacao φ(n) = φ(2n).

Problema 17. Encontre todas as solucoes nos numeros naturais da equacao φ(2n) = φ(3n).

Problema 18. Se n possui k fatores primos distintos, prove que 2k | φ(n).Problema 19. Prove que para qualquer numero natural k, existe pelo menos um numeronatural n tal que

φ(n+ k) = φ(n).

Dica: Considere o menor divisor primo p que nao e um divisor de k e estude o numeron = (p− 1)k.

Problema 20. Mostre que se a e b sao inteiros primos entre si, entao existem inteiros m en tais que am + bn ≡ 1 (mod ab).

Problema 21. (Alemanha) Se n e um numero natural tal que 4n + 2n + 1 e primo, proveque n e potencia de 3.

5

Page 64: OBM - Teoría de Números.pdf

Problema 22. (USAMO 1991) Mostre que para qualquer inteiro fixo n ≥ 1, a sequencia

2, 22, 222

, 2222

, . . . (mod n);

e eventualmente constante, isto e, a partir de um certo termo da sequencia todos os restosobtidos na divisao por n serao iguais.

Dica: Tente considerar os casos em que n e par ou n e ımpar em separado e use inducao.

Problema 23. Encontre os ultimos 8 dıgitos da expansao binaria de 271986

Problema 24. Mostre que, para qualquer inteiro positivo n com n 6= 2 e n 6= 6 temos:

φ(n) ≥√n.

Referencias

[1] F. E. Brochero Martinez, C. G. Moreira, N. C. Saldanha, E. Tengan - Teoria dosNumeros ? um passeio com primos e outros numeros familiares pelo mundo inteiro,Projeto Euclides, IMPA, 2010.

[2] E. Carneiro, O. Campos and F. Paiva, Olimpıadas Cearenses de Matematica 1981-2005(Nıveis Junior e Senior), Ed. Realce, 2005.

[3] S. B. Feitosa, B. Holanda, Y. Lima and C. T. Magalhaes, Treinamento Cone Sul 2008.Fortaleza, Ed. Realce, 2010.

[4] D. Fomin, A. Kirichenko, Leningrad Mathematical Olympiads 1987-1991, MathProPress, Westford, MA, 1994.

[5] D. Fomin, S. Genkin and I. Itenberg, Mathematical Circles, Mathematical Words, Vol.7, American Mathematical Society, Boston, MA, 1966.

[6] I. Niven, H. S. Zuckerman, and H. L. Montgomery, An Introduction to the Theory ofNumbers.

Page 65: OBM - Teoría de Números.pdf

Polos Olímpicos de TreinamentoCurso de Teoria dos Números - Nível 2Prof. Samuel Feitosa

Aula 10

Divisores

Suponha que n = pα1

1 pα2

2 . . . pαk

k e a fatoracao em primos do inteiro n. Todos os divisores

de n sao da forma m = pβ1

1 pβ2

2 . . . pβk

k , onde 0 ≤ βi ≤ αi. Cada um desses numeros, apareceexatamente uma vez no produto:

(1 + p1 + p21 + . . . + pα1

1 )(1 + p2 + p22 + . . . + pα2

2 ) . . . (1 + pn + p2n + . . . + pαk

k ),

quando o mesmo e expandido usando a distributividade. Como existem αi + 1 termos emcada parenteses, O numero de termos dessa expansao e:

(α1 + 1)(α2 + 1) . . . (αk + 1).

Alem disso, sabemos que:

1 + pi + p2i + . . .+ pαi

i =pαi+1i − 1

pi − 1.

Sendo assim, podemos concluir que:

Teorema 1. Se n = pα1

1 pα2

2 . . . pαk

k e a fatoracao em primos de n, entao:

a) O numero de divisores de n, denotado por d(n), e: (α1 + 1)(α2 + 1) . . . (αn + 1).

b) A soma dos divisores de n, denotada por σ(n), e:

(1 + p1 + p21 + . . .+ pα1

1 )(1 + p2 + p22 + . . .+ pα2

2 ) . . . (1 + pn + p2n + . . .+ pαn

n )

ou, de forma mais sucinta,

(

pα1+11 − 1

p1 − 1

)(

pα2+12 − 1

p2 − 1

)

. . .

(

pαn+1n − 1

pn − 1

)

Page 66: OBM - Teoría de Números.pdf

POT 2012 - Teoria dos Numeros - Nıvel 2 - Aula 10 - Samuel Feitosa

Observacao 2. (Pareamento de divisores) Se d e um divisor de n, entaon

dtambem e um

divisor de n.

Portanto, pelo menos um dentre {d, nd} e um divisor de n menor ou igual a

√n.

Exemplo 3. Determine o numero de divisores positivos de 20088 que sao menores que20084.

O numero de divisores de 20088 = 224 · 2518 e 225. Como n e um quadrado perfeito e emvirtude da observacao anterior, 112 desses divisores sao menores que

√20088 = 20084 e 112

sao maiores.

Exemplo 4. Encontre a soma dos inversos dos divisores postivos de 496.

Sejam d1, d2, . . . , dn os divisores de 496 e K a soma de seus inversos. Usando a observacao

anterior, o conjunto {496d1

+496

d2+ . . .+

496

dn} coincide com o conjunto {dn+dn−1+ . . .+d1}

e daı:

1

d1+

1

d2+ . . .+

1

dn= K ⇒

496

d1+

496

d2+ . . .+

496

dn= 496K ⇒

dn + dn−1 + . . .+ d1 = 496K ⇒25 − 1

2− 1· 31

2 − 1

31− 1= 496K ⇒

960

496= K.

Portanto, k =60

31.

Exemplo 5. Um numero natural n possui exatamente dois divisores e n + 1 possui exata-mente 3 divisores. Encontre o numero de divisores de n+ 2.

Se n possui exatamente dois divisores, entao n = p e um numero primo. Se n + 1 possuium numero ımpar de divisores, entao n + 1 = x2 e um quadrado perfeito, para algum x

inteiro positivo. Logo, x2 − 1 = (x− 1)(x+ 1) = p. Como p e primo, a unica possibilidadee x− 1 = 1 e consequentemente n = 3. O numero de divisores de n+ 2 = 5 e 2.

Exemplo 6. Encontre todos os inteiros n que possuem exatamente√n divisores positivos.

Para√n ser inteiro, n deve ser um quadrado perfeito e assim podemos escrever:

n = p2α1

1 p2α2

2 . . . p2αk

k .

A condicao do problema e equivalente a:

pα1

1 pα2

2 . . . pαk

k = (2α1 + 1)(2α2 + 1) . . . (2αk + 1).

2

Page 67: OBM - Teoría de Números.pdf

POT 2012 - Teoria dos Numeros - Nıvel 2 - Aula 10 - Samuel Feitosa

Analisando o lado direito, podemos concluir que cada pi e ımpar e consequentemente

pαi

i ≥ 3αi ≥ 2αi + 1.

Como devemos ter a igualdade, p1 = 3 e 3α1 = 2α1 + 1. Se α1 > 1, vale a desigualdadeestrita(veja o problema 13). Logo, a unica solucao e n = 9.

Exemplo 7. (Suica 2011) Encontre todos os inteiros positivos n para o qual n3 e o produtode todos os divores de n

Claramente n = 1 e solucao. Suponha que n > 1 e sejam d1 < d2 < . . . < dk os divisoresde n. Pela observacao 2, podemos agrupar os divisores em pares cujo produto e n, assim:

n6 = (d1d2 . . . dk)(d1d2 . . . dk)

= (d1dk)(d2dk−1) . . . (dkd1)

= nd(n)

Consequentemente, 6 = d(n) e n = p5 ou n = pq2 com p e q primos distintos. Fica a cargodo leitor verificar que essas solucoes satisfazem o enunciado.

Exemplo 8. (Irlanda 1995) Para cada inteiro positivo n tal que n = p1p2p3p4, onde p1, p2,p3 e p4 sao primos distintos, sejam:

d1 = 1 < d2 < d3 < . . . < d16 = n,

os 16 inteiros positivos que dividem n. Prove que se n < 1995, entao d9 − d8 6= 22.

Suponha que n < 1995 e d9 − d8 = 22. Note inicialmente que d8 nao pode ser par pois n

seria divisıvel por 4 contradizendo o fato de que n possui quatro fatores primos distintos.Consequentemente d8, d9 e n sao ımpares. Tambem temos a fatoracao: 35 · 57 = 1995 =3 · 5 · 7 · 19. Entao, usando a observacao 2, d8d9 = n. Se d8 ≥ 35 terıamos d9 < d8 paramanter n < 1995 e isso seria um absurdo. Logo, d8 < 35. Os divisores d1, d2, . . . , d8 saoprodutos de primos ımpares distintos. Como 3 · 5 · 7 > 35, nenhum dentre d1, d2, . . . , d8e grande o suficiente para possuir tres fatores primos distintos. Como n possui somentequatro fatores primos distintos, quatro desses di’s devem ser o produto de dois primosımpares. Os menores numeros que sao o produto de dois primos sao:

15, 21, 33, 35, . . .

e consequentemente devemos ter d8 ≥ 35, uma contradicao.

Exemplo 9. Prove que nao existe inteiro positivo n tal que σ(n) = nk para algum inteiropositivo k.

Afirmamos que n = 1 e a unica solucao. Suponha que n > 1 seja solucao e sejam

d1 = 1 < d2 < . . . < dk = n,

3

Page 68: OBM - Teoría de Números.pdf

POT 2012 - Teoria dos Numeros - Nıvel 2 - Aula 10 - Samuel Feitosa

os divisores de n. Entao

σ(n) = d1 + d2 + . . .+ dk < 1 + 2 + . . .+ n =n(n+ 1)

2< n2.

Alem disso,n < n+ 1 ≤ d1 + d2 + . . .+ dk = σ(n).

Daı,n < nk < n2,

e obtemos um absurdo.

Exemplo 10. (Olimpıada de Leningrado 1989) Duas pessoas jogam um jogo. O numero 2esta inicialmente escrito no quadro. Cada jogador, na sua vez, muda o numero atual Nno quadro negro pelo numero N + d, onde d e um divisor de N com d < N . O jogador queescrever um numero maior que 19891988 perde o jogo. Qual deles ira vencer se ambos osjogadores sao perfeitos.

Nesse problema, basta determinarmos apenas aquele que possui a estrategia vencedora.Note que o inıcio do jogo e estritamente determinado: 2 → 3 → 4. Suponha que o segundojogador vence o jogo. Apos o movimento 4 → 5 do primeiro jogador, o segundo so podejogar 5 → 6. Isto significa que 6 e uma posicao vencedora. Entretanto, o primeiro jogadorpode obter a posicao 6 jogando 4 → 6, uma contradicao. Logo, o primeiro jogador possuia estrategia vencendora.

Exemplo 11. (Olimpıada de Leningrado) Duas pilhas de palitos sobre uma mesa contem100 e 252 palitos, respectivamente. Dois jogadores jogam o seguinte jogo: Cada jogador emsua vez pode remover alguns palitos de uma das pilhas de modo que o numero de palitosretirados seja um divisor do numero de palitos da outra pilha. O jogador que fizer o ultimomovimento vence. Qual dos dois jogadores ira vencer se ambos sao perfeitos?

O primeiro jogador perde. Em cada momento do jogo, podemos registrar o expoenteda maior potencia de 2 que divide os numeros de palitos em cada pilha. Por exemplo,no inıcio os numeros sao (2, 2). A estrategia do segundo jogador e manter esse numerossempre iguais. Suponha que, em um dado momento, as pilhas possuem 2m · a e 2m · bpalitos com a e b ımpares. O par registrado sera (m,m). Vejamos o que acontece quandoretiramos um divisor d da segunda pilha do numero de palitos da primeira. Se 2m e amaior potencia de 2 que divide d, entao 2m+1 dividira o numero de palitos da primeirapilha e consequentemente o par registrado tera numeros diferentes. Se 2k, com k < m,e a maior potencia de 2 que divide d, entao 2k sera a maior potencia de 2 que divide onumero de palitos da primeira pilha e novamente o par registrado tera numeros diferentes.Assim, sempre que um jogador receber um par registrado com numeros iguais, ele ira passarum par registrado com numeros diferentes para o outro jogador. Suponha agora que, nasua vez, as pilhas possuem 2m · a e 2n · b palitos, com m < n e a ≡ b ≡ 1 (mod 2).Basta o jogador retirar 2m palitos da segunda pilha para passar um par registrado comnumeros iguais a (m,m). Como inicialmente as pilhas possuem numeros registrados iguais,o segundo jogador pode sempre manter essa propriedade e consequentemente o unico quepode passar uma pilha com zero palitos pela primeira vez e o primeiro jogador.

4

Page 69: OBM - Teoría de Números.pdf

POT 2012 - Teoria dos Numeros - Nıvel 2 - Aula 10 - Samuel FeitosaREFERENCIAS

Problemas Propostos

Problema 12. Mostre que se k e um inteiro positivo entao 3k ≥ 2k+1 e vale a desigualdadeestrita quando k > 1.

Problema 13. (Russia 2001) Encontre todos os n tais que quaisquer divisores primos dis-tintos a e b de n o numero a+ b− 1 tambem e um divisor de n

Problema 14. O numero 332 − 1 tem exatamente dois divisores que sao maiores que 75 emenores que 85. Qual o produto desses dois divisores?

Problema 15. (Ira 2012) Sejam a e b inteiros positivos de modo que o numero de divisorespositivos de a,b, ab e 3,4 e 8, respectivamente. Encontre o numero de divisores positivosde b2.

Problema 16. (Olimpıada de Sao Petesburgo) Enconte todos os inteiros positivos n taisque 3n−1 + 5n−1 divide 3n + 5n.

Problema 17. Sejam 1 = d1 < d2 < .... < dk = n o conjunto de todos os divisores de uminteiro positivo n. Determine todos os n tais que:

d26 + d27 − 1 = n.

Problema 18. Um divisor d > 0 de um inteiro positivo n e dito ser um divisor unitario

se mdc(d,n

d) = 1. Suponha que n e um inteiro positivo tal que a soma de seus divisores

unitarios e 2n. Prove que n nao pode ser ımpar.

Referencias

[1] F. E. Brochero Martinez, C. G. Moreira, N. C. Saldanha, E. Tengan - Teoria dosNumeros ? um passeio com primos e outros numeros familiares pelo mundo inteiro,Projeto Euclides, IMPA, 2010.

[2] E. Carneiro, O. Campos and F. Paiva, Olimpıadas Cearenses de Matematica 1981-2005(Nıveis Junior e Senior), Ed. Realce, 2005.

[3] S. B. Feitosa, B. Holanda, Y. Lima and C. T. Magalhaes, Treinamento Cone Sul 2008.Fortaleza, Ed. Realce, 2010.

[4] D. Fomin, A. Kirichenko, Leningrad Mathematical Olympiads 1987-1991, MathProPress, Westford, MA, 1994.

[5] D. Fomin, S. Genkin and I. Itenberg, Mathematical Circles, Mathematical Words, Vol.7, American Mathematical Society, Boston, MA, 1966.

[6] I. Niven, H. S. Zuckerman, and H. L. Montgomery, An Introduction to the Theory ofNumbers.

5

Page 70: OBM - Teoría de Números.pdf

Polos Olímpicos de TreinamentoCurso de Teoria dos Números - Nível 2Prof. Samuel Feitosa

Aula 11

O Teorema Chines dos Restos

Iremos estudar um antigo teorema descoberto pelos chineses no inıcio seculo XIII. Come-cemos recordando um lema da aula 06:

Lema 1. Se mdc(a,m) = 1, entao existe um inteiro x tal que:

ax ≡ 1 (mod m).

Tal inteiro e unico modulo m. Se mdc(a,m) > 1, nao existe x satisfazendo tal equacao.

Demonstracao. Pelo teorema de Bachet-Bezout, existem inteiros x e y tais que ax+my = 1.Analisando essa congruencia modulo m, obtemos ax ≡ 1 (mod m). Se y e outro inteiroque satisfaz a mesma congruencia, temos ax ≡ ay (mod m). Pelo primeiro lema, x ≡ y

(mod m). Se d = mdc(a,m) > 1, nao podemos ter d | m e m | ax− 1 pois d ∤ ax− 1.

Exemplo 2. Encontre x inteiro tal que:

x ≡ 1 (mod 11);

x ≡ 2 (mod 7).

A primeira congruencia nos diz que x = 11k+1 para algum k ∈ Z. Sejam q e r o quocientee o resto da divisao de k por 7, respectivamente. Assim, k = 7q + r e x = 77q + 11r + 1.Para x satisfazer a segunda congruencia, devemos encontrar r ∈ {0, 1, 2, 3, 4, 5, 6} tal que11r + 1 ≡ 2 (mod 7), ou seja, 4r ≡ 1 (mod 7). Como o inverso de 4 (mod 7) e 2, obtemosr = 2 e x = 77q + 23. Veja que para qualquer q inteiro, tal x e solucao do sistema decongruencias.

Exemplo 3. Encontre x inteiro tal que:

x ≡ 1 (mod 11)

x ≡ 2 (mod 7)

x ≡ 4 (mod 5)

Page 71: OBM - Teoría de Números.pdf

POT 2012 - Teoria dos Numeros - Nıvel 2 - Aula 11 - Samuel Feitosa

Pelo exemplo anterior, para x satisfazer as duas primeiras equacoes, devemos terx = 77q+23. Dividindo q por 5, obtemos q = 5l+s com 0 ≤ s < 5. Daı, x = 385l+77s+23.Para satisfazer a ultima congruencia, devemos ter 77s + 23 ≡ 4 (mod 5), ou seja, 2s ≡ 1(mod 5). Como 3 e o inverso de 2 (mod 5), s = 3 e consequentemente x = 385l + 254.

Perceba que nos dois exemplos anteriores, o problema foi reduzido a encontrarmos o inversode um inteiro. No ultimo exemplo, a solucao geral possui a forma: x = 11·7·5l+231+22+1.Essencialmente, o trabalho de encontrar esses inversos foi possıvel pois os inteiros 5, 7 e 11sao primos entre si dois a dois.

Veremos agora um mecanismo levemente diferente para resolver tais sistemas equacoes.

Teorema 4. (Teorema Chines dos Restos) Sejam m1,m2, . . . ,mr, inteiros positivosprimos entre si, dois a dois, e sejam a1, a2, . . . , ar; r inteiros quaisquer. Entao, o sistemade conguencias:

x ≡ a1 (mod m1)

x ≡ a2 (mod m2)

...

x ≡ ar (mod mr)

admite uma solucao x. Alem disso, as solucoes sao unicas modulo m = m1m2 . . . mr.

Demonstracao. Escrevendo m = m1m2 . . . mr, vemos quem

mj

e um inteiro e

mdc

(

m

mj

,mj

)

= 1. Entao, pelo lema inicial, para cada j, existe um inteiro bj tal que(

m

mj

)

bj ≡ 1 (mod mj). Claramente

(

m

mj

)

bj ≡ 0 (mod mi) para i 6= j. Definamos

x0 =m

m1

b1a1 +m

m2

b2a2 + . . .+m

mr

brar

Consideremos x0 modulo mi: x0 ≡m

mi

bjaj ≡ ai (mod mi). Logo, x0 e uma solucao do

nosso sistema. Se x0 e x1 tambem o sao, podemos escrever x0 ≡ x1 (mod mi) para cada i.Como mdc(mi,mj) = 1, para i 6= j, pbtemos x0 ≡ x1 (mod m).

Observacao 5. Se cada uma das equacoes do sistema anterior fosse do tipobix ≡ ai (mod m)i, com mdc(bi,m) = 1, ainda poderıamos usa-lo. Bastaria reescreverbix ≡ ai (mod m)i como x ≡ biai (mod m)i, onde bi e o inverso de bi (mod m)i.

Exemplo 6. Encontre o menor inteiro positivo x tal que x ≡ 5 (mod 7), x ≡ 7 (mod 11) ex ≡ 3 (mod 13).

Usando o teorema anterior com m1 = 5,m2 = 7,m3 = 11, a1 = 5, a2 = 7 e a3 = 3 podemosachar x ≡ 887 (mod 1001) = 7.11.13. Como a solucao e unica modulo m, isso significaque, dentre os numeros 1, 2, · · · , 1001 a menor solucao positiva e 887.

2

Page 72: OBM - Teoría de Números.pdf

POT 2012 - Teoria dos Numeros - Nıvel 2 - Aula 11 - Samuel Feitosa

Exemplo 7. (OBM 2009) Sejam m e n dois inteiros positivos primos entre si. O TeoremaChines dos Restos afirma que, dados inteiros i e j com 0 ≤ i < m e o ≤ j < n, existeexatamente um inteiro a, com 0 ≤ a < mn, tal que o resto da divisao de a por m e iguala i e o resto da divisao de a por n e igual a j. Por exemplo, para m = 3 e n = 7, temosque 19 e o unico numero que deixa restos 1 e 5 quando dividido por 3 e 7, respectivamente.Assim, na tabela a seguir, cada numero de 0 a 20 aparecera exatamente uma vez.

0 1 2 3 4 5 6

0 A B

1 C D

2 E F

Qual a soma dos numeros das casas com as letras A,B,C,D,E e F?

Usando o teorema chines dos restos, podemos encontrar A = 15, B = 12, C = 10,D =13, E = 8 e F = 11. Assim, A+B +C +D + E + F = 69.

Exemplo 8. (Estonia 2000) Determine todos os restos possıveis da divisao do quadrado deum numero primo com 120 por 120.

Seja n tal que mdc(n, 120) = 1. Como 120 = 3 · 5 · 8, temos que n 6≡ 0 (mod 3), (mod 5)(mod 2). Daı, n2 ≡ 1 (mod 3), n2 ≡ 1 (mod 8) e n2 ≡ 1 ou 4 (mod 5). Sendo assim, n2

satisfaz o sistema:

x ≡ 1 (mod 3)

x ≡ 1 (mod 8)

x ≡ ±1 (mod 5)

cujas solucoes sao x ≡ 1 (mod 120) e x ≡ 49 (mod 120).

Aconselhamos ao leitor a resolucao de alguns exemplos numericos ate adquirir pratica com oalgoritmo usado para encontrar x0. Provamos, no teorema passado, que todas as solucoesdaquele sistema de congruencias sao os termos de uma P.A de razao m. Geralmenteusaremos aquele teorema apenas para garantir que um sistema de congruencias admiteuma solucao. Os proximos exemplos podem deixar isso mais claro.

Exemplo 9. Para cada numero natural n, existe uma sequencia arbitrariamente longa denumeros natu rais consecutivos, cada um deles sendo divisıvel por uma s-esima potenciade um numero natural maior que 1.

Demonstracao. Dado m ∈ N, considere o conjunto {p1, p2, . . . , pm} de primos distintos.Como mdc(psi , p

sj) = 1, entao pelo teorema 3, existe x tal que x ≡ −i (mod psi ) para

i = 1, 2, . . . m. Cada um dos numeros do conjunto {x+ 1, x+ 2, . . . , x+m} e divisıvel porum numero da forma psi .

3

Page 73: OBM - Teoría de Números.pdf

POT 2012 - Teoria dos Numeros - Nıvel 2 - Aula 11 - Samuel Feitosa

Exemplo 10. (USAMO 1986)

(a) Existem 14 inteiros positivos consecutivos tais que, cada um e divisıvel por um oumais primos p do intervalo 2 ≤ p ≤ 11?

(b) Existem 21 inteiros positivos consecutivos tais que, cada um e divisıvel por um oumais primos p do intervalo 2 ≤ p ≤ 13?

Demonstracao. (a) Nao. Suponha que existam tais inteiros. Da nossa lista de 14 inteirosconsecutivos, 7 sao numeros pares. Vamos observar os ımpares: a, a + 2, a + 4, a + 6, a +8, a+ 10 e a+ 12. Podemos ter no maximo tres deles divisıveis por 3, dois por 5, um por7 e um por 11. Veja que 3 + 2 + 1 + 1 = 7. Pelo Princıpio da Casa dos Pombos, cada umdesses ımpares e divisıvel por exatamente um primo do conjunto {3, 5, 7, 11}. Alem disso,note que os multiplos de 3 so podem ser {a, a+ 6, a+12}. Dois dos numeros restantes em(a+ 2, a + 4, a + 8, e a + 10) sao divisıveis por 5. Mas isso e impossıvel. (b) Sim. Comoos numeros {210, 11, 13} sao primos entre si, dois a dois, pelo teorema 3 existe um inteiropositivo n > 10 tal que:

n ≡ 0(mod 210 = 2 · 3 · 5 · 7·)n ≡ 1(mod 11)n ≡ −1(mod 13)

Veja que o conjunto {n− 10, n − 9, . . . , n+ 9, n + 10} satisfaz as condicoes do item (b).

Exemplo 11. Sejam a e b inteiros positivos tais que, para qualquer n natural, an+n | bn+n.Prove que a = b.

Seja p um primo maior que a e b. Entaomdc(p, a) = mdc(p, b) = 1. Comomdc(p, p−1) = 1,existe um inteiro positivo n tal que n ≡ 1 (mod p−1) e n ≡ −a (mod p). Pelo teorema deFermat, an + n ≡ 0 (mod ) e bn +n ≡ b− a (mod p). Assim, p | |b− a|. Como |b− a| < p,segue que |b− a| = 0 e a = b.

Exemplo 12. (Olimpıada Nordica 1998)

(a) Para quais inteiros positivos n existe um sequencia x1, x2, . . . , xn contendo cada umdos inteiros 1, 2, . . . , n exatamente uma vez, e tal que k divide x1+x2+ · · ·+xk parak = 1, 2, · · · , n?

(b) Existe uma sequencia infinita x1, x2, . . . contendo todo inteiro positivo exatamenteuma vez, e tal que para cada inteiro positivo k, k divide x1 + x2 + · · ·+ xk?

a) Suponha que n e um inteiro que satisfaz o enunciado. Naturalmente n divide a soma:

x1 + x2 + . . . xn =n(n+ 1)

2.

Daı,n+ 1

2e um inteiro e n deve ser ımpar. Seja m =

n+ 1

2. Usando que

(n− 1) | x1 + x2 + . . . xn−1 = mn− xn,

4

Page 74: OBM - Teoría de Números.pdf

POT 2012 - Teoria dos Numeros - Nıvel 2 - Aula 11 - Samuel Feitosa

temos xn ≡ m (mod n − 1) se n ≥ 3 e, consequentemente, xn = m. Repetindo a mesmaanalise para n − 2 no lugar de n − 1, obtemos xn−1 = m para n ≥ 5. Como nao podemexistir dois termos iguais, temos um absurdo. Analisando os casos quando n ≤ 4, encon-tramos n = 1 e n = 3 como unicas solucoes.

b) Iremos construir a sequencia indutivamente. Suponha que ja tenhamos definido os termosx1, x2, . . . , xn satisfazendo a condicao k | x1+x2 . . . xk para todo k ≤ n. Seja m o menor in-teiro positivo que ainda nao apareceu na sequencia. Pelo Teorema Chines dos Restos, existex tal que x ≡ −(x1+x2+ . . .+xn) (mod n+1) e x ≡ −(x1+x2+ . . .+xn)−m (mod n+2).Escolha l, inteiro positivo, tal que l > x1, x2, . . . , xn,m e l ≡ x (mod (n+1)(n+2)). Definaxn+1 = l e xn+2 = m. Veja que a condicao k | x1 + x2 . . . xk agora e verdadeira para todok ≤ n+ 2. Para o inıcio, basta definir x1 = 1.

Exemplo 13. (Olimpıada de Sao Petesburgo 1990) Dado um polinomio F (x) com coefi-cientes inteiros, tal que, para cada inteiro n, o valor de F (n) e divisıvel por pelo menosum dos inteiros a1, a2, · · · , am. Prove que podemos encontrar um ındice k tal que F (n) edivisıvel por ak para cada inteiro positivo n.

Demonstracao. Suponha que nao exista tal ındice. Para cada ındice k (k = 1, 2, . . . ,m),existe um inteiro xk tal que F (xk) nao e divisıvel por ak. Assim, existem numerosdk = p

αk

k (onde pk sao numeros primos), tais que dk divide ak mas nao divide F (xk). Seexistem potencias do mesmo primo entre esses numeros, podemos apagar aquelas repeti-das deixando apenas uma que tem expoente mınimo. Caso F (x) nao seja divisıvel poruma potencia apagada, nao sera pela potencia que tem expoente mınimo. Essas delecoesgaratem que nossa nova colecao d1, d2, . . . , dj de potencias de primos contenham apenasinteiros primos entre si, dois a dois. Pelo teorema chines dos restos, exite um inteiro N

tal que N ≡ xk (mod d)k, para k ∈ {1, 2, . . . , j}. Suponhamos que dk | F (N). Sabemosque x− y | F (x) − F (y) e consequentemente N − xk | F (N) − F (xk). Como dk | N − xk,devemos ter dk | F (xk). Uma contradicao! Logo, F (N) nao e divisıvel por nenhum dk eisso contradiz a hipotese sobre os ai.

Problemas Propostos

Problema 14. Encontre o menor inteiro positivo (com a excecao de x = 1) que satisfaca oseguinte sistema de congruencias:

x ≡ 1 (mod 3)

x ≡ 1 (mod 5)

x ≡ 1 (mod 7)

5

Page 75: OBM - Teoría de Números.pdf

POT 2012 - Teoria dos Numeros - Nıvel 2 - Aula 11 - Samuel Feitosa

Problema 15. Encontre todas as solucoes do sistema:

x ≡ 2 (mod 3)

x ≡ 3 (mod 5)

x ≡ 5 (mod 2)

Problema 16. Encontre todos os inteiros que deixam restos 1, 2 e 3 quando divididos por3, 4 e 5, respectivamente.

Problema 17. Encontre todas as solucoes do sistema:

3x ≡ 1 (mod 4)

2x ≡ 1 (mod 3)

4x ≡ 5 (mod 7)

Problema 18. Encontre todas as solucoes das congruencias:

a) 20x ≡ 4 (mod 30).

b) 20x ≡ 30 (mod 4).

c) 353x ≡ 254 (mod 400).

Problema 19. Se a e escolhido ao acaso no conjunto {1, 2, 3, . . . , 14} e b e escolhido aoacaso no conjunto {1, 2, . . . , 15}, qual a probabilidade de que a equacao ax ≡ b (mod 15)possua pelo menos uma solucao?

Problema 20. Sejam a e b inteiros tais que mdc(a, b) = 1 e c > 0. Prove que existe uminteiro x tal que mdc(a+ bx, c) = 1.

Problema 21. Existem n inteiros consecutivos tal que cada um contem um fator primorepetido k vezes?

Problema 22. Seja n um numero natural arbitrario. Prove que existe um par de naturais(a, b) tais que mdc(a+ r, b+ s) > 1 ∀ r, s = 1, 2, . . . , n.

Problema 23. Um ponto (x, y) ∈ Z2 e legal se mdc(x, y) = 1. Prove ou disprove: Dadoum inteiro positivo n, existe um ponto (a, b) ∈ Z2 cuja distancia a todo ponto legal e pelomenos n?

Problema 24. Sejam mo,m1, ...,mr inteiros positivos que sao primos entre si, dois a dois.Mostre que existem r+1 inteiros consecutivos s, s+1, ..., s+r tal que mi divide s+i para i =0, 1, ..., r.

Problema 25. (Romenia 1995) Seja f : N−{0, 1} → N definida por f(n) = mmc[1, 2, ..., n].Prove que para todo n ≥ 2, existem n numeros consecutivos para os quais f e constante.

6

Page 76: OBM - Teoría de Números.pdf

Problema 26. (OBM 2005) Dados os inteiros positivos a, c e o inteiro b, prove que existeum inteiro positivo x tal que ax + x ≡ b (mod c).

Problema 27. (Cone Sul 2003) Demonstrar que existe uma sequencia de inteiros positivosx1, x2, . . . que satisfaz as duas condicoes seguintes:

(a) contem exatamente uma vez cada um dos inteiros positivos,

(b) a soma parcial x1 + x2 + . . . xn e divisıvel por nn.

Problema 28. (Republica Tcheca e Eslovaca 1997) Mostre que existe uma sequencia cres-cente {an}

n=1 de numeros naturais tais que para k ≥ 0 , a sequencia {an + k} contem umnumero finito de primos.

Problema 29. Considere o inteiro c ≥ 1 e a sequencia definida por a1 = c e ai+1 = cai .Mostre que esta sequencia se torna eventualmente constante quando a reduzimos modulo n

para algum inteiro positivo n (isto significa que am ≡ aj (mod n) se m ≥ j).

Problema 30. (Coreia 1999) Encontre todos os inteiros n tais que 2n− 1 e um multiplo de

3 e2n − 1

3e um divisor de 4m2 + 1 para algum inteiro m.

Problema 31. (OBM 2006) Prove que, para todo inteiro n ≤ 2, o numero de matrizesquadradas 2 × 2 com entradas inteiras e pertencentes ao conjunto {0, 1, 2, . . . , n − 1} quetem determinante da forma kn+ 1 para algum k inteiro e dado por

p primo

p | n

(

1−1

p2

)

.

Problema 32. Encontre todos os subconjuntos S ⊂ Z+ tais que todas as somas de umaquantidade finita de elementos de S(com possıveis repeticoes de elementos) sao numeroscompostos.

Problema 33. Existe algum natural n para o qual existem n − 1 progressoes aritmeticascom razoes 2, 3, . . . , n tais que qualquer natural esta em pelo menos uma das progressoes?

Problema 34. Seja P (X) um polinomio com coeficientes inteiros e k e um inteior qualquer.Prove que existe um inteiro m tal que P (m) tem pelo menos k fatores primos distintos.

Acompanhe as dicussoes dos problemas propostos no forum do POTI:www.poti.impa.br/forum/

Page 77: OBM - Teoría de Números.pdf

Referencias

[1] F. E. Brochero Martinez, C. G. Moreira, N. C. Saldanha, E. Tengan - Teoria dosNumeros ? um passeio com primos e outros numeros familiares pelo mundo inteiro,Projeto Euclides, IMPA, 2010.

[2] E. Carneiro, O. Campos and F. Paiva, Olimpıadas Cearenses de Matematica 1981-2005(Nıveis Junior e Senior), Ed. Realce, 2005.

[3] S. B. Feitosa, B. Holanda, Y. Lima and C. T. Magalhaes, Treinamento Cone Sul 2008.Fortaleza, Ed. Realce, 2010.

[4] D. Fomin, A. Kirichenko, Leningrad Mathematical Olympiads 1987-1991, MathProPress, Westford, MA, 1994.

[5] D. Fomin, S. Genkin and I. Itenberg, Mathematical Circles, Mathematical Words, Vol.7, American Mathematical Society, Boston, MA, 1966.

[6] I. Niven, H. S. Zuckerman, and H. L. Montgomery, An Introduction to the Theory ofNumbers.

Page 78: OBM - Teoría de Números.pdf

Polos Olímpicos de TreinamentoCurso de Teoria dos Números - Nível 2Prof. Samuel Feitosa

Aula 12

Equacoes Diofantinas II

Continuaremos nosso estudo das equacoes diofantinas abordando agora algumas equacoesquadraticas. Comecaremos peloo classico problema das ternas pitagoricas.

Desejamos encontrar todas as solucoes (x, y, z) da equacao:

x2 + y2 = z2,

em inteiros positivos. Seja d = mdc(x, y). Como d2 | z2, segue que d | z e que(x

d,y

d,z

d

)

tambem e solucao. Alem disso, podemos concluir que:

mdc(x/d, y/d) = mdc(x/d, z/d) = mdc(y/d, z/d) = 1.

Uma terna que e solucao e possui a propriedade de que quaisquer dois de seus termossao primos entre si, sera chamada de solucao primitiva. Assim, toda solucao (x, y, z) e daforma (dx1, dy1, dz1) onde (x1, y1, z1) e uma solucao primitiva. Para cumprimirmos nossoobjetivo, bastara nos concentrarmos em encontrar todas as solucoes primitivas. Analisandoa equacao modulo 4 e lembrando que todo quadrado perfeito pode deixar apenas os restos 0ou 1, concluımos que exatamente um dentre x e y e par. Suponha sem perda de generalidadeque y seja par. Fatorando a equacao, obtemos:

z + x

2·z − x

2=

(y

2

)2

Como mdc((z+x)/2, (z−x)/2) = 1, concluımos que (z+x)/2 e (z−x)/2 devem ser ambosquadrados perfeitos, i.e., existem inteiros positivos r e s, com r > s emdc(r, s) = 1, tais que(z + x)/2 = r2 e (z − x)/2 = s2 (veja o primeiro problema proposto). Consequentemente,x = r2 − s2, y = 2rs e z = r2 + s2. Reciprocamente, se (x, y, z) = (r2 − s2, 2rs, r2 + s2),temos:

x2 + y2 = (r2 − s2)2 + (2rs)2 = (r2 + s2)2 = z2.

O proximo teorema resume nossa discussao original:

Page 79: OBM - Teoría de Números.pdf

POT 2012 - Teoria dos Numeros - Nıvel 2 - Aula 12 - Samuel Feitosa

Teorema 1. Todas as solucoes primitivas de x2+y2 = z2 com y par sao da forma x = r2−s2,y = 2rs e z = r2 + s2, onde r e s sao inteiros de paridade oposta com r > s > 0 emdc(r, s) = 1.

Exemplo 2. Encontre todas as ternas pitagoricas (a, b, c) tais que a+ b+ c = 1000.

Seja k = mdc(a, b, c) e suponha sem perda de generalidade que b/k e par. Pelo teoremaanterior, (a, b, c) = (k(x2−y2), k(2xy), k(x2+y2)), onde x > y, mdc(x, y) = 1 e pelo menosum dentre x e y par. Assim, (x2 − y2) + 2xy+ (x2 + y2) = 2x(x+ y) e um divisor de 1000.Com mais razao, x(x+ y) | 500. Usando que mdc(x, x+ y) = 1 e a fatoracao em primos de500, podemos concluir que um deles e uma potencia de 5 e o outro uma potencia de 2. Vejaque x nao pode ser uma potencia de 5 pois nesse caso y deveria ser ımpar para garantirque x+ y seja uma potencia de 2. Assim, x | 500 e x = 2k, produzindo como possibilidadesx = 1, 2 ou 4. Analisando cada um desses casos e levando em conta que y < x, e facilencontrar que x = 4 e y = 1 sao as unicas opcoes possıveis. Nesse caso, x = 15, y = 8 ez = 17. Consequentemente, (a, b, c) = (20 · 15, 20 · 8, 20 · 17).

Exemplo 3. Mostre que se a, b e c sao inteiros positivos tais que a2 + b2 = c2, entao(ab)4 + (bc)4 + (ca)4 e um quadrado perfeito.

Veja que:

(ab)4 + (bc)4 + (ca)4 = (a2b2 + b4)2 + (a2b2)2 + (a2b2 + a4)2 = (a4 + a2b2 + b4)2.

Exemplo 4. Encontre todas as solucoes de x2 + 2y2 = z2 em inteiros positivos commdc(x, y, z) = 1.

Como 2y2 ≡ 0 (mod 2), devemos ter x ≡ z (mod 2). Alem disso, se fosse x ≡ z ≡ 0(mod 2) terıamos 4 | z2 − x2 = 2y2 e consequentemente 2 | y, contradizendo a hipostesemdc(x, y, z) = 1. Fatorando a expressao, temos:

2y2 = (z − x)(z + x).

Como mdc(x, z) = 1 e ambos sao ımpares; mdc(z − x, z + x) = 2 e apenas um deles econgruo a 2 (mod 4). Temos dois casos a considerar: 1) z + x ≡ 0 (mod 4) e z − x ≡ 2(mod 4). Nesse caso, y2 = (z − x)/2 · (z + x) com mdc((z − x)/2, (z + x)) = 1. Daı,existem inteiros positivos r e s tais que (z − x)/2 = r2 e (z + x)/2 = s2, produzindo asolucao (x, y, z) = ((s2 − 2r2)/2, rs, (2r2 + s2)/2) com mdc(r, s) = 1 e s ≡ 0 (mod 2). Umraciocınio analogo para o caso z + x ≡ 2 (mod 4) e z − x ≡ 0 (mod 4) produz (x, y, z) =((2s2 − r2)/2, rs, (r2 + 2s2)/2) com mdc(r, s) = 1 e r ≡ 0 (mod 2).

Problema 5. (USAMO 1976) Encontre todas as solucoes naturais da equacao

a2 + b2 + c2 = a2b2.

A equacao pode ser reescrita como:

c2 = (a2 − 1)(b2 − 1)− 1.

2

Page 80: OBM - Teoría de Números.pdf

POT 2012 - Teoria dos Numeros - Nıvel 2 - Aula 12 - Samuel Feitosa

Se pelo menos um dentre a ou b e ımpar, teremos c2 ≡ 3 (mod 4). Como os quadradosperfeitos so podem deixar resto 0 ou 1 (mod 4), temos um absurdo. Portanto, a, b econsequentemente c sao numeros pares. Seja k o maior inteiro tal que 2k divida esses tresnumeros. Assim, a = 2x, b = 2ky, c = 2kz onde pelo menos um dentre x, y e z ımpar.Assim,

x2 + y2 + z2 = 22rx2y2.

Como r > 0, x2 + y2 + z2 ≡ 0 (mod 4). Entretanto, isso nao e possıvel se um dentre osx, y, z e ımpar pois a soma so poderia ser congruente a 1, 2, 3 (mod 4).

Exemplo 6. (Extraıdo de [1]) Determine todas as ternas (a, b, c) de inteiros positivos taisque a2 = 2b + c4.

Como a2 = 2b + c4 ⇐⇒ (a − c2)(a + c2) = 2b, pelo Teorema Fundamental da Aritmeticaexistem dois naturais m > n tais que m+ n = b, a− c2 = 2n e a+ c2 = 2m. Subtraindo asduas ultimas equacoes, obtemos que 2c2 = 2m−2n, assim c2 = 2n−1(2m−n−1). Como 2n−1

e 2m−n − 1 sao primos entre si e o seu produto e um quadrado perfeito (i.e. os expoentesdas potencias de primos distintos sao pares), novamente pelo Teorema Fundamental daAritmetica 2n−1 e 2m−n − 1 devem ser ambos quadrados perfeitos, logo n − 1 e par e2m−n−1 = (2k−1)2 para algum inteiro positivo k. Como 2m−n = (2k−1)2+1 = 4k(k−1)+2e divisıvel por 2 mas nao por 4, temos m− n = 1. Assim, fazendo n − 1 = 2t, temos quetodas as solucoes sao da forma (a, b, c) = (3 · 22t, 4t+3, 2t) com t ∈ N e e facil verificar quetodos os numeros desta forma sao solucoes.O proximo exemplo ilustrara o metodo da descida de Fermat que faz uso do princıpioda boa ordenacao: todo subconjunto nao vazio de inteiros positivos possui um elementomınimo.

Exemplo 7. Determine todas as solucoes da equacao x4 + y4 = z2 em inteiros positivoscom mdc(x, y) = 1.

Como (x2)2+(y2)2 = z2 e mdc(x2, y2) = 1, podemos usar o primeiro teorema para concluirque existem u e v tais que x2 = u2 − v2, y2 = 2uv, z = u2 + v2, u > v > 0 e mdc(u, v) = 1(Estamos assumindo sem perda de generalidade que x e ımpar). Se u e par, entao v seraımpar e teremos x2 ≡ 3 (mod 4). Como isso e um absurdo, u deve ser ımpar e v deve serpar. Sendo assim, (y/2)2 = u · v/2 com mdc(u, v/2) = 1. Devemos ter u = r2, v/2 = s2,com mdc(r, s) = 1, r, s > 0, r ımpar e y = 2rs. Alem disso, como x2 + v2 = u2, obtemosx2 +4s2 = r4. Como mdc(r, 2s) = 1, novamente pelo primeiro teorema, existem m e n taisque x = m2 − n2, 2s2 = 2mn e r2 = m2 + n2 com mdc(m,n) = 1 e m > n > 0. Comomn = s2, podemos escrever m = f2 e n = g2 com f, g > 0 e mdc(f, g) = 1. Portanto,r2 = f4 + g4. Note que dada a solucao em inteiros positivos (x, y, z), obtivemos outrasolucao (f, g, r), tambem nos inteiros positivos, com 0 < r < z. Isso nos diz que existeuma infinidade decrescente de possıveis valores para o inteiro positivo z e naturalmenteobtemos uma contradicao do princıpio da boa ordenacao. Sendo assim, a equacao anteriornao possui solucao nos inteiros positivos.

Observacao 8. Outra maneira de formalizar o argumento anterior e escolher dentre asternas nos inteiros positivos que sao solucoes, aquela com z mınimo. A nova terna (f, g, r)caracterizaria um absurdo.

3

Page 81: OBM - Teoría de Números.pdf

POT 2012 - Teoria dos Numeros - Nıvel 2 - Aula 12 - Samuel Feitosa

Exemplo 9. Prove que para todo inteiro n > 2, existem inteiros positivos p e q tais quen2 + q2 = p2.

Fatorando a expressao, obtemos n2 = (p− q)(p+ q). Se n e ımpar, podemos encontrar p eq tais que p+ q = n2 e p− q = 1, bastando para isso resolver o sistema originado, obtendo(n, q, p) = (n, n

2−1

2, n

2+1

2). Se n e par, podemos fazer algo semelhante e encontrar p e q tais

que p+ q = n2/2 e p− q = 2, cuja solucao e (n, q, p) = (n, n2

4− 1, n

2

4+ 1).

Exemplo 10. (Extraıdo de [3]) Prove que a equacao

x2 + y2 + z2 + w2 = 2xyzw (1)

nao possui solucoes inteiras positivas.

Por contradicao, suponha que (1) possua pelo menos uma solucao nao-trivial, digamos(x0, y0, z0, w0). Se x0, y0, z0, w0 forem todos ımpares, o lado esquerdo e um multiplo de 4e o lado direito nao. Se apenas um ou tres deles forem pares, o lado esquerdo e ımpar e odireito e par. Se dois deles forem pares e dois forem ımpares, o lado direito e um multiplode quatro e o esquerdo nao. Desse modo, x0, y0, z0, w0 sao todos pares, ou seja, x0 = 2x1,y0 = 2y1, z0 = 2z1 e w0 = 2w1. Substituindo em (1) e dividindo por quatro, concluımosque x1, y1, z1, w1 satisfazem a igualdade

x21 + y21 + z21 + w21 = 8x1y1z1w1.

Com uma analise de paridades analoga a acima, obtemos x1 = 2x2, y1 = 2y2, z1 = 2z2 ew1 = 2w2, e daı

x22 + y22 + z22 +w22 = 32x2y2z2w2.

Procedendo dessa maneira, x0, y0, z0, w0 devem ser todos multiplos de 2n, qualquer que sejan ≥ 1. Entao x0 = y0 = z0 = w0 = 0, absurdo.

Exemplo 11. (Extraıdo de [3]) Encontre todas as quadruplas (x, y, z, k) de numeros inteiros,com x, y, z > 0 e k ≥ 0, tais que

x6 + y6 + z6 = 4826 · 7k.

Vamos mostrar o seguinte fato:

(x, y, z, k) e solucao, com k ≥ 1 ⇐⇒ (x/7, y/7, z/7, k − 6) e solucao,

e nesse caso k ≥ 6.

(=⇒) Temos x6 + y6 + z6 ≡ 0 (mod 7). Como x6, y6, z6 ≡ 0 ou 1 (mod 7), devemos terx, y, z multiplos de 7. Daı, 76|4826 · 7k ⇒ 76|7k ⇒ k ≥ 6. Ademais, vale a igualdade

(x

7

)6

+(y

7

)6

+(z

7

)6

= 4826 · 7k−6,

4

Page 82: OBM - Teoría de Números.pdf

ou seja, (x/7, y/7, z/7, k − 6) tambem e solucao.(⇐=) Claro.

O fato acima garante que podemos ir subtraindo 6 de k e retirando um fator 7 de x, y,z enquanto k ≥ 1, ate que o expoente de 7 no lado direito da igualdade seja 0. Em outraspalavras, existe n ≥ 0 tal que k = 6n, com x = 7n · x0, y = 7n · y0, z = 7n · z0, e

x06 + y0

6 + z06 = 4826.

A equacao acima so tem a solucao (1, 3, 4) e suas permutacoes. Assim, as solucoes daequacao original sao (7n, 3 · 7n, 4 · 7n, 6n), n ≥ 0, e suas permutacoes nas tres primeirascoordenadas.

Problemas Propostos

Problema 12. Mostre que se a · b = x2 e mdc(a, b) = 1 entao existem r e s tais que a = r2

e b = s2.

Problema 13. Prove que todas as solucoes positivas da equacao1

x2+

1

y2=

1

z2com

mdc(x, y, z) = 1 sao dadas por

(x, y, z) = (r4 − s4, 2rs(r2 + s2), rs(r2 − s2))

ou(x, y, z) = (2rs(r2 + s2), r4 − s4, rs(r2 − s2)),

onde r > s > 0, mdc(r, s) = 1 e r e s de paridades opostas.

Problema 14. Encontre todos os pares de racionais (x, y) tais que x2 + y2 = 1.

Problema 15. Resolva simultaneamente em inteiros positivos:

a2 + b2 = c2

a2 + c2 = d2

onde a, b, c e d sao inteiros positivos relativamente primos entre si dois dois.

Problema 16. (Torneio das Cidades 1997) Prove que a equacao

x2 + y2 − z2 = 1997

tem infinitas solucoes inteiras (x, y, z).

Problema 17. Encontre todas as solucoes inteiras de x2 + y2 + z2 = t2.

Problema 18. Encontre todas as solucoes de 5m2 + n2 = 52011

Problema 19. Encontre todas as solucoes em numeros naturais m e n da equacao:

m2 = 1 + 2 + . . .+ n.

Page 83: OBM - Teoría de Números.pdf

Referencias

[1] F. E. Brochero Martinez, C. G. Moreira, N. C. Saldanha, E. Tengan - Teoria dosNumeros: um passeio com primos e outros numeros familiares pelo mundo inteiro,Projeto Euclides, IMPA, 2010.

[2] E. Carneiro, O. Campos and F. Paiva, Olimpıadas Cearenses de Matematica 1981-2005(Nıveis Junior e Senior), Ed. Realce, 2005.

[3] S. B. Feitosa, B. Holanda, Y. Lima and C. T. Magalhaes, Treinamento Cone Sul 2008.Fortaleza, Ed. Realce, 2010.

[4] D. Fomin, A. Kirichenko, Leningrad Mathematical Olympiads 1987-1991, MathProPress, Westford, MA, 1994.

[5] D. Fomin, S. Genkin and I. Itenberg, Mathematical Circles, Mathematical Words, Vol.7, American Mathematical Society, Boston, MA, 1966.

[6] I. Niven, H. S. Zuckerman, and H. L. Montgomery, An Introduction to the Theory ofNumbers.

Page 84: OBM - Teoría de Números.pdf

Polos Olímpicos de TreinamentoCurso de Teoria dos Números - Nível 2Prof. Samuel Feitosa

Aula 13

Equacoes Diofantinas III

Ja estudamos as equacoes diofantinas lineares e equacoes em que alguma fatoracao conveni-ente poderia facilitar a busca por solucoes. Nesta aula, estaremos interessados em encontrarmodulos convenientes para analisar os termos de uma equacao.

Exemplo 1. Encontre todas as solucoes em inteiros da equacao x2 − 7y = 1004.

Analisando os restos na divisao por 7, obtemos x2 ≡ 3 (mod 7). Entretando, os unicosinteiros que sao restos de quadrados perfeitos na divisao por 7 sao 0, 1, 2 e 4. Como3 ≡ 1004 (mod 7) nao faz parte dessa lista, nao existem soulcoes inteiras para a equacao.

Exemplo 2. Encontre todas as solucoes em inteiros da equacao x3 + 98y2 + 5 = 0.

Analisemos os possıveis restos de x3 (mod 7) fazendo uma tabela dos restos correspondentesde x e x3:

x 0 1 2 3 4 5 6

x3 0 1 1 6 1 6 6

Como os unicos restos possıveis sao 0, 1,−1 (mod 7), o lado esquerdo da equacao so podedeixar resto 5, 6, 4 (mod 7). Como o resto do lado direito nao faz parte dessa lista, naoexistem solucoes em inteiros.

Exemplo 3. Prove que a equacao x2 = 3y2 + 8 nao possui solucoes em inteiros x e y.

Analisando o resto na divisao por 3, obtemos x2 ≡ 2 (mod 3). Como os unicos restos deum quadrado por 3 sao 0 e 1, nao existem solucoes em inteiros.

Nos proximo problema, usaremos congruencias para encontrarmos informacoes sobre asincognitas envolvidas nos expoentes e buscaremos alguma fatoracao apropriada para reduziro problema a resolucao de um sistema de equacoes.

Exemplo 4. Encontre todas as solucoes em inteiros positivos da equacao 3m + 7 = 2n

Page 85: OBM - Teoría de Números.pdf

POT 2012 - Teoria dos Numeros - Nıvel 2 - Aula 13 - Samuel Feitosa

Analisando o resto modulo 3 do lado esquerdo, podemos concluir que 2n ≡ 1 (mod 3).Como 2n ≡ (−1)n (mod 3), concluımos que n e par, ou seja, n = 2k, para algum k ∈ N.Assim, como o lado direito e multiplo de 4, podemos concluir que:

3m ≡ −7 (mod 4)

(−1)m ≡ 1 (mod 4)

Logo, m e par, ou seja, m = 2t, para algum t ∈ N. Usando diferenca de quadrados,podemos escrever:

7 = (2k − 3t)(2k + 3t).

Como 7 e primo, temos as seguintes opcoes:

7 = 2k + 3t ⇒ 2k − 3t = 1

1 = 2k + 3t ⇒ 2k − 3t = 7

Em ambos os casos, 8 = 2k+1 e daı k = 2. Substituindo nas equacoes, obtemos solucaoapenas no primeiro caso com t = 1. Assim, (m,n) = (2, 4).

Exemplo 5. Encontre todas as solucoes em inteiros positivos da equacao 3 · 2m + 1 = n2.

Analisandoa equacao modulo 3, n2 ≡ 1 (mod 3) e assim, n ≡ ±1 (mod 3). No primeirocaso, se n = 3k + 2, temos 3 · 2m + 1 = n2 = 9k2 + 12k + 4 e daı 2m = (3k + 1)(k + 1).Como o lado esquerdo possui apenas um fatores 2, temos 3k + 1 = 2i, k + 1 = 2j ,comj ≤ i. Daı, 3 · 2j − 2i = 2. Se j = i, temos 2i+1 = 2 e consequentemente i = 0 produzindok = 0 e (m,n) = (0, 2). Se j < i, temos j = 1 pois o lado esquerdo possui um unico fator2 e por conseguinte, i = 2, (m,n) = (3, 5). No segundo caso, quando n = 3k+ 1, e tratadoanalogamente produzindo apenas a nova solucao (m,n) = (4, 7).

Exemplo 6. Encontre todas as solucoes da equacao x2 − xy + y = 3 em inteiros x, y.

Fixado o valor de y, podemos encontrar os valores de x usando a formula de Baskara. Comox e inteiro, o discriminante y2 − 4(y − 3) = (y − 2)2 + 8 deve ser um quadrado perfeito,digamos z2. Assim,

z2 − (y − 2)2 = (z − y + 2)(z + y − 2) = 8.

Como z−y+2 e z+y−2 possuem a mesma paridade, o produto anterior dever (±2) · (±4).Em qualquer caso, somando ambos os termos, obtemos 2z = ±6 e z = ±3. Logo, y−2 = ±1.Substituindo os valores de y na equacao original, obtemos os valores correspondentes parax. As solucoes sao: (x, y) = (2, 1), (−1, 1), (0, 3), (3, 3).

Exemplo 7. (Hungria 1969) Seja n um inteiro positivo. Prove que se 2+2√28n2 + 1 e um

inteiro, entao e um quadrado perfeito.

2

Page 86: OBM - Teoría de Números.pdf

POT 2012 - Teoria dos Numeros - Nıvel 2 - Aula 13 - Samuel Feitosa

Necessariamente√28n2 + 1 deve ser racional e para isso 28n2 + 1 deve ser um quadrado

perfeito. Assim,

28n2 + 1 = t2

7n2 =

(

t− 1

2

)(

t+ 1

2

)

Como 7 e primo, 7 | t+ 1

2ou que 7 | t− 1

2. No primeiro caso,

n2 + 1 =

(

t+ 1

14

)(

t− 1

2

)

Alem disso, como mdc((t− 1)/2, (t+ 1)/2) = 1, existem a e b tais que

a2 =t+ 1

14

b2 =t− 1

2

Daı, 7a2 − b2 = 1 e b2 ≡ −1 (mod 7). Como quadrados perfeitos so podem deixar restos0, 1, 2, 4 (mod 7), esse caso nao gera solucoes. No segundo caso,

a2 =t+ 1

2

b2 =t− 1

14.

Logo, 2 + 2√28n2 + 1 = 2 + 2t = 2 + 2(2a2 − 1) = (2a)2.

Exemplo 8. (Reino Unido 1996) Encontre todas as solucoes em inteiros nao negativosx, y, z da equacao:

2x + 3y = z2.

Se y = 0, entao 2x = z2 − 1 = (z − 1)(z + 1). Analisando a fatoracao em primos, existemi, j, com i > j, tais que z + 1 = 2i e z − 1 = 2j . A diferenca das duas equacoes produz2 = 2i − 2j = 2j(2i−j − 1). Como o lado esquerdo possui apenas um fator 2, j = 1 ei − j = 1. Nossa primeira solucao encontrada e (x, y, z) = (3, 0, 3). Se y > 0, 2x ≡ z2

(mod 3). Como 2x ≡ ±1 (mod 3) e z2 ≡ 0, 1 (mod 3) temos, 2x ≡ z2 ≡ 1. Isso implicaque x e par, ou seja, x = 2m. Fatorando, obtemos:

3y = z2 − 22m

= (z − 2m)(z + 2m)

Novamente, analisando a fatoracao em primos, existem l e k, com l < k, tais que z− 2m =3l, z + 2m = 3k. A diferenca das duas equacoes produz 2m+1 = 3l(3k−l − 1). Novamente

3

Page 87: OBM - Teoría de Números.pdf

POT 2012 - Teoria dos Numeros - Nıvel 2 - Aula 13 - Samuel Feitosa

analisando a fatoracao em primos, l = 0 e 2m+1 = 3k − 1. Se m = 0, temos k = 1 e(x, y, z) = (0, 1, 2). Se m > 0,

3k = 1 (mod 4)

(−1)k = 1 (mod 4).

e devemos ter k par, ou seja, existe t tal que k = 2t. Fatorando novamente, 2m+1 =(3k − 1)(3k + 1). Escrevendo 3k + 1 = 2p e 3k − 1 = 2q, temos 2 = 2q(2p−q − 1). Vejaque ja tratamos essa equacao no inıcio e assim podemos concluir que q = 1 e p − q = 1.Produzindo a solucao (x, y, z) = (4, 2, 5).

Nos proximos dois problemas, contruiremos solucoes indutivamente.

Exemplo 9. (Bulgaria) Prove que para qualquer numero natural n ≥ 3, existem numerosnaturais ımpares xn e yn tais que 7x2n + y2n = 2n.

Para n = 3, basta tomar x1 = y1 = 1. Suponha que tenhamos encontrado xk e yk ımpares,satisfazendo

7x2k + y2k = 2k.

Um dos numeros (xk + yk)/2, (xk − yk)/2 e ımpar e assim podemos escolher um deles demodo a satisfazer o enunciado para k + 1:

7

(

xk ± yk2

)2

+

(

xk ∓ yk2

)2

= 2(7x2k + y2k) = 2k+1.

Exemplo 10. Mostre que existe uma sequencia infinita de inteiros positivos a1, a2, . . . taisque a21 + a22 + . . .+ a2n e um quadrado perfeito para todo n natural.

Definamos a1 = 3. Suponha que a sequencia ja esteja definida para a1, a2, . . . , ak com

a21 + a22 + . . .+ a2k = (2t+ 1)2.

Vejamos que podemos definir o proximo termo de modo que a soma de todos os primeirosk + 1 termos ao quadrado ainda seja um quadrado perfeito de um inteiro ımpar. Bastafazer ak+1 = 2t2 + 2t. Veja que:

a21 + a22 + . . .+ a2k + a2k+1 = (2t+ 1)2 + (2t2 + 2t)2

= (2t2 + 2t+ 1)2.

que e novamente o quadrado de um ımpar.

Problemas Propostos

Problema 11. Encontre todas as solucoes em inteiros x, y, z, t da equacao:

x2 + y2 + z2 = 8t− 1.

4

Page 88: OBM - Teoría de Números.pdf

POT 2012 - Teoria dos Numeros - Nıvel 2 - Aula 13 - Samuel Feitosa

Problema 12. Encontre todas as solucoes em inteiros positivos da equacao

1

a+

1

b+

1

c= 1.

Problema 13. Encontre todas as solucoes em inteiros de x2 − y2 − 1988

Problema 14. Mostre que para todo inteiro z, existem inteiros x e y satisfazendo x2−y2 =z3

Problema 15. Encontre todas as solucoes de 1 + x+ x2 + x3 = 2y em inteiros positivos xe y.

Problema 16. Mostre que a equacao diofantina 5m2 − 6mn + 7n2 = 1988, nao possuisolucao nos inteiros.

Problema 17. (Russia 1996) Sejam x, y, p, n, k numeros naturais tais que

xn − yn = pk.

Prove que se n > 1 e ımpar, e p e um primo ımpar, entao n e uma potencia de p.

Problema 18. (Russia 1997) Encontre todas as solucoes inteiras da equacao

(x2 − y2)2 = 1 + 16y.

Problema 19. (OBM 2009) Prove que nao existem inteiros positivos x e y tais que x3+y3 =22009.

Apendice: A conjectura de Catalan

Em alguns dos problemas anteriores, nos deparamos com a questao de encontrarmos duaspotencias perfeitas consecutivas nao triviais. As unicas solucoes que apareceram foram23 = 8 e 32 = 9. Em 1844, Eugene Catalan conjecturou que essa seria a unica solucao.Recentemente, tal conjectura se mostrou verdadeira atraves do:

Teorema 20. (Mihalnescu - 2002)Existe uma unica solucao nos numeros naturais de

xa − yb = 1,

com x, a, y, b > 1 que e (x, y, a, b) = (3, 2, 2, 3).

Problema 21. Encontre toda as solucoes em inteiros positivos da seguinte equacao diofan-tina:

2y2 = x4 + x.

5

Page 89: OBM - Teoría de Números.pdf

Polos Olímpicos de TreinamentoCurso de Teoria dos Números - Nível 2Prof. Samuel Feitosa

Aula 14

A Funcao Parte Inteira - I

1 O jogo de Wythoff

O objetivo da aula de hoje e resolver o seguinte problema:

Exemplo 1. Dois jogadores jogam alternadamente removendo pedras de duas pilhas sobreuma mesa. Na sua vez, cada jogador pode remover qualquer quantidade de pedras de umapilha ou igual numero de pedras de ambas as pilhas. O ganhador e aquele que retirar aultima pedra. Determine todas posicoes perdedoras.

Uma boa estrategia para identificar as posicoes perdedoras nesse jogo, e associar o mo-vimento dos jogadores ao movimento de uma peca em um tabuleiro. Suponha que ini-cialmente as duas pilhas possuem 5 e 7 pedras. Colocaremos uma peca no canto direitosuperior em um tabuleiro 8 × 6. O movimento de retirar x pedras da coluna de 5 seratraduzido como um deslocamento vertical de x casas para baixo, enquanto que a mesmaretirada da outra coluna sera traduzido como um movimento horizontal para a esquerdade mesmo deslocamento. Um movimento de retirada de x pedras de ambas as colunassera traduzido como um deslocamento diagonal da direita para a esquerda dessa mesmaquantidade de casas. O jogo terminara quando a peca chegar na casa do canto extremoesquerdo simbolizando que ambas as colunas estao com 0 pedras.

A posicao (0, 0) e perdedora porque uma vez que um jogador a receba, ele tera perdidoo jogo. Qualquer posicao do tipo (x, 0), (0, x) ou (x, x), com x > 0, sera uma posicaovencedora. Marquemos essas posicoes no tabuleiro:

Page 90: OBM - Teoría de Números.pdf

POT 2012 - Teoria dos Numeros - Nıvel 2 - Aula 14 - Samuel Feitosa2 DEFINICAO E PROPRIEDADES

−+++++

++

++

+

+ + + + + + +

As proximas posicoes perdedoras que encontramos sao (1, 2),(2, 1). A partir dessa novaposicao, podemos preencher o tabuleiro com as novas posicoes vencedoras.

−+++++

++

++

+

+ + + + + + +

−+

++

−+

++

+

+++

++++

+ + + + + ++ + + + +

As proximas posicoes perdedoras que encontramos sao (3, 5) e (5, 3). Como existe simetriaentre as duas pilhas, basta procurarmos as posicoes perdedoras (x, y) com x < y. Repetindoo processo anterior, podemos listar as primeiras posicoes perdedoras ordenadas (xn, yn) comxn < yn.

n 0 1 2 3 4 5 6 7 8 9 10 11 12

xn 0 1 3 4 6 8 9 11 12 14 16 17 19

yn 0 2 5 7 10 13 15 18 20 23 26 28 31

As proximas secoes nos ajudarao a estabelecer alguma padrao entre os valores de (xn, yn)em funcao de n.

2 Definicao e Propriedades

Definicao 2. A parte inteira de um numero real x e o maior inteiro ⌊x⌋ que nao e maiorque x. Definimos a parte fracionaria {x} de x por {x} = x − ⌊x⌋. (exemplos: ⌊3⌋ =3 , ⌊3, 5⌋ = 3 e ⌊−4, 7⌋ = −5)

Teorema 3. Sejam x e y numeros reais. Entao:

1. ⌊x⌋ ≤ x < ⌊x⌋+ 1 e 0 ≤ {x} < 1.

2. ⌊x+m⌋ = ⌊x⌋+m se m e um inteiro.

3. ⌊x⌋ + ⌊y⌋ ≤ ⌊x+ y⌋ ≤ ⌊x⌋+ ⌊y⌋+ 1.

4.⌊

⌊x⌋m

=⌊

xm

se m e um inteiro positivo.

5. Se n e a sao inteiros positivos,⌊n

a

e o numero de inteiros entre 1, 2, ..., n que sao

divisıveis por a.

2

Page 91: OBM - Teoría de Números.pdf

POT 2012 - Teoria dos Numeros - Nıvel 2 - Aula 14 - Samuel Feitosa2 DEFINICAO E PROPRIEDADES

Demonstracao. Os primeiros dois itens decorrem facilmente da definicao e serao deixadosa cargo do leitor.Para provar (3), veja que:

⌊x⌋+ ⌊y⌋ ≤ ⌊x⌋+ ⌊y⌋+ ⌊{x}+ {y}⌋= ⌊⌊x⌋+ ⌊y⌋+ {x}+ {y}⌋= ⌊x+ y⌋.

Como {x}+ {y} < 2, ⌊{x}+ {y}⌋ ≤ 1 e daı:

⌊x+ y⌋ = ⌊x⌋+ ⌊y⌋+ ⌊{x}+ {y}⌋≤ ⌊x⌋+ ⌊y⌋+ 1.

Para provar (4), seja ⌊x⌋ = qm+ r com 0 ≤ r < m− 1, entao:

⌊⌊x⌋m

=⌊

q +r

m

= q.

Como 0 ≤ {x} < 1,

q = q +

r + {x}m

=

qm+ r + {x}m

=⌊ x

m

.

Finalmente, para provar (5), sejam a, 2a, ..., ja todos os inteiros positivos ≤ n que saodivisıveis por a. Entao,

ja ≤ n < (j + 1)a ⇒ j ≤ n

a< j + 1

⇒ j =⌊n

a

.

Observacao 4. Em alguns dos problemas desta secao, sera usada a notacao de somatorio.Recomenda-se que o professor escrevar por extenso os primeiros somatorios ate que osalunos se sintam confortaveis com a manipulacao dos ındices.

Teorema 5 (Formula de Polignac). Seja p um primo. Entao o maior expoente p na fa-toracao em primos de n! e:

tp(n) =∞∑

i=1

n

pi

.

Demonstracao. O que significa

n

pi

? Ele conta o numero de inteiros positivos menores ou

iguais a n divisıveis por pi. Cada multiplo de p contribui com um expoente 1 para p em n!,cada multiplo de p2 contribui com expoente 2 para p em n! e assim sucessivamente. Entao,∞∑

i=1

n

pi

e a soma de todas essas contribuicoes (veja que um multiplo de pi e contado i

vezes em⌊

np

,⌊

np2

, ...,⌊

npi

).

3

Page 92: OBM - Teoría de Números.pdf

POT 2012 - Teoria dos Numeros - Nıvel 2 - Aula 14 - Samuel Feitosa2 DEFINICAO E PROPRIEDADES

Observacao 6. Se p e primo e pα e a maior potencia de p que divide n, usaremos a notacaopα ‖ n.

Exemplo 7. Em quantos zeros termina a representacao decimal de 1000!?

Para determinarmos o numero de zeros, basta determinarmos a maior potencia de 10 quedivide 1000!. Existem mais fatores 2 do que fatores 5 e assim bastara encontrarmos oexpoente de 5 na fatoracao de 1000!. Pelo teorema anterior, tal numero e:

∞∑

i=1

1000

5i

= 200 + 40 + 8 + 1 = 249.

Exemplo 8. Mostre quek−1∑

i=0

x+i

k

= ⌊kx⌋.

Sejam a = ⌊x⌋, b = {x} e j ∈ {0, 1, . . . , k − 1} tal quej

k≤ b <

j + 1

k. Entao ⌊kb⌋ = j.

Alem disso, ⌊b+ i/k⌋ = 0 se i < k − j e ⌊b+ j/k⌋ = 1 se k − j ≤ i ≤ k − 1. Daı,

k−1∑

i=0

⌊b+ i/k⌋ =k−1∑

i=k−j

⌊b+ i/k⌋

= j

= ⌊kb⌋

Exemplo 9. Mostre que ⌊x+ y⌋+ ⌊x⌋+ ⌊y⌋ ≤ ⌊2x⌋+ ⌊2y⌋.Sejam a = {x} e b = {y}. A desigualdade e equivalente a

2⌊x⌋+ 2⌊y⌋+ ⌊a⌋+ ⌊b⌋+ ⌊a+ b⌋ ≤ 2⌊x⌋+ 2⌊y⌋+ ⌊2a⌋+ ⌊2b⌋.que pode ser reescrita como:

⌊a⌋+ ⌊b⌋+ ⌊a+ b⌋ ≤ ⌊2a⌋+ ⌊2b⌋.

Temos que 0 ≤ ⌊a⌋ + ⌊b⌋ < 2 e ⌊a⌋ = ⌊b⌋ = 0. Se ⌊a + b⌋ = 1 segue que pelo menos umdentre a, b e maior ou igual a 1/2. Daı,

⌊a⌋+ ⌊b⌋+ ⌊a+ b⌋ = 1

≤ ⌊2a⌋+ ⌊2b⌋.

Caso contrario, ⌊a⌋+ ⌊b⌋+ ⌊a+ b⌋ = 0 e a desigualdade segue.

Exemplo 10. Mostre que se m e n sao inteiros positivos, entao(2m)!(2n)!

(m)!(n)!(m+ n)!e um

inteiro.

4

Page 93: OBM - Teoría de Números.pdf

POT 2012 - Teoria dos Numeros - Nıvel 2 - Aula 14 - Samuel Feitosa2 DEFINICAO E PROPRIEDADES

Pelo teorema anterior, basta mostrarmos que:

2m

pk

+

2n

pk

≥⌊

m

pk

+

n

pk

+

m+ n

pk

,

para todo primo p e todo inteiro k. A desigualdade segue do exemplo anterior.

Exemplo 11. Prove que(

2nn

)

divide MMC{1, 2, ..., 2n}.

Seja p um primo. Se pα ‖(

2nn

)

e pβ ≤ 2n < pβ+1, pelo teorema anterior, temos:

α =∞∑

j=1

2n

pj

− 2

n

pj

=

β∑

j=1

2n

pj

− 2

n

pj

≤ β,

pois ⌊2x⌋ − 2⌊x⌋ = 0 ou 1. Como pβ ‖ MMC{1, 2, ..., 2n} e α ≤ β, segue o resultado.

Exemplo 12. Mostre que∞∑

k=1

n

2i+

1

2

= n.

O numero de inteiros que sao multiplos de 2k, mas nao de 2k+1 e ⌊n/2k⌋ − ⌊n/2k+1⌋. Sen = 2k+1q+ r, esse numero e q+ ⌊r/2k⌋. Para x ∈ [0, 1), e verdade que ⌊2x⌋ = ⌊x+ 1/2⌋.Assim,

⌊n/2k⌋ − ⌊n/2k+1⌋ = q + ⌊r/2k⌋= ⌊r/2k + 1/2 + q⌋= ⌊r/2k+1 + 1/2⌋

Somando a quantidade de numeros que sao multiplos de 2k mas nao de 2k+1 entre 1 e npara k = 0, 1, . . . obtemos a quantidade total de numeros, ou seja,

∞∑

k=1

n

2i+

1

2

= n.

Teorema 13. Seja vp(n) a soma dos dıgitos da representacao de n na base p. Mostre que

o expoente de p na fatoracao em primos de n! en− vpn

p− 1.

Demonstracao. Seja kp(n!) o maior expoente de p que divide n!. Considere a representacaode n na base p: n = d0 + d1p+ ...+ drp

r onde 0 ≤ di < p. Entao,

5

Page 94: OBM - Teoría de Números.pdf

POT 2012 - Teoria dos Numeros - Nıvel 2 - Aula 14 - Samuel Feitosa2 DEFINICAO E PROPRIEDADES

n

p

= d1 + d2p+ . . .+ drpr−1,

n

p2

= d2 + d3p+ . . .+ drpr−2,

. . .⌊

n

p

= dr.

Somando tudo, obtemos:

kp(n!) = d1 + d2(p+ 1) + d3(p2 + p+ 1) + · · ·+ dr(p

r−1 + · · ·+ 1)

=1

p− 1

(

d1(p− 1) + d2(p2 − 1) + · · ·+ dr(p

r − 1))

=n− vp(n)

p− 1

Exemplo 14. Seja B(m) o conjunto dos inteiros r tais que 2r e um termo na representacaona base 2 de n. Por exemplo, B(100) = {2, 5, 6} pois 100 = 22 + 25 + 26. Prove que

(

nk

)

eımpar se, e somente se, B(k) ⊆ B(n).

Aproveitando a notacao dos teoremas anteriores, t2(n) = n− v2(n). Assim,

(

n

k

)

≡ 1 (mod 2) ⇔ [n− v2(n)]− [(k − v2(k))− ((n− k)− v2(n− k))] = 0

⇔ v2(k) + v2(n− k)− v2(n) = 0

⇔ v2(k) + v2(n− k) = v2(n).

A ultima equacao nos diz que na soma das representacoes na base 2 de n−k e k nao ocorrea operacao de ”vai um”, ou seja, B(k) ⊆ B(n).

Observacao 15. Esse exemplo tambem mostra que(

nk

)

≡ 0 (mod 2) para todo k ∈ {1, 2, . . . , n−1} se, e somente se, n e uma potencia de 2.

Exemplo 16. (Olimpıada Rioplatense) Seja r um real tal que

r +19

100

+

r +20

100

+ · · ·+⌊

r +92

100

= 554.

Calcule ⌊100r⌋ .Sejam ⌊r⌋ = a e {x} = b. Podemos reescrever a equacao como:

74a+

73∑

i=0

b+19 + i

100

= 554.

6

Page 95: OBM - Teoría de Números.pdf

POT 2012 - Teoria dos Numeros - Nıvel 2 - Aula 14 - Samuel Feitosa2 DEFINICAO E PROPRIEDADES

Como 0 ≤⌊

b+19 + i

100

< 2 para todo i ∈ {0, 1, . . . , 73}, segue que 74a ≤ 554 ≤ 74a+ 74.

Nao podemos ter 554 = 74(a+1) pois 74 ∤ 554. Logo, a < 554/74 < a+1 e por conseguintea = ⌊554/74⌋ = 7. Assim,

73∑

i=0

b+19 + i

100

= 554− 74 · 7 = 36.

Pelo primeiro exemplo,99∑

i=0

b+i

100

= j.

ondej

100≤ b <

j + 1

100. Nao podemos ter

r +19

100

= 1 pois nesse caso terıamos

73∑

i=0

r +19 + i

100

≥ 73 6= 36.

Logo,

r +i

100

= 0 para todo i ≤ 19. Como

r +i

100

= 1 se 100− j ≤ i ≤ 99, temos:

⌊100b⌋ = j

=99∑

i=0

r +i

100

=92∑

i=19

r +i

100

+

(⌊

r +93

100

. . .

r +99

100

⌋)

= 36 + 7

= 43.

Finalmente,

⌊100r⌋ = 100a+ ⌊100b⌋= 700 + 43

= 743.

Exemplo 17. Prove que existe um natural n tal que a representacao decimal de n2 comeca( da esquerda para a direita ) com o numero 201120112011. . . 2011 ( 2011 vezes).

Podemos encontrar n tal que n2 comece com qualquer sequencia de dıgitos (c1c2...cr) = m.Tome k suficientemente grande tal que 2

√m < 10k−1. Seja n = ⌊10k√m+ 1⌋. Entao,

0 < 10k√m < n ≤ 10k

√m+ 1 ⇒

102km < n2 ≤ 102km+ 2.10k√m+ 1 ⇒

102km < n2 ≤ 102km+ 102k−1 + 1 ⇒102km < n2 < 102k(m+ 1).

Assim, n2 comeca com a sequencia de dıgitos m.

7

Page 96: OBM - Teoría de Números.pdf

POT 2012 - Teoria dos Numeros - Nıvel 2 - Aula 14 - Samuel Feitosa2 DEFINICAO E PROPRIEDADES

Exemplo 18. (OBM 1999) Prove que ha pelo menos um algarismo diferente de zero entrea 1000000a e a 3000000a casa decimal de

√2 apos a vırgula.

Suponha que nao, entao 102·106⌊10106

√2⌋ = ⌊103·106

√2⌋. Se k = ⌊10106

√2⌋, temos:

102·106

k ≤ 103·106√2 < 102·10

6

k + 1 ⇒k

10·106<

√2 <

k

10·106+

1

103·106⇒

k2

102·106< 2 <

k2

102·106+

2k

104·106+

1

106·106

Como2k

102·106<

2√21010

6

102·106≤ 1

2,

k2 < 2 · 102·106 < k2 + 1 ⇒0 < 2 · 102·106 − k2 < 1.

Isso e um absurdo pois 2 · 102·106 − k2 ∈ Z.

Exemplo 19. ( Sequencia de Beatty) Se α e β sao irracionais satisfazendo 1

α+ 1

β= 1 entao,

as sequencias⌊α⌋, ⌊2α⌋, ⌊3α⌋, . . . ;

e⌊β⌋, ⌊2β⌋, ⌊3β⌋, . . . ;

incluem todos os numeros naturais exatamente uma vez.

Primeiramente, provemos a unicidade. Suponha que ⌊kα⌋ = ⌊lβ⌋ = n, como α e β saoirracionais, n < kα < n+ 1 e n < lβ < n+ 1. Assim,

k

n+ 1+

l

n+ 1<

1

α+

1

β

<k

n+

l

n

Isso nos diz quek + l

n+ 1< 1 <

k + l

n. Temos um absurdo pois a desigualdade anterior diz

que o inteiro k + l esta entre dois inteiros consecutivos.

Mostremos agora que todo natural aparece nas sequencias. Dado n ∈ N, existe k ∈ Z+ talque

k − 1

n<

1

α<

k

n.

8

Page 97: OBM - Teoría de Números.pdf

POT 2012 - Teoria dos Numeros - Nıvel 2 - Aula 14 - Samuel Feitosa3 A SOLUCAO DO PROBLEMA INICIAL

Dividamos o intervalo [k/(n+1), k/n] em duas partes. Sek − 1

n<

1

α<

k

n, temos ⌊kα⌋ = n.

Se por outro lado,k − 1

n<

1

α<

k

n+ 1, temos:

k − 1

n< 1− 1

β<

k

n+ 1⇒

n+ 1− k

n+ 1<

1

β<

n+ 1− k

n⇒

⌊(n+ 1− k)β⌋ = n

Em qualquer caso, n faz parte da sequencia.

3 A solucao do problema inicial

Voltemos ao exemplo inicial. Veja que toda linha ou coluna do tabuleiro deve possuir nomaximo uma posicao perdedora. Se a k-esima coluna nao possuir nenhuma posicao per-dedora, para cada uma de suas casinhas, poderemos encontrar um posicao perdedora namesma linha ou diagonal e isso implicaria na existencia de uma infinidade de posicoesperdedoras entre as k − 1 primeiras colunas. O mesmo argumeto se aplica para as linhas.Consequentemente, cada natural aparece exatamente uma vez dentre os termos da sequenciax0, x1, . . . ; y0, y1, . . .. Alem disso, tambem e facil concluir que as sequencias (xn) e (yn) saocrescentes. Induzidos do exemplo anterior, isso nos leva a conjecturar a existencia de doisirracionais α e β que possam de alguma forma gerar os termos das posicoes perdedoras.

A insercao dos pontos (xn, yn) em um grafico sugere que esses pontos estao proximos aalguma reta. Isso poderia ser traduzido dizendo que o quociente yn/xn e proximo a algum

valor. De fato, se α =1 +

√5

2, xn ≈ αn e yn ≈ (α+ 1)n. Como as posicoes sao numeros

inteiros, podemos conjecturar que:

Conjectura 20. Se α =1 +

√5

2, entao (xn, yn) = (⌊n · α⌋, ⌊n · (α+ 1)⌋)

Veja que α e irracional e que:

1

α+

1

α+ 1=

2α+ 1

α2 + α

=2α+ 1

2α+ 1= 1.

Provemos a afirmacao anterior por inducao. Ela e facilmente verificavel para os casosiniciais apresentados na primeira tabela. Suponha sua validade para todos os inteiros noconjunto {0, 1, 2, . . . , k}. Provemos que o mesmo tambem pe valido para k + 1. Seja t omenor natural que nao esta no conjunto {x0, x1, . . . , xk, y0, y1, . . . , yk}. Como as sequencias

9

Page 98: OBM - Teoría de Números.pdf

POT 2012 - Teoria dos Numeros - Nıvel 2 - Aula 14 - Samuel Feitosa3 A SOLUCAO DO PROBLEMA INICIAL

xn e yn sao crescentes e xn < yn, se xk+1 6= t, o inteiro t nao aprecera entre os termosdas sequencias e isso contradiz nossa observacao inicial. Em virtude da unicidade de rep-sentacao da sequencia de Beatty, o inteiro ⌊α(k+1)⌋ ainda nao apareceu dentre os termosdas k + 1 primeiras posicoes perdedoras. Se t < ⌊α(k + 1)⌋, como xn e crescente, deveexistir j tal que ⌊(α+ 1)j⌋ = t com j > k. Nesse caso,

t = ⌊(α+ 1)j⌋≥ ⌊(α+ 1)(k + 1)⌋= ⌊α(k + 1)⌋+ k + 1

> ⌊α(k + 1)⌋

Contrariando a suposicao inicial sobre t. Logo, devemos ter xk+1 = ⌊α(k + 1)⌋. Sejal = yk+1 − xk+1. Se l < k + 1, O movimento diagonal

(xk+1, yk+1) → (xl, yk+1 − xk+1 + xl) = (xl, yl),

passa uma posicao perdedora para contradizendo o fato de que (xk+1, yk+1) era uma posicaoperdedora. Se l > k+1, yk+1 > ⌊(α+1)(k+1)⌋ e o jogador naquela posicao pode removerpedras de apenas uma pilha obtendo:

(xk+1, yk+1) → (⌊α(k + 1)⌋, ⌊(α+ 1)(k + 1)⌋).

Usando novamente hipotese de inducao e lembrando que yi − xi 6= k + 1 para todo i ∈{0, 1, . . . , k}, qualquer movimento do proximo jogador, conduzira a uma posicao em que exa-tamente uma das pilhas possui um numero de pedras igual a um dos numeros x0, y0, . . . , xk, yk.Assim, o oponente podera passar uma posicao perdedora. Novamente temos um absurdopois (xk+1, yk+1). Logo, yk+1 − xk+1 = k + 1 e consequentemente yk+1 = ⌊(α + 1)(k + 1)⌋concluindo a prova da conjectura para k + 1.

Problemas Propostos

Problema 21. Mostre que a parte fracionaria do numero√4n2 + n nao e maior que 0, 25.

Problema 22. Sejam {ai}0≤i≤r, inteiros nao negativos com n = a1 + a2 + · · ·+ ar. Mostre

quen!

a1!a2!...ar!e um inteiro.

Problema 23. Prove que, para qualquer n natural,∞∑

i=1

n+ 2i−1

2i

= n.

Problema 24. Seja p um divisor primo do numero

(

2n

n

)

, com p ≥√2n. Entao o expoente

de p na fatoracao em primos do do numero

(

2n

n

)

e igual a 1.

10

Page 99: OBM - Teoría de Números.pdf

POT 2012 - Teoria dos Numeros - Nıvel 2 - Aula 14 - Samuel Feitosa3 A SOLUCAO DO PROBLEMA INICIAL

Problema 25. (Coreia 1997) Expresse∑n

k=1⌊√k⌋ em termos de n e ⌊√n⌋.

Problema 26. (Canada 1998) Determine o numero de solucoes reais da equacao

⌊a

2

+⌊a

3

+⌊a

5

= a.

Problema 27. Encontre todos o reais α tais que a igualdade ⌊√n⌋+⌊√n+ α⌋ = ⌊

√4n+ 1⌋

e verdadeira para todos os naturais n.

Problema 28. Se a, b, c sao reais e ⌊na⌋ + ⌊nb⌋ = ⌊nc⌋ para todo n natural, entao a ∈ Zou b ∈ Z.

Problema 29. Sejam a, b, c e d numeros reais. Suponha que ⌊an⌋ + ⌊bn⌋ = ⌊cn⌋ + ⌊dn⌋para todos os inteiros positivos n. Mostre que pelo menos um dentre a + b, a − c, a − d einteiro.

Problema 30. Seja n ≥ 3 um inteiro positivo. Mostre que e possıvel eliminar no maximodois elementos do conjunto {1, 2, . . . , n} de modo que a soma dos numeros restantes sejaum quadrado perfeito.

Problema 31. Sejam a,m, b inteiros dados, com mdc(a,m) = 1. Calculem−1∑

x=0

ax+ b

m

.

Problema 32. Encontre todos os naturais n tais que 2n−1 | n!.Problema 33. Determine os pares (a, b) de reais tais que a⌊bn⌋ = b⌊an⌋ para todo inteiropositivo n.

Problema 34. Se p e primo , entao

(

pk

i

)

≡ 0 (mod p)( para 1 ≤ i ≤ pk − 1).

Problema 35. Prove que⌊

( 3√n+ 3

√n+ 2)3

e divisıvel por 8.

Problema 36. Prove que, t1 + t2 + . . .+ tn =⌊n

1

+⌊n

2

+ ...+⌊n

n

, onde tn e o numero

de divisores do natural n.

Problema 37. Prove que, se p e um numero primo, entao a diferenca

(

n

p

)

−⌊

n

p

e divisıvel

por p.

11

Page 100: OBM - Teoría de Números.pdf

Polos Olímpicos de TreinamentoCurso de Teoria dos Números - Nível 2Prof. Samuel Feitosa

Aula 16

Ordem

Definicao 1. O menor inteiro positivo k para o qual ak ≡ 1 (mod m), onde mdc(a,m) = 1,e chamado ”ordem de a modulo m”e sera denotado por ordma.

Teorema 2. Se a e um inteiro relativamente primo com m, entao an ≡ 1 (mod m) se, esomente se, ordma|n. Ademais, an0 ≡ an1 (mod m) se , e somente se, n0 ≡ n1 (mod ordma)

Demonstracao. Sejam b = ordma e n = qb+ r com 0 ≤ r < b. Como ab ≡ 1 (mod m),

an ≡ 1 (mod m) ⇔ aqb+r ≡ 1 (mod m)

⇔ ar ≡ 1 (mod m)

Como 0 ≤ r < b, devemos ter r = 0. Usando que mdc(a,m) = 1 e supondo que n0 > n1,

an0 ≡ an1 (mod m) ⇔ an0−n1 ≡ 1 (mod m)

⇔ n0 − n1 ≡ 0 (mod b)

Teorema 3. Se mdc(a,m) = 1, ordma|φ(m)

Demonstracao. Pelo teorema de Euler, aφ(m) ≡ 1 (mod m). O resultado segue do teoremaanterior.

Problema 4. (Putnam 1972) Prove que nao existe inteiro positivo n > 1 tal que n|2n − 1.

Suponha, por absurdo, que existe um inteiro positivo n > 1 com essa propriedade e que ke o menor dentre eles. Se d = ordk2, entao d | k. Como 2d ≡ 1 (mod k), temos 2d ≡ 1(mod d). Em virtude da minimalidade de k, temos d = 1 ou d = k. No primeiro caso,terıamos k = 1 produzindo uma contradicao. No segundo caso, em decorrencia do teoremaanterior, k | φ(k). Entretanto, se k > 1, φ(k) ≤ k − 1 e obtemos assim um absurdo.

Problema 5. (Leningrado 1990) Prove que para todos os inteiros a > 1 e n , n|φ(an − 1).

Se k = ordan−1a, como an ≡ 1 (mod an − 1), temos k | n e consequentemente k ≤ n. Naopodemos ter k < n porque an − 1 | ak − 1 ⇒ an − 1 ≤ ak − 1. Assim, k = n e usando oteorema anterior podemos concluir que k | φ(an − 1).

Problema 6. Mostre que

a) ord3n2 = 2 · 3n−1

b) Se 2m ≡ −1 (mod 3n), entao ⇒ 3n−1 | m.

Page 101: OBM - Teoría de Números.pdf

Provaremos por inducao que 23k

+ 1 = 3k+1mk com 3 ∤ mk. Suponha que a afirmacao valepara k. Provemos para k + 1:

23k+1

= (3k+1mk − 1)3

= 33k+3m3k − 32k+3m2

k + 3k+2mk − 1

= 3k+2(32k+1m3k − 3k+1 +mk)− 1

= 3k+2mk+1 − 1

Claramente 3 ∤ mk+1. Voltemos ao problema. Seja b = ord3n2, entao b | φ(3n) = 2 · 3n−1.Temos duas possibilidades: ou b = 2 · 3j ou b = 3j . Como 23

n−1

≡ −1 (mod 3n) e 3j | 3n−1

se j ≤ n − 1, devemos ter b = 2 · 3j . Assim, (23j

− 1)(23j

+ 1) ≡ 1 (mod 3)n. Usando que23

j

− 1 ≡ 1 (mod 3), temos 23j

≡ −1 (mod 3n). Novamente pelo lema provado no inıcio,3j ≥ 3n−1 e assim b = 2 · 3n−1. Para o item b), de 2m ≡ −1 (mod 3n), podemos concluirque 22m ≡ 1 (mod 3)n. Daı, 2 · 3n−1 | 2m e o resultado segue.

Problema 7. (Bulgaria 1997) Encontre todos os numeros inteiros m,n ≥ 2 tais que

1 +m3n +m2·3n

n

e um inteiro

Claramente n e ımpar, mdc(m,n) = 1 e n > 2. Se n = 3, como mdc(m,n) = 1 devemoster que m ≡ 1 (mod 3) pois caso contrario 1 + m3n + m2·3n ≡ 1 − 1 + 1 ≡ 1 (mod 3).E facil ver que todo par (m,n) = (3k + 1, 3) e solucao. Suponha agora n > 3 e seja

k = ordnm. Se n > 3 ⇒ m3n 6≡ 1 (mod n). Como 1 + m3n + m2·3n =m3n+1

− 1

m3n − 1segue

que n | m3n+1

− 1 ⇒ k | 3n+1. Logo, k = 3n+1. Pelo teorema de Euler, mφ(n) ≡ 1 (mod n)entao k ≤ φ(n) e 3n+1 ≤ φ(n) ≤ n− 1, uma contradicao.

Problema 8. Prove que que se p e primo, entao pp − 1 tem um fator primo congruente a1 modulo p

Seja q um primo que dividepp − 1

p− 1. Como q | pp − 1 segue que ordqp | p. Se ordqp = 1

entao q | pp − 1 e 0 ≡ pp−1 + pp−2 + . . . p+1 ≡ 1 + 1+ . . .+ 1+ 1 ≡ p (mod q). Mas isso eum absurdo pois p 6= q. Logo ordqp = p e obtemos p | φ(q) = q− 1. Daı, todos os divisores

primos depp − 1

p− 1sao congruentes a 1 modulo p.

Problemas Propostos

Problema 9. Se ordam = h, ordmb = k e mdc(h, k) = 1 mostre que ordmab = hk.

2

Page 102: OBM - Teoría de Números.pdf

Problema 10. Prove que se a, b sao numeros naturais tais que a > b , n > 1, entao cadadivisor primo do numero an− bn e ou da forma nk+1, onde k e um inteiro, ou um divisorde um numero an1 − bn1, onde n1|n e n1 < n.

Problema 11. Prove que se a, b sao numeros naturais tais que a > b , n > 1, entao cadadivisor primo do numero an + bn e ou da forma 2nk + 1, onde k e um inteiro, ou umdivisor de um numero an1 + bn1, onde n1 e o quociente obtido por dividir o numero n porum numero ımpar mairo que 1.

Problema 12. Seja p um primo que nao divide 10, e seja n um inteiro, 0 < n < p. Seja da ordem de 10 modulo p.

1. Mostre que o comprimento do perıodo da representacao decimal de n/p e d.

2. Prove que que se d e par, entao o perıodo da representacao decimal de n/p pode serdividido em duas partes cuja soma e 10d/2 − 1. Por exmeplo, 1/7 = 0, 142857, entaod = 6, e 142 + 857 = 999 = 103 − 1.

3. Se ordma = h ⇒ ordmak =h

mdc(h, k)

Problema 13. Se p e um primo maior que 3, entao qualquer divisor maior que 1 do numero2p + 1

3e da forma 2kp+ 1, onde k e um numero natural.

Teorema 14. Se p e um primo maior que 2, entao qualquer numero natural que divida onumero 2p − 1 e da forma 2kp+ 1, onde k e um inteiro.

Problema 15. (Bulgaria 1995) Encontre todos os primos p e q tais que o numero 2p + 2q

seja divisıvel por pq.

Problema 16. Mostre que se k > 1 entao 2k−1 6≡ −1 (mod k)

Problema 17. Mostre que se 3 ≤ d ≤ 2n−1, entao d ∤ (a2n

+1) para qualquer inteiro positivoa.

Problema 18. (Eureka) Prove que se p e um primo da forma 4k+3, entao 2p+1 tambeme primo se e somente se 2p+ 1 divide 2p − 1.

Problema 19. Prove que todos os divisores dos numeros de Fermat 22n

+ 1, n > 1, sao daforma 2n+2k + 1.

Problema 20. (IMO 1990) Encontre todos os inteiros positivos n > 1 tais que

2n + 1

n2

e um inteiro.

Problema 21. (Teste Cone Sul 2002) Encontre o perıodo na representacao decimal de1

32002.

3

Page 103: OBM - Teoría de Números.pdf

Problema 22. (Teste de Selecao do Ira para a IMO) Seja a um natural fixo. Mostre que oconjunto dos divisores primos de 22

n

+ a, para n ∈ N, e infinito.

Problema 23. (Colombia 2009) Encontre todas as triplas de inteiros positivos (a, b, n) quesatisfazem a equacao:

ab = 1 + b+ . . . + bn.

Problema 24. (IMO 2003) Seja p um numero primo. Demonstre que existe um numeroprimo q tal que, para todo inteiro n, o numero np − p nao e divisıvel por q.

4

Page 104: OBM - Teoría de Números.pdf

Polos Olímpicos de TreinamentoCurso de Teoria dos Números - Nível 2Prof. Samuel Feitosa

Aula 17

Aula de Revisao e Aprofundamento

Exemplo 1. O mınimo multiplo comum dos inteiros a, b, c, d e d e igual a a + b + c + d.Prove que abcd e divisıvel por 3 ou por 5.

Solucao: Suponha inicialmente que mdc(a, b, c, d) = 1 e seja L = a+ b+ c+ d. Como L eo mınimo multiplo comum, existem x, y, z, w tais que aw = bx = cy = dz = L. E facil verque L tambem e o mınimo multiplo comum de x, y, z, w e que

1

x+

1

y+

1

z+

1

w=

a

L+

b

L+

c

L+

d

L= 1.

Suponha sem perda de generalidade que w ≤ x ≤ y ≤ z. Da equacao anterior, o maiorvalor de w e 4 e ocorrendo igualdade deverıamos ter a = b = c = d = 1 que nao satisfazo enunciado. Para w = 3, o leitor podera facilmente verificar que a = b = 4, c = 3e d = 1 e a unica solucao. Para w = 2, temos as seguintes solucoes: (a, b, c, d) =(5, 2, 2, 1), (10, 5, 4, 1), (6, 4, 1, 1) , (9, 6, 2, 1), (12, 8, 3, 1), (21, 14, 6, 1). Em todos os casos, Le divisıvel por 3 ou 5.

Exemplo 2. Seja A = {a1 < a2 < a3 < · · · } uma sequencia crescente de inteiros positivosem que o numero de fatores primos de cada termo, contando fatores repetidos, nunca emaior que 2007. Prove que e sempre possıvel extrair do conjunto A um subconjunto infinito

B = {b1 < b2 < b3 < · · · }

tal que o maximo divisor comum entre bi e bj e sempre o mesmo para quaisquer naturaisi 6= j.

O numero de primos usados como fatores dos ai nao pode ser finito, pois se essa quantidadee n, terıamos apenas n2007 possıveis elementos ai. Podem ocorrer duas situacoes:

a) Nenhum primo divide infinitos ai. Seja b1 = a1. Como cada fator primo de a1aparece um numero finito de vezes como fator dos ai, existira um termo al > a1 tal que a1e al sao primos entre si. Seja b2 = al. Esse argumento pode ser repetido para gerar umsubconjunto infinito B = {b1 < b2 < b3 < · · · } de modo que mdc(bi, bj) = 1, quaisquer que

Page 105: OBM - Teoría de Números.pdf

sejam os naturais i, j.

b) Existe um primo p que divide infinitos ai. Como o expoente de p1 = p em cadaai que e multiplo de p1 e um elemento do conjunto {1, . . . , 2007}, pelo menos um deles,digamos r1 > 0, devera ocorrer infinitas vezes. Seja A1 o conjunto de todos os termosai para os quais o expoente de p1 em ai e r1. Se nenhum primo p2 6= p1 divide infinitoselementos de A1, o caso anterior mostra que A1 possui um subconjunto tal que o maximodivisor comum de quaisquer dois elementos e p1

r1 . Senao, seja p2 6= p1 um primo que divideinfinitos elementos de A1. O expoente de p2 em cada elemento de A1 que e multiplo de p2pertence ao conjunto {1, . . . , 2007 − r1}. Sejam r2 > 0 um expoente que ocorre infinitasvezes e A2 o conjunto dos elementos de A1 para os quais p1 e p2 tem expoentes r1 e r2,respectivamente. Esse processo deve terminar em i passos, i ≤ 2007, e nessa situacao Ai

e um subconjunto de A para o qual todo elemento e um multiplo de P = pr11 pr22 · · · prii ,digamos

Ai = {Pc1 < Pc2 < Pc3 < · · · },

onde C={c1 < c2 < c3 < · · · } e um conjunto em que cada termo e o produto de nao maisque 2007− (r1 + r2 + · · ·+ ri) fatores primos, nenhum dos quais ocorrendo infinitas vezes.Pelo caso a), C possui um subconjunto B1 tal que quaisquer dois elementos sao primosentre si. O conjunto

B = P ·B1 = {Px |x ∈ B1}

satisfaz as condicoes do problema, pois o maximo divisor comum entre quaisquer dois deseus elementos e igual a P .

Exemplo 3. (Seletiva Rioplatense 2001) Encontre todos os pares (m,n) de numeros naturaiscom m < n tais que m2 + 1 e um multiplo de n e n2 + 1 e um multiplo de m.

Afirmamos que todas as solucoes sao da forma (F2k−1, F2k+1), k ≥ 0 (Fn e o n-esimotermo da sequencia de Fibonacci). E facil ver que F2k−1F2k+3 = F 2

2k+1 + 1 e portantoos pares anteriores sao solucoes. Seja P o conjunto das solucoes que nao sao da forma(F2k−1, F2k+1). O conjunto P contem um par (a, b) tal que a+ b e mınimo. Suponhamosa < b(se a = b ⇒ (a, b) = (1, 1) = (F−1, F1) 6∈ P ). Como b | a2 + 1, a2 + 1 = bb′

e b′ < a. E facil ver que a | b′2 + 1 e b′ | a2 + 1. Logo (b′, a) e uma solucao comb′ + a < a + b Entretanto, (b′, a) 6∈ P e daı (b′, a) = (F2k−1, F2k+1). COnsequentemente,F2k−1b = b′b = a2 + 1 = F 2

2k+1 ⇒ b = F2k+3 ⇒ (a, b) = (F2k+1, F2k+3) 6∈ P . Logo P deveser vazio.

Exemplo 4. (URSS 1988) A sequencia de inteiros an e dada por a0 = 0, an = P (an−1),onde P (x) e um polinomio cujos coeficientes sao inteiros positivos. Mostre que para quais-quer inteiros positivos m,k com maximo divisor comum d, o maximo divisor comum de ame ak e ad.

Quando temos um polinomio com coeficientes inteiros e sempre bom lembrar que a − b |P (a)− P (b). Essa sera nossa principal ferramenta nesta solucao.

2

Page 106: OBM - Teoría de Números.pdf

1. am | amr. Provaremos por inducao. Se am(r−1) ≡ 0 (mod am) ⇒ am(r−1)+1 ≡ P (0)(mod am) ⇒ am(r−1)+2 ≡ P (P (0)) (mod am) ⇒ amr = am(r−1)+m ≡ P (P (. . . (P (0))

︸ ︷︷ ︸

m vezes

=

am ≡ 0 (mod am).

2. Se l | at e l | af ⇒ l | at−f (Supondo t > f). (Deixaremos a prova dessa afirmacaopara o leitor).

Pelo teorema de Bezout, existem inteiros positivos x, y tais que mx − ky = d. Seja n =mdc(am, ak). Como n | am | amx e n | ak | aky, pelo item 2, n | amx−ky = ad. Mas ad | ame ad | ak, entao ad | n. Portanto ad = n.

Exemplo 5. Prove que existem infinitos numeros compostos n para os quais

n | 3n−1 − 2n−1.

Lema: 2t | 32t

− 1 ∀t ∈ N

Vamos provar o lema por inducao. Para t = 1 e trivial. Suponha que a afirmacaoseja valida para t = r, provemos que tambem e valida para t = r + 1. Fatorando22

t+1

− 33t+1

= (22t

− 33t

)(22t

+ 33t

), como o primeiro parentesis e multiplo de 2t e osegundo e multiplo de 2, o produto deles e multiplo de 2t+1.

Seja n = 32t

− 22t

com t > 1. Como n − 1 ≡ (32t

− 1) − 22t

≡ 0 (mod 2t), pelo lema,obtemos n = 32

t

− 22t

| 3n−1 − 2n−1. Aqui estamos usando o fato que xk − yk | xmk − ymk.

Exemplo 6. Consideramos todas as sequencias (xn)n≥1 de inteiros positivos satisfazendo

xn+2 = mdc(xn, xn+1) + 2008, ∀n ≥ 1.

Alguma dessas sequencias contem exatamente 102008 numeros distintos?

A ideia e construir a sequencia de tras para frente. Mostraremos que para qualquer inteiropositivo k > 1, existe uma tal sequencia contendo exatamente k numeros distintos. Bastaencontrarmos uma sequencia que satisfaca:

xn+2 = mdc(xn, xn+1) + 2, ∀n ≥ 1. (1)

pois a multiplicacao de todos os termos por 1004 produz a sequencia do problema. De-

finamos a1 = 4, a2 = 6, a3 = 8, a4 = 2(a3 − 2) e an =(an−1 − 2)(an−2 − 2)

2para n ≥ 5.

E facil ver por inducao que todos os ai serao pares e que a sequencia e crescente. Con-sequentemente, todos os ai com i ≥ 1 sao inteiros e distintos. Alem disso, definamos ostermos com ındices nao positivos por a3k = 4, a3k−1 = 6, a3k−2 = 4 para k ≤ 0.

. . . a6, a5, a4, a3 = 8, a2 = 6, a1 = 4, a0 = 4, a−1 = 6, a−2 = 4, a−3 = 4, a−4 = 6, . . .

Afirmamos que a sequencia anterior satisfaz mdc(an+2, an+1)+2 = an ∀n ∈ Z. E imediatoverificar isso para n ≤ 1 Para n = 2, mdc(a4, a3) + 2 = mdc(12, 8) + 2 = 6 = a2. Paran > 2,

mdc(an+2, an+1)+2 = mdc((an+1 − 2)(an − 2)

2, an+1)+2 = mdc((an+1)

an − 2

2−an+2, an+1)+2

3

Page 107: OBM - Teoría de Números.pdf

Portanto,

mdc(an+2, an+1) + 2 = mdc(an − 2, (an − 2)an−1 − 2

2) + 2 = an − 2 + 2 = an.

Para encontrarmos uma sequencia satisfazendo (1) com exatamente k > 1 termos distintosbasta escolhermos x1 = ak e x2 = ak−1 e definirmos o resto da sequencia usando a relacaode recorrencia xn+2 = mdc(xn, xn+1) + 2

Problemas Propostos

Problema 7. Resolva em inteiros a equacao:

xy

z+

xz

y+

yz

x= 3

Exemplo 8. Alguns inteiros positivos estao escritos no quadro. Podemos apagar quaisquerdois inteiros distintos e substituı-los pelo maximo divisor comum e o mınimo divisor comumdos dois numeros. Prove que eventualmente a operacao de alterar os numeros nao sera maisexecutada.

Exemplo 9. Mostre que se k > 1 entao 2k−1 6≡ −1 (mod k)

Exemplo 10. (Estonia 2005) Sejam a, b inteiros positivos primos entre si tais que (a +b)/(a−b) e um inteiro positivo. Prove que ao menos um dentre os numeros ab+1 e 4ab+1e um quadrado perfeito.

Exemplo 11. (Ibero 1999) Seja n um inteiro maior que 10 tal que cada um de seus dıgitospertence ao conjunto S = {1, 3, 7, 9}. Prove que n tem algum divisor primo maior ou iguala 11.

4